12th Standard CBSE Chemistry Study material & Free Online Practice Tests - View Model Question Papers with Solutions for Class 12 Session 2020 - 2021
CBSE [ Chapter , Marks , Book Back, Creative & Term Based Questions Papers - Syllabus, Study Materials, MCQ's Practice Tests etc..]

Chemistry Question Papers

Class 12th Chemsitry - Chemistry in Everyday Life Case Study Questions and Answers 2022 - 2023 - by Study Materials - View & Read

  • 1)

    Medicines are used to cure diseases. Disinfectants are used on non-living objects whereas antiseptics are used on living tissues and are not harmful to living tissues. Antimicrobials are used to kill bacteria, virus and other micro-organisms. Anitbiotics are most important. Antifertility drugs are used to prevent birth control so as to control population which is essential. Antacids are used to treat hyperacidity in stomach.
    Diabetic patients are increasing day by day so we .. need artificial sweeteners Aspartame is unstable at cooking temperature. Alitame is stable on heating but its sweetness can't be control. Food preservatives are used in large amount so as to increase shelf life. Sugar and salt are most common preservatives. Antioxidants are added to prevent spoilage of food materials by oxidation. Soaps and detergents are widely used cleansing agents. Soaps are biodegradable where as some detergents having branched chain are not biodegradable. Detergents are more effective than soaps and work well even in hard water where as soaps do not work with hard water. Shaving soaps contain glycerol. Anionic detergents are used in tooth paste, cationic detergents are used in hair conditioners and non-ionic detergents are used in dishwashing liquids.
    (a) Name the antiseptic which is added to soap.
    (b) Name the sweetening agent used in making sweets for diabetic patients.
    (c) Which structural unit present in detergent make it non-biodegradable?
    (d) Name a substance which is used as antiseptic as well as disinfectant.
    (e) Why do shaving soaps have glycerol?
    (f) Name an antihistamin which acts as antacid.
    (g) Name a preservative used in cold drinks.

  • 2)

    Drugs are important parts of our life. The alkaloid reserpine (Isolated from Rauwolfia serpentinay is a powerful tranquillizer. Morphine, codeine are Narcotic analgesics used in acute pain in cancer, child birth. Non-narcotics Antipyretics and analgesics like aspirin, paracetamol etc are widely used.
    Chloramphenicol, a wide spectrum antibiotic is used for treatment of typhoid, bacteriostatic antibiotics like tetracycline, erythromycin are widely used. Penicilline, Aminoglycoside, Ofloxacin are bactericidal antibiotics. Antibacterial sulpha drugs like sulphadiazine, used for urinary and respiratory infections. Antihistamines are anti-inflammatory drugs. Some antihistamine like Ranitidine is used as antacids.
    (a) What are tranquilizer? Give one example.
    (b) Why is penicilline narrow (limited) spectrum antibiotics?
    (c) What is difference between bacteriocidal and bacteriostatic antibioitics?
    (d) Name the antibiotics used for treatment of T.B.
    (e) Give one example of Antihistamin.

  • 3)

    Observe the table in which pH of various soaps, shampoo, facewash are given. Study the table and answer the questions that follow based on table and related studied concepts.

    S. No Soap pH
    1 Dove 6
    2 Caress, oil of olay 7
    3 Lever 2000, Lava 9
    4 Camay, Dial Yardley, Nivea 10
    5 Baby Soap 5.5
    6 Shampoo 6 to 8
    7 Face wash 4 to 7

    (a) Which pH range of soap is harmful for skin?
    (b) Why is Dove is better than other soaps?
    (c) Why do baby soaps have pH 5.5?
    (d) Why is face wash better than soaps?
    (e) Which chemical is added to transparent soaps and why?
    (f) Which range for shampoo is good?
    (g) What is advantage of cationic detergents used in shampoo and hair Conditioners?
    (h) Which type of detergent is used in tooth paste? Why?
    (i) Name the detergent present in most of tooth paste. Which compound is present in sensitive tooth paste?

Class 12th Chemsitry - Polymers Case Study Questions and Answers 2022 - 2023 - by Study Materials - View & Read

  • 1)

    Polymers are backbone of modern civilisation. We are using polymers in our daily life to lot of extent. Automobile tyres are made up of Buna-S. Nylon is being extensively used for making socks and stockings, climbing ropes, fishing nets. Bakelite is used for making combs and electrical switches. Teflon is coated to make non-stick kitchen wares. Plastic eating bacteria is capable of breaking down poly (ethylene phthalate) (PET) and consuming as a sole carbon and energy source. Single use plastics are quite harmful. Biotlegradable polymers like PHBV, Nylon-2, Nylon-6, Polylactic acid, poly glycollic acid have been discovered which are very useful and ecofriendly.
    (a) Name a biodegradable polymer used in control during release and orthopaedic devices.
    (b) Name a biodegradable step growth polymer used for sutures after surgery and give its monomers .
    (c) How can plastics be converted into fuels?
    (d) Can we use polythene in making roads?
    (e) Why is bakelite used for electrical switches and switch boards?

  • 2)

    Study the table related to tensile strength of polymers and answers the questions based on table and related concepts.

    Polymer Tensile Strength (MPa)
    Polycarbonate 70
    HDPE 15
    Polyethylene Terephthalate 55
    Polyamide 85
    Nylon 82.7

    (a) Arrange Nylon, polyester, cotton, wool and silk in weaving order of tensile strength.
    (b) Give repeating unit of Nylon-66
    (c) Which type of polymers are used in bullet proof vests?
    (d) Why do thermoplastic soft on heating?
    (e) What is repeating unit of Terylene?

  • 3)

    Observe the graph between stress and strain for some types of plastics and answer the questions based on graph and related studied concepts.

    (a) Which type of polymers have more elasticity but less tensile strength?
    (b) Why is High density Polythene (HDP) is tough whereas LDP (low density polythene) Flexible?
    (c) Which property of teflon makes it suitable for non-stick kitchen wares?
    (d) Why is Nylon used in parachute fabrics?
    (e) Name the polymer used in making unbreakable Crockery.
    (f) What type of polymer is urea formaldehyde resin?
    (g) Name an elastomer which is non-inflammable used in making conveyor belts in coal mines.
    (h) Name an elastomer used for automobile tyres.
    (i) How can we increase tensile strength of Natural rubber?
    (j) Arrange Nylon-6,6, Polythene, Buna-N in increasing order of forces of attraction.

Class 12th Chemsitry - Biomolecules Case Study Questions and Answers 2022 - 2023 - by Study Materials - View & Read

  • 1)

    Read the passage given below and answer the following questions:
    Pentose and hexose undergo intramolecular hemiacetal or hemiketal formation due to combination of the -OH group with the carbonyl group. The actual structure is either of five or six membered ring containing an oxygen atom. In the free state all pentoses and hexoses exist in pyranose form (resembling pyran). However, in the combined state some of them exist as five membered cyclic structures, called furanose (resembling furan).

    The cyclic structure of glucose is represented by Haworth structure:

    \(\alpha \) and \(\beta\) -D-glucose have different configuration at anomeric (C-l) carbon atom, hence are called anomers and the C-l carbon atom is called anomeric carbon (glycosidic carbon).
    The six membered cyclic structure of glucose is called pyranose structure.
    The following questions are multiple choice questions. Choose the most appropriate answer:
    (i) \(\alpha\) -D(+)-glucose and \(\beta\) -D( +)glucose are

    (a) enantiomers (b) conformers (c) epimers (d) anomers

    (ii) The following carbohydrate is

    (a) a ketohexose (b) an aldohexose
    (c) an \(\alpha \)-furanose (d) an \(\alpha \)-pyranose

    (iii) In the following structure,

    anomeric carbon is

    (a) C-l (b) C-2 (c) C-3 (d) C-4

    (iv) The term anomers of glucose refers to

    (a) isomers of glucose that differ in configurations at carbons one and four (C-l and C-4)
    (b) a mixture of (D)-glucose and (L)-glucose
    (c) enantiomers of glucose
    (d) isomers of glucose that differ in configuration at carbon one (C-l).
  • 2)

    Read the passage given below and answer the following questions:
    When a solution of an \(\alpha \) -amino acid is placed in an electric field depending on the pH of the medium, following three cases may happen.

    (i) In alkaline solution,\(\alpha \) -amino acids exist as anion II, and there is a net migration of amino acid towards the anode.
    (ii) In acidic solution, \(\alpha \) -amino acids exist as cation III, and there is a net migration of amino acid towards the cathode.
    (iii) If II and III are exactly balanced there is no net migration; under such conditions anyone molecule exists as a positive ion and as a negative ion for exactly the same amount of time, and any small movement in the direction of one electrode is subsequently cancelled by an equal movement back toward the other electrode. The pH of the solution in which a particular amino acid does not migrate under the influence of an electric field is called the isoelectric point of that amino acid.
    The following questions are multiple choice questions. Choose the most appropriate answer:
    (i) 
    Arrange in order of increasing acid strengths.

    (a) X>Z>Y (b) Z (c) X>Y>Z (d) Z>X>Y

    (ii) In aqueous solutions, amino acids mostly exist as

    \(\text { (a) } \mathrm{NH}_{2}-\mathrm{CH} R-\mathrm{COOH}\) \(\text { (b) } \mathrm{NH}_{2}-\mathrm{CH} R-\mathrm{COO}^{-}\)
    \(\text { (c) } \mathrm{NH}_{3} \mathrm{CH} R \mathrm{COOH}\) \(\text { (d) } \mathrm{H}_{3} \mathrm{NCH} R \mathrm{COO}^{-}\)

    (iii) Amino acids are least soluble

    (a) at pH 1 (b) at pH 7
    (c) at their isoelectric points (d) none of these.

    (iv) The \(pK_{a_{1}}\)and \(pK_{a_{2}}\) of an amino acid are 2.3 and 9.7 respectively. The isoelectric point of the amino acid is

    (a) 12.0 (b) 7.4 (c) 6.0 (d) 3.7
  • 3)

    Read the passage given below and answer the following questions:
    Carbohydrates can exist in either of two conformations, as determined by the orientation of the hydroxyl group about the asymmetric carbon farthest from the carbonyl.

    By convention, a monosaccharide is said to have D-configuration if the hydroxyl group attached to the asymmetric carbon atom adjacent to the - CH2OH group is on the right hand side irrespective of the positions of the other hydroxyl groups. On the other hand, the molecule is assigned L-configuration if the - OH group attached to the carbon adjacent to the - CH2OH group is on the left hand side.
    The following questions are multiple choice questions. Choose the most appropriate answer:
    (i) D-Glyceraldehyde and L-Glyc~raldehyde are

    (a) epimers (b) enantiomers
    (c) anomers (d) conformational diasteriomers

    (ii) Which of the following monosaccharides, is the majority found in the human body?

    (a) D-type (b) L-type (c) Both of these (d) None of these

    (iii) Monosaccharides contain

    (a) always six carbon atoms (b) always five carbon atoms
    (c) always four carbon atoms (d) may contain 3 to 7 carbon atoms

    (iv) The correct corresponding order of names of four aldoses with configuration given below

    respectively, is

    (a) L-erythrose, L-threose, L-erythrose, D-threose
    (b) D-threose, D-erythrose, L-tl?repse, L-erythrose
    (c) L-erythrose, L-threose, D-erythrose, D-threose
    (d) D-erythrose, D-threose, L-erythrose, L-threose.
  • 4)

    Read the passage given below and answer the following questions:
    Carbohydrates are polyhydroxy aldehydes and ketones and those compounds which on hydrolysis give such compounds are also carbohydrates. The carbohydrates which are not hydrolysed are called monosaccharides. Monosaccharides with aldehydic group are called aldose and those which free ketonic groups are called ketose. Carbohydrates are optically active. Number of optical isomers = 2n
    Where n = number of asymmetric carbons. Carbohydrates are mainly synthesised by plants during photosynthesis.
    The monosaccharides give the characteristic reactions of alcohols and carbonyl group (aldehydes and ketones). It has been found that these monosaccharides exist in the form of cyclic structures. In cyclization, the -OH groups (generally C5 or C4 in aldohexoses and C5 or C6 in ketohexoses) combine with the aldehyde or keto group. As a result, cyclic structures of five or six membered rings containing one oxygen atom are formed, e.g., glucose forms a ring structure. Glucose contains one aldehyde group, one 1o alcoholic group and four 2o alcoholic groups in its open chain structure.
    The following questions are multiple choice questions. Choose the most appropriate answer:
    (i) First member of ketos sugar is

    (a) ketotriose (b) ketotetrose (c) ketopentose (d) ketohexose

    (ii) In CH2OHCHOHCHOHCHOHCHOHCHO, the number of optical isomers will be

    (a) 16 (b) 8 (c) 32 (d) 4

    (iii) Some statements are given below:
    1. Glucose is aldohexose.
    2. Naturally occurring glucose is dextrorotatory.
    3. Glucose contains three, chiral centres.
    4. Glucose contains one 1o alcoholic group and four 2o alcoholic groups.
    Among the above, correct statements are

    (a) 1 and 2 only (b) 3 and 4 only
    (c) 1,2 and 4 only (d) 1,2,3 and 4

    (iv) Which of the following reactions of glucose can be explained only by its cyclic structure?

    (a) Glucose forms cyanohydrin with HCN
    (b) Glucose reacts with hydroxylamine to form an oxime
    (c) Pentaacetate of glucose does not react with hydroxylamine
    (d) Glucose is oxidised by nitric acid to gluconic acid .
  • 5)

    Read the passage given below and answer the following questions:
    When a protein in its native form, is subjected to physical changes like change in temperature or chemical changes like change in pH, the hydrogenbonds are disturbed. Due to this, globules unfold and helix get uncoiled and protein loses its biological activity. This is called denaturation of protein.
    The denaturation causes change in secondary and tertiary structures but primary structures remains intact. Examples of denaturation of protein are coagulation of egg white on boiling, curdling of milk, formation of cheese when an acid is added to milk.
    The following questions are multiple choice questions. Choose the most appropriate answer:
    (i) Mark the wrong statement about denaturation of proteins

    (a) The primary structure of the protein does not change
    (b) Globular proteins are converted into fibrous proteins.
    (c) Fibrous proteins are converted into globular proteins.
    (d) The biological activity of the protein is destroyed.

    (ii) Which structure(s) of proteins remains(s) intact during denaturation process?

    (a) Both secondary and tertiary structures (b) Primary structure only
    (c) Secondary structure only (d) Tertiary structure only

    (iii) Cheese is a

    (a) globular protein (b) conjugated protein
    (c) denatured protein (d) derived protein

    (iv) Secondary structure of protein refers to

    (a) mainly denatured proteins and structure of prosthetic groups
    (b) three-dimensional structure, especially the bond between amino acid residues that are distant from each other in the polypeptide chain
    (c) linear sequence of amino acid residues in the polypeptide chain
    (d) regular folding patterns of continuous portions of the polypeptide chain

Class 12th Chemsitry - Amines Case Study Questions and Answers 2022 - 2023 - by Study Materials - View & Read

  • 1)

    Read the passage given below and answer the following questions:
    When the mixture contains the three amine salts (1°, 2° and 3°) along with quaternary salt, it is distilled with KOH solution. The three amines distill, leaving the quaternary salt unchanged in the solution. Then the mixture of amines is separated by fractional distillation, Hinsbergs method and Hoffmann's method.

    The following questions are multiple choice questions. Choose the most appropriate answer:
    (i) Hinsberg reagent is

    (a) aliphatic sulphonyl chloride (b) phthalamide
    (c) aromatic sulphonyl chloride (d) anhydrous ZnCl2 + conc. HCl.

    (ii) Primary amine with Hinsberg's reagent forms

    (a) N-alkyl benzene sulphonamide soluble in KOH solution
    (b) N-alkyl benzene sulphonamide insoluble in KOH solution
    (c) N, N-dialkyl benzene sulphonamide soluble in KOH solution
    (d) N, N-dialkyl benzene sulphonamide insoluble in KOH solution.

    (iii) To separate amines in a mixture Hoffmann's method is used. The Hoffmann's reagent is

    (a) benzenesulphonyl chloride (b) diethyloxalate
    (c) benzeneisocyanide (d) p-toulenesulphonic acid.

    (iv) 3o amines with Hinsberg's reagent give

    (a) no reaction (b) product which is same as that of 10 amine
    (c) product which is same as that of 2° amine (d) products which is a quaternary salt.
  • 2)

    Read the passage given below and answer the following questions:
    A mixture of two aromatic compounds (A) and (B) was separated by dissolving in chloroform followed by extraction with aqueous KOH solution. The organic layer containing compound (A), when heated with alcoholic solution of KOH produce C7H5N (C) associated with unpleasant odour.
    The following questions are multiple choice questions. Choose the most appropriate answer:
    The reaction of (A) with alcoholic solution of KOH to produce (C) of unpleasant odour is called

    (a) Sandmeyer reaction (b) Carbylamine reaction
    (c) Ullmann reaction (d) Reimer-Tiemann reaction

    (ii) The alkaline aqueous layer (B) when heated with chloroform and then acidified give a mixture of isomeric compounds of molecular formula C7H6O2. (B) is

    (a) C6H5CHO (b) C6H5COOH (c) C6H5CH3 (d) C6H5OH

    (iii) In the chemical reaction, \(\mathrm{CH}_{3} \mathrm{CH}_{2} \mathrm{NH}_{2}+\mathrm{CHCl}_{3}+3 \mathrm{KOH} \longrightarrow(A)+(B)+3 \mathrm{H}_{2} \mathrm{O},\) the compounds (A) and (B)  are respectively

    (a) C2H5NC and KCI (b) C2H5CN and KCI
    (c) CH3CH2CONH2 and KCI (d) C2H5NC and K2CO3

    (iv) Direct nitration of an aromatic compound (A) is not feasible because

    (a) the reaction cannot be stopped at the mononitration stage
    (b) a mixture of o, m and p-nitroaniline is always obtained
    (c) nitric acid oxidises most of the aromatic compound to give oxidation products along with only a small amount of nitrated products

    (d) all of the above

  • 3)

    Read the passage given below and answer the following questions:
    The amines are basic in nature due to the presence of a lone pair of electron on N-atom of the -NH2 group, which it can donate to electron deficient compounds. Aliphatic amines are stronger bases than NH3 because of the +1 effect of the alkyl groups. Greater the number of alkyl groups attached to N-atom, higher is the electron density on it and more will be the basicity. Thus, the order of basic nature of amines is expected to be 3° > 2° > 1°, however the observed order is 2° > 1° > 3°. This is explained on the basis of crowding on N-atom of the amine by alkyl groups which hinders the approach and bonding by a proton, consequently, the electron pair which is present on N is unavailable for donation and hence 3° amines are the weakest bases. Aromatic amines are weaker bases than ammonia and aliphatic amines. Electron -donating groups such as -CH3 , -OCH3 , etc. increase the basicity while electron-withdrawing substitutes such as -NO2 , -CN, halogens, etc. decrease the basicity of amines. The effect of these substituents is more at p than at m-positions.
    The following questions are multiple choice questions. Choose the most appropriate answer :
    (i) Which one of the following is the strongest base in aqueous solution?

    (a) Methyl amine (b) Trimethyl amine (c) Aniline (d) Dimethyl amine

    (ii) Which order of basicity is correct?

    (a) Aniline> m-toluidine > o-toluidine (b) Aniline> o-toluidine > m-toluidine (c) o-toluidine> aniline> m-toluidine (d) o-toluidine < aniline < m-toluidine

    (iii) What ts the decreasing order of basicity of primary, secondary and tertiary ethylamines and NH3?

    (a) NH3 > C2H5NH2 > (C2H5)2NH > (C2H5)3N (b) (C2H5)3N> (C2H5)2NH > C2H5NH2 > NH3
    (c) (C2H5)2NH >C2H5NH2 > (C2H5)3N > NH3 (d) (C2H5)2NH> (C2H5)3N > C2H5NH2 > NH3

    (iv) Choose the correct statement.

    (a) Methylamine is slightly acidic. (b) Methylamine is less basic than ammonia. (c) Methylamine is a stronger base than ammonia. (d) Methylamine forms salts with alkalie
  • 4)

    Read the passage given below and answer the following questions:
    Amines are alkyl or aryl derivatives of ammonia formed by replacement of one or more hydrogen atoms. Alkyl derivatives are called aliphatic amines and aryl derivatives are known as aromatic amines. The presence of aromatic amines can be identified by performing dye test. Aniline is the simplest example of aromatic amine. It undergoes electrophilic substitution reactions in which - NH2 group strongly activates the aromatic ring through delocalisation oflone pair of electrons of N-atom. Aniline undergoes electrophilic substitution reactions. Ortho and para positions to the -NH2 group become centres of high electrons density. Thus, -NH2 group is ortho and para-directing and powerful activating group. The following questions are multiple choice questions.
    Choose the most appropriate answer:
    (i) Cyclohexylamine and aniline can be distinguished by

    (a) Hinsberg test (b) carbylamine test (c) Lassaigne test (d) azo dye test

    (ii) Which of the following compounds gives-dye test?

    (a) Aniline (b) Methyl amine (c) Diphenyl amine   (d) Ethyl amine

    (iii) Oxidation of aniline with manganese dioxide and sulphuric acid produces

    (a) phenylhydroxylamine (b) nitrobenzene (c) p-benzoquinone   (d) phenol.

    (iv) Aniline when treated with conc, HNO3 and H2SO4 gives

    (a) phenylhydroxylamine (b) m-nitroaniline (c) p-benzoquinone  (d) nitrobenzene.
  • 5)

    Read the passage given below and answer the following questions:
    Amines are basic in nature. The basic strength of amines can be expressed by their dissociation constant, Kb or pKb.
    \(\mathrm{RNH}_{2}+\mathrm{H}_{2} \mathrm{O} \rightleftharpoons \mathrm{RNH}_{3}^{+}+\mathrm{OH}^{-}\)
    \(K_{b}=\frac{\left[R \mathrm{NH}_{3}^{+}\right]\left[\mathrm{OH}^{-}\right]}{\left[R \mathrm{NH}_{2}\right]} \text { and } \mathrm{p} K_{b}=-\log K_{b}\)

    Greater the Kb value or smaller the pKb value, more is the basic strength of amine. Aryl amines such as aniline are less basic than allphatic amines due to the involvement of lone pair of electrons on N-atom with the resonance in benzene. In derivatives of aniline, the electron releasing groups increase the basic strength while electron withdrawing groups decrease the basic strength. The base weakening effect of electron withdrawing group and base strengthening effect of electron releasing group is more marked at p-position than at m-position. o-Substituted aniline is less basic than aniline due to ortho effect and is probable due to combination of electronic and steric effect.
    The following questions are multiple choice questions.Choose the most appropriate answer :
    (i) Which of the following has lowest pKb value?

    (ii) The strongest base among the following is

    (a) C6H5NH2 (b) p-NO2 - C6H4CH2NH2
    (c) m-NO2 - C6H4NH2 (d) C6H5NH2

    (iii) Maximum pKb value of

    (c) (CH3CH2)2NH (d) (CH3)2NH

    (iv) Which of the following statements is not correct?

    (a) Methylamine is more basic than NH3 (b) Amines form hydrogen bonds.
    (c) Ethylamine has higher boiling point than propane. (d) Dimethylamine is less basic than methylamine.

Class 12th Chemsitry - Aldehydes , Ketones and Carboxylic Acids Case Study Questions and Answers 2022 - 2023 - by Study Materials - View & Read

  • 1)

    Read the passage given below and answer the following questions:
    The addition reaction of enol or enolate to the carbonyl functional group of aldehyde or ketone is known as aldol addition. The \(\beta\)-hydroxyaldehyde or \(\beta\)-hydroxyketone so obtained undergo dehydration in second step to produce a conjugated enone. The first part of reaction is an addition reaction and the second part is an elimination reaction. Carbonyl compound having \(\alpha\)-hydrogen undergoes aldol condensation reaction.

    The following questions are multiple choice questions. Choose the most appropriate answer:
    (i) Condensation reaction is the reverse of which of the following reaction?

    (a) Lock and key hypothesis (b) Oxidation
    (c) Hydrolysis (d) Glycogen formation

    (ii) Which of the following compounds would be the main product of an aldol condensation of acetaldehyde and acetone?

    (a) CH3CH=CHCHO (b) CH3CH=CHCOCH3
    (c) (CH3)2C=CHCHO (d) (CH3)2C=CHCOCH3

    (ii) Which combination of carbonyl compounds gives phenyl vinyl ketone by an aldol condensation?

    (a) Acetophenone and Formaldehyde (b) Acetophenone and acetaldehyde
    (c) Benzaldehyde and acetaldehyde (d) Benzaldehyde and acetone

    (iv) Which of the following will undergo aldol condensation?

    (a) HCHO (b) CH3CH2OH
    (c) C6H5CHO (d) CH3CH2CHO
  • 2)

    Read the passage given below and answer the following questions :
    When an aldehyde with no a-hydrogen reacts with concentrated aqueous NaOH, half the aldehyde is converted to carboxylic acid salt and other half is converted to an alcohol. In other words, half of the reactant is oxidized
    and other half is reduced. This reaction is known as Cannizzaro reaction 

    The following questions are multiple choice questions. Choose the most appropriate answer :
    (i) A mixture of benzaldehyde and formaldehyde on heating with aqueous NaOH solution gives

    (a) benzyl alcohol and sodium formate (b) sodium benzoate and methyl alcohol
    (c) sodium benzoate and sodium formate (d) benzyl alcohol and methyl alcohol.

    (ii) Which of the following compounds will undergo Cannizzaro reaction?

    (a) CH3CHO (b) CH3COCH3
    (c) C6H5CHO (d) C6H5CH2CHO

    (iii) Trichloroacetaldehyde is subjected to Cannizzaro's reaction by using NaOH. The mixture of the products contains sodium trichloroacetate ion and another compound. The other compounds is

    (a) 2, 2, 2-trichloroethanol (b) trichloromethanol
    (c) 2, 2, 2-trichloropropanol (d) chloroform

    (iv) Which of the following reaction will not result in the formation of carbon-carbon bonds?

    (a) Cannizzaro reaction (b) Wurtz reaction
    (c) Reimer- Tiemann reaction (d) Friedel-Crafts acylation
  • 3)

    Read the passage given below and answer the following questions
    A tertiary alcohol H upon acid catalysed dehydration gives a product I. Ozonolysis of I leads to compounds J and K. Compound J upon reaction with KOH gives benzyl alcohol and a compound L, whereas K on reaction with KOH gives only M.

    The following questions are multiple choice questions. Choose the most appropriate answer :
    (i) Compound H is formed by the reaction of

    (ii) The structures of compound J, Kand L, respectively, are

    (a) PhCOCH3 , PhCH2COCH3 and PhCH2COO-K+ (b) PhCHO, PhCH2CHO and PhCOO-K+
    (c) PhCOCH3, PhCH2CHO and CH3 COO-K+ (d) PhCHO, PhCOCH3 and PhCOO-K+

    (iii) When (J) is treated with acetic anhydride, in the presence of corresponding salt of an acid, the product obtained is

    (a) cinnamic acid (b) crotonic acid (c) maleic acid (d) benzylic acida

    (iv) Which of the following statements is correct for compound (K)?

    (a) It reacts with alkaline KMnO4 followed by acidic hydrolysis and forms benzoic acid.
    (b) It reacts with iodine and NaOH to form triiodomethane.
    (c) It is prepared by the reaction of benzene with benzoyl chloride in presence of anhydrous aluminium chloride
    (d) It reacts with freshly prepared ammoniacal silver nitrate solution
  • 4)

    Read the passage given below and answer the following questions :
    Carboxylic acids dissociate in water to give carboxylate ion and hydronium ion.
    RCOOH + H2O \(\longrightarrow\) RCOO- + H3O+
    The acidity of carboxyl group is due to the presence of positive charge on oxygen which liberates proton. The carboxylate ion formed is resonance stabilised.

    Carboxylic acids are stronger acids than phenols. Electron withdrawing groups (EWG) increase the acidity of carboxylic acids by stabilising the conjugate base through delocalisation of negative charge by inductive and/ or resonance effects. Electron donating group (EDG) decrease the acidity by destabilising the conjugate base.
    The following questions are multiple choice questions. Choose the most appropriate answer :
    (i) Which of the following reactions is showing the acidic property of carboxylic acid?

    (ii) Which one of the following is the correct order of acidic strength?

    (a) CF3COOH > CHCl2COOH > HCOOH > C6H5CH2COOH > CH3COOH
    (b) CH3COOH > HCOOH > CF3COOH > CHCl2COOH > C6H5CH2COOH
    (c) HCOOH > C6H5CH2COOH > CF3COOH > CHCl2COOH > CH3COOH
    (d) CF3COOH > CH3COOH > HCOOH > CHCl2COOH > C6H5CH2COOH

    (iii) Which of the following acids has the smallest dissociation constant?

    (a) CH3CHFCOOH (b) FCH2CH2COOH
    (c) BrCH2CH2COOH (d) CH3CHBrCOOH

    (iv) The correct order of acidity for the following compounds is

    (a) I > II > III > IV (b) III > I > II > IV
    (c) III> IV > II> I (d) I > III > IV > II
  • 5)

    Read the passage given below and answer the following questions:
    Aldehydes and ketones having acetyl group are oxidised by sodium hypohalate (NaOX) or halogen and alkali (X2 + OH-) to corresponding sodium salt having one carbon atoms less than the carbonyl compound and give a haloform.

    Sodium hypoiodite (NaOI) when treated with compounds containing CH3CO - group gives yellow precipitate of iodoform. Haloform reaction does not affect a carbon-carbon double bond present in the compound.
    The following questions are multiple choice questions. Choose the most appropriate answer:

    (a) Isopropyl alcohol (b) Propionaldehyde
    (c) Ethylphenyl ketone (d) Benzyl alcohol

    (ii) Which of the following compounds is not formed in iodoform reaction of acetone?

    (a) CH3COCH2I (b) ICH2COCH2I
    (c) CH3COCHI2 (d) CH3COCI3

    (iii) For the given set of reactions

    starting compound A corresponds to

    (iv) An organic compound 'A' has the molecular formula C3H6O. It undergoes iodoform test. When saturated with HCI it gives 'B' of molecular formula C9H14O. 'A' and 'B' respectively are

    (a) propanal and mesityl oxide (b) propanone and mesityl oxide
    (c) propanone and 2,6-dimethyl-2,5-hepta-dien-4-one (d) propanone and propionaldehyde

Class 12th Chemsitry - Alcohols , Phenols and Ethers Case Study Questions and Answers 2022 - 2023 - by Study Materials - View & Read

  • 1)

    Read the passage given below and answer the following questions:
    Although chlorobenzene is inert to nucleophilic substitution, however it gives quantitative yield of phenol when heated with aq. NaOH at high temperature and under high pressure. As far as electrophilic substitution in phenol is concerned the - OH group is an activating group, hence, its presence enhances the electrophilic substitution at o- and p-positions.
    The following questions are multiple choice questions. Choose the most appropriate answer:
    (i) Conversion of chlorobenzene into phenol involves

    (a) modified SN1 mechanism (b) modified SN2 mechanism
    (c) both (a) and (b) (d) elimination-addition mechanism.

    (ii) Phenol undergoes electrophilic substitution more readily than benzene because

    (a) the intermediate carbo cation is a resonance hybrid of more resonating structures than that from benzene
    (b) the intermediate is more stable as it has positive charge on oxygen, which can be better accommodated than on carbon
    (c) in one of the canonical structures, every atom (except hydrogen) has complete octet
    (d) the -OH group is o, p-directing which like all other o, p-directing group, is activating.

    (iii) Phenol on treatment with excess of conc. HNO3 gives

    (a) o-nitrophenol (b) p-nitrophenol
    (c) o-and p-nitrophenol (d) 2, 4, 6-trinitrophenol

    (iv) The major product of the following reaction is
     

  • 2)

    Read the passage given below and answer the following questions:
    A compound (X) containing C, Hand O is unreactive towards sodium. It also does not react with Schiff's reagent. On refluxing with an excess ofhydroiodic acid, (X) yields only one organic product (Y). On hydrolysis, (Y) yields a new compound (Z) which can be converted into (Y) by reaction with red phosphorus and iodine. The compound (Z) on oxidation with potassium permanganate gives a carboxylic acid. The equivalent weight of this acid is 60.
    The following questions are multiple choice questions. Choose the most appropriate answer:
    (i) The compound (X) is an

    (a) acid (b) aldehyde (c) alcohol (d) ether

    (ii) The IUPAC name of the acid formed is

    (a) methanoic acid (b) ethanoic acid (c) propanoic acid (d) butanoic acid.

    (iii) Compound (Y) is

    (a) ethyl iodide (b) methyl iodide (c) propyl iodide (d) mixture of (a) and (b).

    (iv) Compound (X) on treatment with excess of Cl2 in presence of light gives

    (a) \(\alpha\) - chlorodiethyl ether (b) \(\alpha\), \(\alpha\)' -dichlorodiethyl ether (c) perchlorodiethyl ether (d) none of these.
  • 3)

    Read the passage given below and answer the following questions:
    Both alcohols and phenols are acidic in nature, but phenols are more acidic than alcohols. Acidic strength of alcohols mainly depends upon the inductive effect. Acidic strength of phenols depends upon a combination of both inductive effect and resonance effects of the substituent and its position on the benzene ring. Electron withdrawing groups increases the acidic strength of phenols whereas electron donating groups decreases the acidic strength of phenols. Phenol is a weaker acid than carboxylic acid.
    The following questions are multiple choice questions. Choose the most appropriate answer:
    (i) Phenols are highly acidic as compare to alcohols due to

    (a) the higher molecular mass of phenols (b) the stronger hydrogen bonds in phenols
    (c) alkoxide ion is a strong conjugate base (d) phenoxide ion is resonance stabilised.

    (ii)  The correct decreasing order of pKa value is

    (a) II > IV > I > III (b) IV > II > III > I (c) III > II > IV > I (d) IV > I > II > III

    (iii) The compound that does not liberate CO2, on treatment with aqueous sodium bicarbonate solution is

    (a) benzoic acid (b) benzenesulphonic acid (c) salicylic acid (d) carbolic acid.

    (iv) Most acidic amongst the following is

  • 4)

    Read the passage given below and answer the following questions:
    Ethers are readily cleaved by HI or HBr at 373 K to form an alcohol and an alkyl halide.
    \(R-\mathrm{O}-R+\mathrm{HX} \stackrel{373 \mathrm{~K}}{\longrightarrow} R-X+R-\mathrm{OH}\)
    \(R-\mathrm{OH}+\mathrm{HX} \stackrel{373 \mathrm{~K}}{\longrightarrow} R-X+\mathrm{H}_{2} \mathrm{O}\)

    Mixed ether, containing primary or secondary alkyl group, when heated with hydrogen halide, the lower alkyl group forms halide and higher will form an alcohol. Tertiary alkyl ether when heated with hydrogen halide gives tertiary alkyl halide.
    The following questions are multiple choice questions. Choose the most appropriate answer :
    (i) Among the following ethers, which one will produce methyl alcohol on treatment with hot concentrated HI?

    (c) CH3-CH2-CH2-CH2-O-CH3

    (ii) When CH2=CH-O-CH2-CH3 reacts with one mole of HI, one of the products formed is

    (a) ethane (b) ethanol (c) iodoethene (d) ethanal

    (iii) (CH3)3COCH3 and CH3OC2H5 are treated with hydroiodic acid. The fragments obtained after reactions are respectively

    (a\(\left(\mathrm{CH}_{3}\right)_{3} \mathrm{CI}+\mathrm{CH}_{3} \mathrm{OH} ; \mathrm{CH}_{3} \mathrm{I}+\mathrm{C}_{2} \mathrm{H}_{5} \mathrm{OH}\)

    (b) \(\left(\mathrm{CH}_{3}\right)_{3} \mathrm{CI}+\mathrm{CH}_{3} \mathrm{OH} ; \mathrm{CH}_{3} \mathrm{OH}+\mathrm{C}_{2} \mathrm{H}_{5} \mathrm{I}\)
    (c) \(\left(\mathrm{CH}_{3}\right)_{3} \mathrm{COH}+\mathrm{CH}_{3} \mathrm{I} ; \mathrm{CH}_{3} \mathrm{OH}+\mathrm{C}_{2} \mathrm{H}_{5} \mathrm{I}\) (d) \(\mathrm{CH}_{3} \mathrm{I}+\left(\mathrm{CH}_{3}\right)_{3} \mathrm{COH} ; \mathrm{CH}_{3} \mathrm{I}+\mathrm{C}_{2} \mathrm{H}_{5} \mathrm{OH}\)

    (iv) Which of the following ether is unlikely to be cleaved by hot conc. HBr?

  • 5)

    Read the passage given below and answer the following questions:
    An organic compound (A) having molecular formula C6H6O gives a characteristic colour with aqueous FeCl3 solution. (A) on treatment with CO2 and NaOH at 400 K under pressure gives (B), which on acidification gives a compound (C). The compound (C) reacts with acetyl chloride to give (D) which is a popular pain killer.
    The following questions are multiple choice questions. Choose the most appropriate answer:
    (i) Compound (A) is

    (a) 2-hexanol (b) dimethyl ether (c) phenol (d) 2-methyl pentanol.

    (ii) Number of carbon atoms in compound (D) is

    (a) 7 (b) 6 (c) 8 (d) 9

    (iii) The conversion of compound (A) to (C) is known as

    (a) Reimer- Tiemann reaction (b) Kolbe's reaction (c) Schimdt reaction (d) Swarts reaction

    (iv) Compound (A) on heating with compound (C) in presence of POCl3 gives a compound (D) which is used

    (a) in perfumery as a flavouring agent (b) as an antipyretic (c) as an analgesic (d) as an intestinal antiseptic.

Class 12th Chemsitry - Haloalkanes and Haloarenes Case Study Questions and Answers 2022 - 2023 - by Study Materials - View & Read

  • 1)

    Read the passage given below and answer the following questions:
    A primary alkyl halide (A) C4H9Br reacted with alcoholic KOH to give compound (B). Compound (B) is reacted with HBr to give compound (C) which is an isomer of (A). When (A) reacted with sodium metal, it gave a compound (D) C8H18 that is different than the compound obtained when n-butyl bromide reacted with sodium metal
    The following questions are multiple choice questions. Choose the most ap appropriate answer:
    (i) Compound (A) is

    (a) \(\mathrm{CH}_{3} \mathrm{CH}_{2} \mathrm{CH}_{2} \mathrm{CH}_{2} \mathrm{Br}\) (b) 
    (c)  (d) \(\mathrm{CH}_{3} \mathrm{CH}_{2} \mathrm{CH}_{2} \mathrm{Br}\)

    (ii) Which type of isomerism is present in compound (A) and (C)?

    (a) Positional (b) Functional (c) Chain (d) Both (a) and (c)

    (iii) IUPAC name of compound (D) is

    (a) n-octane (b) 2,5-dimethylhexane (c) 2-methylheptane (d) 3,4-dimethyl hexane.

    (iv) When compoound (C) is treated with alc. KOH and then treated with HBr in presence of peroxide, the compound obtained is

    a)  (b) 
    (c) \(\mathrm{CH}_{3} \mathrm{CH}_{2} \mathrm{CH}_{2} \mathrm{CH}_{2} \mathrm{Br}\) (d) 
  • 2)

    Read the passage given below and answer the following questions:
    Nucleophilic substitution reactions are of two types; substitution nucleophilic bimolecular (SN2) and substitution nucleophilic unimolecular (SN1) depending on molecules taking part in determining the rate of reaction. Reactivity of alkyl halide towards SN1 and SN2 reactions depends on various factors such as steric hindrance, stability of intermediate or transition state and polarity of solvent. SN2 reaction mechanism is favoured mostly by primary alkyl halide then secondary and then tertiary. This order is reversed in case of SN1 reactions.
    The following questions are multiple choice questions. Choose the most appropriate answer:
    (i) Which of the following is most reactive towards nucleophilic substitution reaction?

    (a) \(\mathrm{C}_{6} \mathrm{H}_{5} \mathrm{Cl}\) (b) \(\mathrm{CH}_{2}=\mathrm{CHCl}\) (c) \(\mathrm{ClCH}_{2} \mathrm{CH}=\mathrm{CH}_{2}\) (d) \(\mathrm{CH}_{3} \mathrm{CH}=\mathrm{CHCl}\)

    (ii) Isopropyl chloride undergoes hydrolysis by

    (a) SN1mechanism (b) SN2mechanism (c) SN1 and SN2mechanism (d) neither SN1 and SN2mechanism

    (iii) The most reactive nucleophile among the following is

    (a) CH3O- (b) C6H5O- (c) (CH3)2CHO- (d) (CH3)3CO-

    (iv) Tertiary alkyl halides are practically inert to substitution by SN2mechanism because of

    (a) insolubility (b) instability (c) inductive effect (d) stearic hindrance.
  • 3)

    Read the passage given below and a.9swer the following questions:
    A chlorocompound (A) on reduction with Zn-Cu and ethanol gives the hydrocarbon (B) with five carbon atoms. When (A) is dissolved in dry ether and treated with sodium metal it gave 2,2,5,5 tetramethylhexane. The treatment of (A) with alcoholic KCN gives compound (C).
    The following questions are multiple choice questions. Choose the most appropriate answer:
    (i) The compound (A) is

    (a) 1-chloro- 2, 2-dimethylpropane (b) 1-chloro- 2, 2-dimethyl butane
    (c) 1-chloro-2-methyl butane (d) 2-chloro-2-methyl butane.

    (ii) The reaction of (C) with Na, C2H5OH gives

    (a) (CH3)3C CH2CONH2 (b) (CH3)3C NH2
    (c) (CH)3C CH2CH2NH2 (d) (CH3)2CHCH2NH2

    (iii) The reaction of (C) with Na, C2H5OH is called

    (a) Gilman reaction (b) Mendius reaction
    (c) Grooves process (d) Swart's reaction.

    (iv) Compound (B) is

    (a) n-pentane (b) 2, 2-dimethylpropane
    (c) 2-methylbutane (d) none of these.
  • 4)

    Read the passage given below and answer the following questions:
    When haloalkanes with \(\beta\)-hydrogen atom are boiled with alcoholic solution of KOH, they undergo elimination of hydrogen halide resulting in the formation of alkenes. These reactions are called \(\beta\)-elimination reactions or dehydrohalogenation reactions. These reactions follow Saytzeff's rule. Substitution and elimination reactions often compete with each other. Mostly bases behave as nucleophiles and therefore can engage in substitution or elimination reactions depending upon the alkyl halide and the reaction conditions.
    The following questions are multiple choice questions. Choose the most appropriate answer:
    (i) Among the following the most reactive towards alcoholic KOH is

    (a) \(\mathrm{CH}_{2}=\mathrm{CHBr}\)

    (b) \(\mathrm{CH}_{3} \mathrm{COCH}_{2} \mathrm{CH}_{2} \mathrm{Br}\)

    (c) \(\mathrm{CH}_{3} \mathrm{CH}_{2} \mathrm{Br}\)

    (d) \(\mathrm{CH}_{3} \mathrm{CH}_{2} \mathrm{CH}_{2} \mathrm{Br}\)

    (ii) The general reaction, \(R-X \stackrel{\text { aq. } \mathrm{OH}^{-}}{\longrightarrow} R \mathrm{OH}+X^{-}\) is expected to follow decreasing order of reactivity as in (t- Bu = tertiary Butyl group)

    (a) t-BuI> t-BuBr > t-BuCI > t-BuF (b) t-BuF> t-BuCI > t-BuBr > t-BuI
    (c) t-Bu'Br> t-BuCI > t-BuI > t-BuF (d) t-BuF> t-BuCI > t-BuI > t-BuBr

    (iii) Reaction of t-butyl bromide with sodium methoxide produces

    (a) sodium t-butoxide (b) t-butyl methyl ether
    (c) iso-butane (d) iso-butylene.

    (iv) In the elimination reactions, the reactivity of alkyl halides follows the sequence

    (a) R - F > R - Cl > R - Br > R - I (b) R - I > R - Br > R - Cl > R - F
    (c) R - I > R - F > R - Br > R - Cl (d) R - F > R-I > R-Br > R-CI
  • 5)

    Read the passage given below and answer the following questions:
    Consider the given sequence of reactions:
    The following questions are multiple choice questions. Choose the most appropriate answer:
    (i) Identify w.

    (ii) When X reacts with CH3COCI in presence of anhy. AICI3, the reaction is known as

    (a) Fittig reaction (b) Ullmann reaction (c) Wurtz-Fittig reaction (d) Friedel-Crafts acylation reaction.

    (iii) When X is treated Ni-Al / NaOH the product obtained is

    (a) benzene (b) phenol (c) p-chlorophenol (d) triphenyl.

    (iv) Compound Z is

    (a) phenol (b) p-chlorophenol (c) p-nitrophenol (d)  nitrobenzene

Class 12th Chemsitry - Coordination Compounds Case Study Questions and Answers 2022 - 2023 - by Study Materials - View & Read

  • 1)

    Read the passage given below and answer the following questions:
    The molecular compounds which are formed from the combination of two or more simple stable compounds and retain their identity in the solid as well as in the dissolved state are called coordination compounds. Their properties are completely different from the constituents. In coordination compounds, the central metal atom or ion is linked to a number of ions or neutral molecules, called ligands, by coordinate bonds. For example, Dimethyl glyoxime (dmg) is a bidendate ligand chelating large amounts of metals. When dimethyl glyoxime is added to alcoholic solution of NiCl2 and ammonium hydroxide is slowly added to it, a rosy red precipitate of a complex is formed.
    The following questions are multiple choice questions. Choose the most appropriate answer:
    (i) The structure of the complex is


    (ii) Oxidation number of Ni in the given complex is

    (a) +3 (b) +1 (c) +2 (d) zero.

    (iii) Which of the following is true about this complex?

    (a) It is paramagnetic, containing 2 unpaired electrons.
    (b) It is paramagnetic, containing 1 unpaired electron.
    (c) It is paramagnetic, containing 4 unpaired electrons.
    (d) It is diamagnetic with no unpaired electron

    (iv) Which one will give test for Fe3+ ions in the solution?

    (a) [Fe(CN)6]3- (b) [Fe(CN)6]2-
    (c) (NH4)2SO4· FeSO4· 6H2O (d) Fe2(SO4)3
  • 2)

    Read the passage given below and answer the following questions:
    Coordination compounds are formulated and named according to the IUPAC system.
    Few rules for naming coordination compounds are:
    (I) In ionic complex, the cation is named first and then the anion.
    (II) In the coordination entity, the ligands are named first and then the central metal ion.
    (III) When more than one type of ligands are present, they are named in alphabetical order of preference without any consideration of charge.
    The following questions are multiple choice questions. Choose the most appropriate answer:
    (i) The IUPAC name of the complex \(\left[\mathrm{Pt}\left(\mathrm{NH}_{3}\right)_{3} \mathrm{Br}\left(\mathrm{NO}_{2}\right) \mathrm{Cl}\right] \mathrm{Cl}\) is

    (a) triamminechlorobromonitroplatinum (IV) chloride
    (b) triamminebromonitrochloroplatinum (IV) chloride
    (c) triamminebromidochloridonitroplatinum (IV) chloride
    (d) triamminenitrochlorobromoplatinum (IV) chloride

    (ii) The lUPAC name of [Ni(CO)4] is 

    (a) tetracarbonylnickel (II) (b) tetracarbonylnickel (0)
    (c) tetracarbonylnickelate (II) (d) tetracarbonylnickelate (0).

    (iii) As per IUPAC nomenclature, the name of the complex \(\left[\mathrm{Co}\left(\mathrm{H}_{2} \mathrm{O}\right)_{4}\left(\mathrm{NH}_{3}\right)_{2}\right] \mathrm{Cl}_{3}\) is

    (a) tetraaquadiamminecobalt (II) chloride (b) tetraaquadiamminecobalt (III) chloride
    (c) diamminetetraaquacobalt (II) chloride (d) diamminetetraaquacobalt (III) chloride

    (iv) Which of the following represents correct formula of dichloridobis( ethane-1, 2-diamine ) cobalt (III) ion?

    (a) [CoCl2(en)]2+ (b) [Co(ONO)(NH3)5]SO4 (c) [Co(NO2)(NH3)4] (SO4)2 (d) [Co(NO)(NH3)4] (SO4)2
  • 3)

    Read the passage given below and answer the following questions:
    Iron forms many complexes in its +2 and +3 oxidation states such as \(\left[\mathrm{Fe}\left(\mathrm{H}_{2} \mathrm{O}\right)_{6}\right]^{2+}(A) ;\left[\mathrm{Fe}(\mathrm{CN})_{6}\right]^{4-}(B)\) \(\left[\mathrm{Fe}\left(\mathrm{H}_{2} \mathrm{O}\right)_{6}\right]^{3+}(C) ;\left[\mathrm{Fe}(\mathrm{CN})_{6}\right]^{3-}(D)\) ,etc., They exhibit, different magnetic properties and undergo different hybridisation of iron.
    The following questions are multiple choice questions. Choose the most appropriate answer:
    (i) Which of the following statements is correct?

    (a) (B) is paramagnetic while (C) is diamagnetic.
    (b) Both (B) and (D) are outer orbital complexes.
    (c) Both (A) and (C) are paramagnetic.
    (d) (A) is outer orbital complex and (C) is inner orbital complex.

    (ii) The complex having maximum magnetic moment is

    (a) (A) (b) (B) (c) (C) (d) (D)

    (iii) The spin only magnetic moment of complexes (A), (B), (C) and (D) are respectively (in BM)

    (a) \(2 \sqrt{6}, 0, \sqrt{35}, \sqrt{3}\) (b) \(0,2 \sqrt{6}, \sqrt{35}, \sqrt{3}\) (c) \(\sqrt{15}, 2 \sqrt{6}, \sqrt{3}, 0\) (d) \(\sqrt{3}, \sqrt{8}, 0, \sqrt{15}\)

    (iv) Which of the given complexes are outer orbital complexes?

    (a) (A) and (B) only (b) (B) and (C) only (c) (A) and (C) only  (d) (B) and (D) only
  • 4)

    Read the passage given below and answer the following questions:
    To explain bonding in coordination compounds various theories were proposed. One of the important theory was valence bond theory. According to that, the central metal ion in the complex makes available a number of empty orbitals for the formation ofcoordination bonds with suitable ligands. The appropriate atomic orbitals of the metal hybridise to give a set of equivalent orbitals of definite geometry. The d-orbitals involved in the hybridisation may be either inner d-orbitals i.e., (n - 1) d or outer d-orbitals i.e., nd. For example, CO3+ forms both inner orbital and outer orbital complexes, with ammonia it forms [Co(NH3)6]3+ and with fluorine it forms [CoF6]3- complex ion.
    The following questions are multiple choice questions. Choose the most appropriate answer:
    (i) Which of the following is not true for [CoF6]3- ?

    (a) It is paramagnetic. (b) It has coordination number of 6.
    (c) It is outer orbital complex. (d) It involves d2sp3 hybridisation.

    Which of the following is true for  [Co(NH3)6]3+ ?

    (a) It is an octahedral, dimagnetic and outer orbital complex.
    (b) It is an octahedral, paramagnetic and outer orbital complex.
    (c) It is an octahedral, paramagnetic and inner orbital complex.
    (d) It is an octahedral, dimagnetic and inner orbital complex.

    (iii) The paramagnetism of [CoF6]3- is due to

    (a) 3 electrons (b) 4 electrons (c) 2 electrons (d) 2 electrons

    (iv) Which of the following is an inner orbital or low spin complex?

    (a) \(\left[\mathrm{Ni}\left(\mathrm{H}_{2} \mathrm{O}\right)_{6}\right]^{3+}\) (b)\(\left[\mathrm{FeF}_{6}\right]^{3-}\) (c) \(\left[\mathrm{Co}(\mathrm{CN})_{6}\right]^{3-}\) (d) \(\left[\mathrm{NiCl}_{4}\right]^{2-}\)
  • 5)

    Read the passage given below and answer the following questions:
    Valence bond theory considers the bonding between the metal ion and the ligands as purely covalent. On the other hand, crystal field theory considers the metal-ligand bond to be ionic arising from electrostatic interaction between the metal ion and the ligands. In coordination compounds, the interaction between the ligand and the metal ion causes the five d-orbitals to split-up. This is called crystal field splitting and the energy difference between the two sets of energy level is called crystal field splitting energy. The crystal field splitting energy (Δo) depends upon the nature of the ligand. The actual configuration of complexes is divided by the relative values of Δo and P (pairing energy)
    If Δo < P, then complex will be high spin.
    If Δo > P, then complex will be low spin.
    The following questions are multiple choice questions. Choose the most appropriate answer :
    (i) Which of the following ligand has lowest Δo value?

    (a) CN- (b) CO (c) F- (d) NH3

    (ii) The crystal field splitting energy for octahedral (Δo) and tetrahedral (Δt) complex is related as

    (a) \( \Delta_{t}=\frac{1}{2} \Delta_{o}\) (b) \(\Delta_{t}=\frac{4}{9} \Delta_{o}\)
    (c) \(\Delta_{t}=\frac{3}{5} \Delta_{o}\) (d) \(\Delta_{t}=\frac{2}{5} \Delta_{o}\)

    (iii) On the basis of crystal field theory, the electronic configuration of d4 in two situations: (i) Δo > P and (ii) Δo< P are

    (i) (ii)
    (a) \(t_{2 g}^{4} e_{g}^{0}\) \(t_{2 g}^{3} e_{g}^{1}\)
    (b) \(t_{2 g}^{3} e_{g}^{1}\) \(t_{2 g}^{4} e_{g}^{0}\)
    (c) \(t_{2 g}^{3} e_{g}^{1}\) \(t_{2 g}^{3} e_{g}^{1}\)

    (d) \(t_{2 g}^{4} e_{g}^{0}\)

    \(t_{2 g}^{4} e_{g}^{0}\)

    (iv) Using crystal field theory, calculate magnetic moment of central metal ion of [FeF6]4-.

    (a) 1.79 B.M. (b) 2.83 B.M. (c) 3.85 B.M. (d) 4.9 B.M.

Class 12th Chemsitry - The d- and f- Block Elements Case Study Questions and Answers 2022 - 2023 - by Study Materials - View & Read

  • 1)

    Read the passage given below and answer the following questions:
    The f-block elements are those in which the differentiating electron enters the (n -2) forbital. There are two series of f-block elements corresponding to filling of 4f and 5f-orbitals. The series of 4f- orbitals is called lanthanides. Lanthanides show different oxidation states depending upon stability of f0, f7 and f14 configurations, though the most common oxidation states is +3. There is a regular decrease in size of lanthanides ions with increase in atomic number which is known as lanthanide contraction.
    The following questions are multiple choice questions. Choose the most appropriate answer:
    (i) The atomic numbers of three lanthanide elements X, Y and 2 are 65, 68 and 70 respectively, their Ln3+ electronic configuration is

    (a) 4f8, 4f11, 4f13 (b) 4f11, 4f8 , 4f13 (c) 4fo, 4f2, 4f11 (d) 4f3, 4f7, 4f9

    (ii) Lanthanide contraction is observed in

    (a) Gd (b) At (c) Xe (d) Te

    (iii) Name a member of the lanthanoid series which is well known to exhibit +4 oxidation state.

    (a) Cerium (Z = 58) (b) Europium (Z = 63) (c) Lanthanum (Z = 57) (d) Gadolinium (Z = 64)

    (iv) Identify the incorrect statement among the following.

    (a) Lanthanojd contraction is the accumulation of successive shrinkages.
    (b) The different radii of Zr and Hf due to consequence of the lanthanoid contraction.
    (c) Shielding power of 4f electrons is quite weak.
    (d) There is a decrease in the radii of the atoms or ions as one proceeds from La to Lu.
  • 2)

    Read the passage given below and answer the following questions:
    The transition elements have incompletely filled d-subshells in their ground state or in any of their oxidation states. The transition elements occupy position in between s- and p-blocks in groups 3-12 of the Periodic table. Starting from fourth period, transition elements consists of four complete series : Sc to Zn, Y to Cd and La, Hf to Hg and Ac, Rf to Cn. In general, the electronic configuration of outer orbitals of these elements is (n - 1) d1-10 ns1-2. The electronic configurations of outer orbitals of Zn, Cd, Hg and Cn are represented by the general formula (n - 1)d10 n2. All the transition elements have typical metallic properties such as high tensile strength, ductility, malleability. Except mercury, which is liquid at room temperature, other transition elements have typical metallic structures. The transition metals and their compounds also exhibit catalytic property and paramagnetic behaviour. Transition metal also forms alloys. An alloy is a blend of metals prepared by mixing the components. Alloys may be homogeneous solid solutions in which the atoms of one metal are distributed randomly among the atoms of the other.
    The following questions are multiple choice questions. Choose the most appropriate answer :
    (i) Which of the following characteristics of transition metals is associated with higher catalytic activity?

    (a) High enthalpy of atomisation (b) Variable oxidation states
    (c) Paramagnetic behaviour (d) Colour of hydrated ions

    (ii) Transition elements form alloys easily because they have

    (a) same atomic number (b) same electronic configuration
    (c) nearly same atomic size (d) same oxidation states.

    (iii) The electronic configuration of tantalum (Ta) is

    (a) \([\mathrm{Xe}] 4 f^{0} 5 d^{1} 6 s^{2}\) (b) \([\mathrm{Xe}) 4 f^{14} 5 d^{2} 6 s^{2}\)
    (c) \([\mathrm{Xe}] 4 f^{14} 5 d^{3} 6 s^{2}\) (d) \(\left[\mathrm{Xe}\left]4 f^{14} 5 d^{4} 6 s^{2}\right.\right.\)

    (iv) Which one of the following outer orbital configurations may exhibit the largest number of oxidation states?

    (a) 3d54s1 (b) 3d54s2 (c) 3d24s2 (d) 3d34s2
  • 3)

    Read the passage given below and answer the following questions:
    The unique behaviour of Cu, having a positive Eaccounts for its inability to liberate H2 from acids. Only oxidising acids (nitric and hot concentrated sulphuric acid) react with Cu, the acids being reduced. The stability of the half-filled (d5) subshell in Mn2+ and the completely filled (d10 ) configuration in Zn2+ are related to their Eo (M3+ /M2+) values. The low value for Sc reflects the stability of Sc3+ which has a noble gas configuration. The comparatively high value for Mn shows that Mn2+(d5) is particularly stable, whereas a comparatively low value for Fe shows the extra stability of Fe3+ (d5). The comparatively low value for V is related to the stability of V2+ (half-filled t2g level).
    The following questions are multiple choice questions.Choose the most appropriate answer :
    (i) Standard reduction electrode potential of Zn2+ /Zn is - 0.76 V. This means

    (a) ZnO cannot be reduced to Zn by H2 under standard conditions
    (b) Zn cannot liberate H2 with concentrated acids
    (c) Zn is generally the anode in an electrochemical cell
    (d) Zn is generally the cathode in an electrochemical cell.

    (ii) Eo values for the couples Cr3+/Cr2+ and Mn3+ /Mn2+ are -0.41 and +1.51 volts respectively. These values suggest that

    (a) Cr2+ acts as a reducing agent whereas Mn3+ acts as an oxidizing agent
    (b) Cr2+ is more stable than Cr3+ state
    (c) Mn3+ is more stable than Mn2+
    (d) Cr2+ acts as an oxidizing agent whereas Mn3+ acts as a reducing agent

    (iii) The reduction potential values of M, Nand O are +2.46, -1.13 and -3.13 Y respectively. Which of the following order is correct regarding their reducing property?

    (a) O>N>M (b) O>M>N (c) M>N>O (d) M>O>N

    (iv) Which of the following statements are true?
    (i) Mn2+ compounds are more stable than Fe2+ towards oxidation to +3 state.
    (ii) Titanium and copper both in the first series of transition metals exhibits +1 oxidation state most frequently.
    (iii) Cu+ ion is stable in aqueous solutions.
    (iv) The E0 value for the Mn3+ /Mn2+ couple is much more positive than that for Cr3+/Cr2+ or Fe3+/Fe2+.

    (a) (ii) and (iii) (b) (i) and (iv) (c) (i) and (iii) (d) (ii) and (iv)
  • 4)

    Read the passage given below and answer the following questions:
    Transition metal oxides are compounds formed by the reaction of metals with oxygen at high temperature. The highest oxidation number in the oxides coincides with the group number. In vanadium, there is a gradual change from the basic V2O3 to less basic V2O4 and to amphoteric V2O5・V2O4 dissolves in acids to give VO2+ salts. Transition metal oxides are commonly utilized for their catalytic activity and semiconductive properties. Transition metal oxides are also frequently used as pigments in paints and plastic. Most notably titatnium dioxide. One of the earliest application of transition metal oxides to chemical industry involved the use of vanadium oxide for catalytic oxidation of sulfur dioxide to sulphuric acid. Since then, many other applications have emerged, which include benzene oxidation to maleic anhydride on vandium oxides; cyclohexane oxidation to adipic acid on cobalt oxides. An important property of the catalyst material used in these processes is the ability of transition metals to change their oxidation state under a given chemical potential of reductants and oxidants.
    The following questions are multiple choice questions. Choose the most appropriate answer:
    (i) Which oxide of vanadium is most likely to be basic and ionic ?

    (a) VO (b) V2O3 (c) VO2 (d) V2O5

    (ii) Vanadyl ion is

    (a) VO2+ (b) VO2+ (c) V2O+ (d) VO43-

    (iii) The oxidation state of vanadium in V2O5 is

    (a) +5/2 (b) +7 (c) +5 (d) +6

    (iv) Identify the oxidising agent in the following reaction.

    (a) V2O5 (b)Ca (c) V  (d) None of these
  • 5)

    Read the passage given below and answer the following questions:
    Transition elements are elements that have partially filled d-orbitals. The configuration of these elements corresponds to (n - 1)d1-10 ns1-2. It is important to note that the elements mercury, cadmium and zinc (Ire not considered transition elements because of their electronic configurations, which corresponds to (n - 1)d1-10 ns2.
    Some general properties of transition elements are :
    These elements can form coloured compounds and ions due to d-d transition;
    These elements exhibit many oxidation states;
    A large variety of ligands can bind themselves to these elements, due to this, a wide variety of stable complexes formed by these ions. The boiling and melting point of these elements are high. These elements have a large ratio of charge to the radius.
    In these questions (i-iv), a statement of assertion followed by a statement of reason is given. Choose the correct answer out of the following choices.
    (a) Assertion and reason both are correct statements and reason is correct explanation for assertion.
    (b) Assertion and reason both are correct statements but reason is not correct explanation for assertion.
    (c) Assertion is correct statement but reason is wrong statement.
    (d) Assertion is wrong statement but reason is correct statement.
    (i) Assertion: Tungsten has very high melting point.
     Reason: Tungsten is a covalent compound.
    (ii) Assertion: Zn, Cd and Hg are normally not considered transition metals.
    Reason: d-Orbitals in Zn, Cd and Hg elements are completely filled, hence these metals do not show the general characteristics properties of the transition elements.
    (iii) Assertion: Copper metal gets readily corroded in acidic aqueous solution such as HCI and dil. H2SO4
    Reason: Free energy change for this process is positive.
    (iv) Assertion: Tailing of mercury occurs on passing ozone through it.
    Reason: Due to oxidation of mercury.

Class 12th Chemsitry - The p-Block Elements Case Study Questions and Answers 2022 - 2023 - by Study Materials - View & Read

  • 1)

    Read the passage given below and answer the following questions:
    Under the normal conditions, noble gases are monoatomic and have closed shell electronic configuration. Lighter noble gases have low boiling points due to weak dispersion forces between the atoms and the absence of other interatomic interactions. Xenon, one of the important noble gas, forms a series of compounds with fluorine with oxidation number +2, +4 and +6. All xenon fluorides are strong oxidising agents. XeF4 reacts violently with water to give XeO3. The geometry of xenon compounds can be deduced by considering the total number of electron pairs in their valence shell.
    The following questions are multiple choice questions. Choose the most appropriate answer:
    (i) Among noble gases (from He to Xe) only xenon reacts with fluorine to form stable xenon fluorides because xenon

    (a) has the largest size
    (b) has the lowest ionisation enthalpy
    (c) has the highest heat of vapourisation
    (d) is the most readily available noble gas.

    (ii) The structure of XeO3 is

    (a) square planar (b) pyramidal (c) linear (d) T-shaped.

    (iii) In the preparation of compound of xenon, Bartlett had taken \(\mathrm{O}_{2}^{+} \mathrm{PtF}_{6}^{-}\) as a base compound. This is because

    (a) both O2 and Xe have same size
    (b) both Xe and O2 have same electron gain enthalpy
    (c) both have almost same ionisation enthalpy
    (d) both Xe and O2 are gases.

    (iv) The oxidation state of xenon in XeO3 is

    (a) +4 (b) +2 (c) +8 (d) +6
  • 2)

    Read the passage given below and answer the following questions :
    Interhalogen compounds are formed when halogen group elements react with each other. These are the compounds which consist of two or more different elements of group - 17. A halogen with large size and low electronegativity reacts with an element of group - 17 with small size and high electronegativity. As the ratio of  radius of larger and smaller halogen increases, the number of atoms in a molecule also increases.
    The following questions are multiple choice questions. Choose the most appropriate answer :
    (i) The stability of interhalogen compounds follows the order

    (a) IF3> BrF3 > ClF3 (b) ClF3 > BrF3 > IF3
    (c) BrF3 > IF3 > ClF3 (d) ClF3 > IF3 > BrF3

    (ii) Identify the correct match from the following.

    (a) [ICl2]- -bent (b) IF7 - pentagonal bipyramidal
    (c) ClF3 - trigonal planar (d) [BrF4r]- -square pyramidal

    (iii) In XA5, the central atom has (both X and A are halogens)

    (a) 5 bond pairs and no lone pairs (b) 5 bond pairs and one lone pair
    (c) 6 bond pairs and no lone pairs (d) 4 bond pairs and one lone pair.

    (iv) In the known interhalogen compounds, the maximum number of atoms are

    (a) 4 (b) 5
    (c) 8 (d) 7
  • 3)

    Read the passage given below and answer the following questions :
    Noble gases are inert gases with general electronic configuration of ns2np6. These are mono atomic, colourless, odourless and tasteless gases. The first compound of noble gases was obtained by the reaction of Xe with PtF6. A large number of compounds of Xe and fluorine have been prepared till now. The structure of these compounds can be explained on the basis of VSEPR theory as well as concept of hybridisation. The compounds of krypton are fewer. Only the difluoride of krypton (KrF2) has been studied in detail. Compounds of radon have not isolated but only identified by radio tracer technique. However, no true compounds of helium, neon or argon are yet known.
    The following questions are multiple choice questions. Choose the most appropriate answer:
    (i) The formula of the compound when Xe and PtF6 are mixed, is

    (a) XeF6 (b) XeF4 (c) Xe2PtF6 (d) Xe+[PtF6]-

    (ii) Which of the following is not formed by Xe?

    (a) XeFs (b) XeF (c) XeF3 (d) All of these

    (iii) The number of lone pairs and bond pairs of electrons around Xe in XeOF4 respectively are

    (a) O and 5 (b) 1 and 5 (c) 1 and 4 (d) 2 and 3

    (iv) Which of the following compounds has more than one lone pair of electrons around central atom?

    (a) XeO3 (b) XeF2 (c) XeOF4 (d) XeO2F2
  • 4)

    Read the passage given below and answer the following questions:
    All the elements of group 16 have ns2 np4 configuration in their outermost shell. Therefore, the atoms of these elements try to gain or share two electrons to achieve noble gas configuration. Sulphur and other elements of group 16 are less electronegative than oxygen, so, they cannot accept electrons easily. By sharing of two electrons with other elements, these elements acquire ns2 np6 configuration and exhibit +2 oxidation state. Except oxygen, group 16 elements have vacant d-orbitals in their valence shell to which electrons can be promoted from p- and s-orbitals of the same shell. As a result, they can show +4 and +6 oxidation states also.
    The following questions are multiple choice questions. Choose the most appropriate answer:
    (i) Oxygen shows +2 oxidation state in

    (a) OF2 (b) H2O (c) Cl2O (d) H2O2

    (ii) Like sulphur, oxygen is not able to show +4 and +6 oxidation states because

    (a) oxygen is a gas while sulphur is a solid
    (b) sulphur has high ionisation enthalpy as compared to oxygen
    (c) oxygen has no d-orbitals in its valence shell
    (d) oxygen has high electron affinity as compared to sulphur.

    (iii) Oxidation state of sulphur in Na2S4O6

    (a) 7/2 (b) 5/2 (c) 1/2 (d) 3/2

    (iv) The oxidation states of sulphur in S8' SO3 and H2S are respectively

    (a) 0, +6 and -2 (b) +6,0 and -2 (c) -2,0 and +6 (d) +2, +6 and -2
  • 5)

    Read the passage given below and answer the following questions:
    Nitric acid reacts with most of the metals (except noble metals like gold and platinum) and non-metals. Towards its reaction with metals, HNO3 acts as an acid as well as an oxidising agent. Like other acids, HNO3 liberate nascent hydrogen from metals which further reduces the nitric acid into number of products like NO, NO2, N2O or NH3. The different stages of reduction of nitric acid are:
    \(\mathrm{HNO}_{3} \stackrel{+e^{-}}{\longrightarrow} \mathrm{NO}_{2} \stackrel{+4}{\longrightarrow} \stackrel{+2 e^{-}}{\longrightarrow} \mathrm{NO} \frac{+2^{-}}{\mathrm{NaOH}} \stackrel{+1}{\mathrm{~N}}_{2} \mathrm{O} \stackrel{+4 e^{-}}{\longrightarrow} \stackrel{-3}{\mathrm{NH}}_{3}\) 
    The product of the reduction of HNO3 depends upon the nature of the metal, concentration of nitric acid and temperature.
    The following questions are multiple choice questions. Choose the most appropriate answer :
    (i) Which of the following reactions Is used to prepare laughing gas?

    (a) \(\mathrm{Pb}+\text { dil. } \mathrm{HNO}_{3} \longrightarrow\) (b) \(\mathrm{Hg}+\text { dil. } \mathrm{HNO}_{3} \longrightarrow\)
    (c) \(\mathrm{Zn}+\mathrm{dil} . \mathrm{HNO}_{2} \longrightarrow\) (d) \(\mathrm{Cu}+\text { dil. } \mathrm{HNO}_{3} \longrightarrow\)

    (ii) Gold and platinum does not dissolve in HN03 but soluble in 1 : 3 mixture of HNO3 and HCI due to the formation of respectively

    (a) Au(NO3)2' [Pt(NO3)2] (b) H[AuCI4], H2[PtCI6]
    (c) [AuCI6]2-, [PtCI2]2- (d) [Au(NO3)4]+, [Pt(NO3)6]2-

    (iii) Identify B in the following reaction.
    \(\mathrm{Cu}+\mathrm{HNO}_{3(\text { conc. })} \rightarrow(A)+(B)+\mathrm{H}_{2} \mathrm{O}\) 
                  Deep blue colour   Gas

    (a) NO2 (b) N2 (c) NO (d) N2O

    (iv) In which of the following reactions HN03 will not act as an oxidising agent?

    (a) \(\mathrm{HNO}_{3}+\mathrm{H}_{2} \mathrm{SO}_{4} \rightarrow\) (b) \(\mathrm{HNO}_{3}+\mathrm{FeSO}_{4}+\mathrm{H}_{2} \mathrm{SO}_{4} \rightarrow\)
    (c) \(\mathrm{KI}+\mathrm{HNO}_{3} \rightarrow\) (d) \(\mathrm{Au}+\mathrm{HNO}_{3} \rightarrow\)

Class 12th Chemsitry - General Principles and Processes of Isolation of Elements Case Study Questions and Answers 2022 - 2023 - by Study Materials - View & Read

  • 1)

    Aluminium is most abundant metal in earth crust. It is extracted by electrolytic reduction of molten Al2 O3 mixed with cryolite. Chemical reduction is used in reduction of oxide of zinc, iron, copper. Mn and Cr can also extracted by chemical reduction using' AI' as reducing agent. If at a particular temperature \(\triangle\)Go of the reaction is -ve with a particular reducing agent then that reducing agent is most effective of that temperature. In electrolytic reduction \(\Delta \mathbf{G}^{\circ}=-\mathbf{n} \mathbf{E}^{\circ} \mathbf{F},+\mathbf{v e}, \mathbf{E}^{\circ} \mathbf{y}\) value will make \(\triangle\)Go negative and make the process feasible. More reactive metal can displace less reactive metal from its salt solution or complex.
    (a) \(\mathbf{2 C l}+2 \mathrm{H}_{2} \mathbf{O} \rightarrow \mathbf{2} \mathbf{O H}^{-}+\mathbf{H}_{2}+\mathbf{C l}_{2}, \Delta \mathbf{G}=+\mathbf{4 2 2} \mathbf{k J}\) calculate the minimum potential difference required for the process?
    (b) Which reducing agent is used in extraction of gold from \(\left[\mathbf{A u}(\mathbf{C N})_{2}\right]^{\ominus}\) why?
    (c) What is role of limestone in extraction of iron?
    (d) What is use of Ellingham diagram?
    (e) What are coupled reactions?

  • 2)

    Observe the plots of \(\triangle\)Go Vs T for formation of various oxides and answer the questions based on this diagram.

    (a) What are these plots called?
    (b) Below whattemperature Mg can reduce Al2 O3 ?
    (c) What is limitation of Ellingham diagram?
    (d) Why is reduction easier if metal formed is in molten state?
    (e) Name the reducing agent used in extraction of Fe2O3 . Give chemical reaction.

  • 3)

    The melting point of some metals are given in the table. Observe the table and answer the questions that follow:

    Metal Melting Point
    1. Sodium 98°C
    2. Iron 1538°C
    3. Aluminium 660.3°C
    4. Tin 231.9°C
    5. Zinc 419.5°C
    6. Germanium 938.2°C
    7. Titanium 1668°C
    8. Mercury 356.7° (Boiling point)
    9. Copper 1085°C
    10. Gold 1064°C

    (a) Name two metals which can be purified by distillation and why?
    (b) Which methods is used for refining AI, Cu and Zn?
    (c) Which method is most suitable for tin and why?
    (d) How is Titanium refined?
    (e) How is germanium of high purity obtained? What is its basic principle?

Class 12th Chemsitry - Surface Chemistry Case Study Questions and Answers 2022 - 2023 - by Study Materials - View & Read

  • 1)

    Read the passage given below and answer the following questions :
    Adsorption depends on the nature of the adsorbent. The rough solid surface has more number of pores and adsorb more number of gases than the smooth surface. Most common adsorbents are silica gel, activated charcoaL The extent of adsorption also depends on the surface area of the solid. Specific surface area of an adsorbent is the surface area available for adsorption per gram of the adsorbent. The greater the surface area of the solid, the greater would be the adsorption. Charcoal is a more effective adsorbent than solid wood. Desorption is a process of removing an adsorbed substance from a surface on which it is absorbed.
    Physisorption is non-specific and any gas can be adsorbed. But the gases which are easily liquefiable (e.g., NH3 , HCl, CO2 ) are adsorbed at a faster rate and to a large extent than the gases which are difficult to liquefy (e.g., H2 , O2, N2 ). It depends on the critical temperature. Higher the critical temperature of a gas, more easily liquefiable the gas is and more is the rate of adsorption. Chemisorption is specific in nature. Therefore, only those gases can be adsorbed which are capable of forming chemical bonds with the adsorbent.
    The following questions are multiple choice questions. Choose the most appropriate answer :
    (i) Select the correct statement regarding desorption.

    (a) It is done by cooling or by increasing the pressure applied.
    (b) It is done by cooling or by reducing the pressure applied.
    (c) It is done by heating or by reducing the pressure applied
    (d) It is done by heating or by increasing the pressure applied.

    (ii) Which of the following statements regarding the physical adsorption of a gas on surface of solid is not correct?

    (a) On increasing temperature, adsorption increases continuously
    (b) Enthalpy changes are negative
    (c) It is non-specific in nature
    (d) It is reversible in nature

    (iii) At the same temperature and pressure, select the correct order of adsorption of the following gases on the same mass of charcoal.

    (a) SO> CH4 > H2 (b) CH4 < SO2 < H2
    (c) H> CH4 > SO2 (d) CH4 < H2 < SO2

    (iv) Select the incorrect statement among the following.

    (a) Physical adsorption occurs at a low temperature and chemisorption occurs at all temperature
    (b) In physisorption heat of adsorption is low while in chemisorption it is high
    (c) Chemisorption is irreversible and physisorption is reversible
    (d) Magnitude of chemisorption decreases with rise in temperature while physisorption increases with rise in temperature.
  • 2)

    Read the passage given below and answer the following questions :
    A graph between the amount adsorbed (x/m) by an adsorbent and the equilibrium pressure of the adsorbate at a constant temperature is called the adsorption isotherm. A relationship between the amount adsorbed (x/m) and the equilibrium pressure (P) can be obtained as follows:

    In the intermediate range of pressure, x/m = kP1/n (was originally put forward by Freundlich and is known as Freundlich adsorption isotherm).
    The following questions are multiple choice questions. Choose the most appropriate answer :
    (i) According to Freundlich adsorption isotherm, which of the following is correct?

    (a) \(\frac{x}{m} \propto P^{0}\) (b) \(\frac{x}{m} \propto P^{1}\)
    (c) \(\frac{x}{m} \propto P^{1 / n}\) (d) All the above are correct for different range of pressure

    (ii) In the Freundlich adsorption isotherm equation
    \(\log \frac{x}{m}=\log k+\frac{1}{n} \log p\) ,the value of 1/n is

    (a) any value from 0 to 1 (b) a negative integer
    (c) a positive integer (d) a positive or a negative fractional number.

    (iii) Plot of log xlm against log p is a straight line inclined at an angle of 45°. When the pressure is 0.5 atm and Freundlich parameter, k is 10, the amount of solute adsorbed per gram of adsorbent will be (log 5 = 0.6990)

    (a) 1g (b) 2g (c) 3g (d) 5g

    (iv) In the plot of log \(\frac{x}{m}\) vs log p for an adsorption, a straight line inclined at an angle of \(\theta=14.04^{\circ}\) to the x-axis was obtained.The 'n' value for this adsorption process is (tan 14.04° = 0.25)

    (a) 5 (b) 8 (c) 4 (d) 2
  • 3)

    Read the passage given below and answer the following questions :
    Hardy Schulze rule states that the precipitating effect of an ion on dispersed phase of opposite charge increases with the valency of the ion. The higher the valency of the flocculating ion, the greater is its precipitating power. Thus, for the precipitation of AS2 S3 sol (-ve sol) the precipitating power of Al3+, Ba2+ and Naions is of the order, Al3+> Ba2+ > Na+. Similarly, for precipitating Fe(OH)3 sol (-ve sol) the precipitating power of \(\left[\mathrm{Fe}(\mathrm{CN})_{6}\right]^{3-}\)\(\mathrm{SO}_{4}^{2-}\)   and Cl- is of the order,\(\left[\mathrm{Fe}(\mathrm{CN})_{6}\right]^{3-}>\mathrm{SO}_{4}^{2-}>\mathrm{Cl}^{-}\) . The minimum concentration of an electrolyte in millimoles per litre required to cause precipitation of a sol in 2 hours is called flocculation value. The smaller the flocculation value, the higher will be the coagulating power of the ion. The minimum mass of the protective colloid (lyophilic colloid) in milligrams that must be added to 10 mL of a standard red gold sol so that no coagulation occurs when 1 mL of 10% NaCl solution is rapidly added to it is called the gold number of the protective colloid. 
    The following questions are multiple choice questions. Choose the most appropriate answer:
    (i) The gold number of four protective colloids A, B, C and Dare 0.03, 0.003, 10 and 30 respectively. Protective power of these colloids will be of the order:

    (a) B > A > C > D (b) A > B > C > D (c) C >B> D > A (d) D > A >  C >

    (ii) Which of the following has least flocculating value for positive sol?

    (a) Cl- (b) \(\mathrm{SO}_{4}^{2-}\) (c) \(\mathrm{PO}_{4}^{3-}\) (d) [Fe(CN6)4-

    (iii) Which of the following colloidal solutions is positively charged?

    (a) TiO2 (b) AS2 S3 (c) Blood (d) Gold sol

    (iv) The coagulation value in millimoles per litre of electrolytes used for the coagulation of As2 S3 are as below:

    I. NaCl = 52 II. KCl = 50 III. BaCl2 = 0.69 IV. MgSO4 = 0.72

    The correct order of their flocculating power is

    (a) I > II > III > IV (b) I > II > IV > III (c) I < II < IV <III (d) IV < I < II < III
  • 4)

    Read the passage given below and answer the following questions :
    Adsorption is a spontaneous process and involves unequal distribution of the molecules of the gaseous substance on the surface of solid or liquid. Adsorption is an exothermic process. The attractive forces between adsorbate and adsorbent are either van der Waals' forces or chemical bonds. Adsorption of gases on solids is generally controlled by the factors like temperature, pressure and nature of adsorbate and adsorbent.
    The following questions are multiple choice questions.Choose the most appropriate answer:
    (i) In physisorption process, the attractive forces between adsorbate and adsorbent are

    (a) covalent bonds (b) ionic bonds
    (c) van der Waals' force (d) H-bonds

    (ii) Which of the following graph represents the variation of physical adsorption with temperature?

    (iii) Which one of the following processes does not use adsorption?

    (a) Froth floatation process (b) Chromatography
    (c) Decolourisation of sugar liquors (d) Dissolution of sugar in water

    (iv) Which of the following statements is true?

    (a) Chemisorption forms unimolecular layer
    (b) Chemisorption is a reversible process.
    (c) Chemisorption is independent of pressure.
    (d) Chemisorption has low enthalpy change.
  • 5)

    Read the passage given below and answer the following questions :
    Colloidal particles carry either positive or negative charge. The nature of this charge is the same on all the dispersed particles in a given colloidal solution and may be either positive or negative. When two or more ions are present in the dispersion medium, preferential adsorption of the ion common to the colloidal particles usually takes place. When KI solution is added to the AgNO3 solution till KI is in excess, the precipitated silver iodide adsorbs iodide ions from the dispersion medium, and negatively charged colloidal solution results. If the colloidal sol of AgI is prepared by adding AgNO3 solution to KI solution till AgNO3 is in slight excess, Ag+ ions will be adsorbed giving positive charge to the colloidal particles. The combination of the two layers of opposite charges around the colloidal particles is called Helmholtz electrical double layer. The presence of equal and similar charges on colloidal particles is largely responsible for providing stability to the colloidal solution.
    In these questions (i-iv), a statement of assertion followed by a statement of reason is given. Choose the correct answer out of the following choices.

    (a) Assertion and reason both are correct statements and reason is correct explanation for assertion. 
    (b) Assertion and reason both are correct statements but reason is not correct explanation for assertion.
    (c) Assertion is correct statement but reason is wrong statement
    (d) Assertion is wrong statement but reason is correct statement.

    (i) Assertion : The presence of electric charge on colloidal particles is indicated by electrophoresis.
    Reason : Lyophilic sols, in contrast to lyophobic sols are easily coagulated on addition of small amounts of electrolytes.
    (ii) Assertion : Gold number is a measure of protective action by a lyophilic colloid on a lyophobic colloid.
    Reason : Zeta potential (or electrokinetic potential) is the potential difference between fixed charged layer and the diffused layer having opposite charge.
    (iii) Assertion: The presence of equal and similar charges on colloidal particles islargely responsible in providing stability to the colloidal solution.
    Reason : The repulsive forces between charged particles having the same charge prevent them from aggregating and provide stability.
    (iv) Assertion: When FeCl3 is added to an excess of hot water, a positively charged sol of hydrated ferric oxide is formed.
    Reason : When ferric chloride is added to NaOH solution a negatively charged sol is obtained due to adsorption of OH- ions.

Class 12th Chemsitry - Chemical Kinetics Case Study Questions and Answers 2022 - 2023 - by Study Materials - View & Read

  • 1)

    Read the passage given below and answer the following questions :
    The progress of the reaction, \(A \rightleftharpoons n B\) with time is represented in the following figure.

    The following questions are multiple choice questions. Choose the most appropriate answer:
    (i) What is the value of n?

    (a) 1 (b) 2 (c) 3 (d) 4

    (ii) Find the-value of the equilibrium constant

    (a) 0.6 M (b) 1.2M (c) 0.3M (d) 2.4M

    (iii) The initial rate of conversion of A will be

    (a) 0.1 mol L-1 hr-1 (b) 0.2 mol L-1 hr-1 (c) 0.4 mol L-1 hr-1 (d) 0.8 mol L-1 hr-1

    (iv) For the reaction,  if \(\frac{d[B]}{d t}=2 \times 10^{-4}\) , value of \(-\frac{d[A]}{d t}\) will be

    (a) 2 x 10- 4 (b) 10-4 (c) 4 x 10- 4 (d) 0.5 x 10- 4
  • 2)

    Read the passage given below and answer the following questions:
    For the reaction: \(2 \mathrm{NO}_{(g)}+\mathrm{Cl}_{2(g)} \rightarrow 2 \mathrm{NOCl}_{(g)}\),  the following data were collected. All the measurements were taken at 263 K.

    Experiment 
    No.
    Initial [NO] (M) Initial [Cl2] (M) Initial rate of disapp. of Cl2 (M/min)
    1. 0.15 0.15 0.60
    2.` 0.15 0.30 1.20
    3. 0.30` 0.15 2.40
    4 0.25 0.25 ?

    The following questions are multiple choice questions. Choose the most appropriate answer:
    (i) The molecularity of the reaction is 

    (a) 1 (b) 2 (c) 3 (d) 4

    (ii) The expression for rate law is

    (a) r = k[NO][Cl2] (b) r = k[NO]2[Cl2 ] (c) ) r = k[NO][Cl2]2 (d) r = k[NO]2[Cl2]2

    (iii) The overall order of the reaction is

    (a) 2 (b) 0 (c) 1 (d) 3

    (iv) The value of rate constant is

    (a) 150.32 M-2 min-1 (b) 200.08 M-1 min-1 (c) 177.77 M-2 min-1 (d) 155.75 M-1 min-1
  • 3)

    Read the passage given below and answer the following questions:
    A reaction is said to be of the first order if the rate of the reaction depends upon one concentration term only. For a first order reaction of the type A \(\rightarrow\) Products, the rate of the reaction is given as : rate = k[A]. The differential rate law is given as \(\frac{d A}{d t}=-k[A]\) .The integrated rate law : In \(\frac{[A]}{[A]_{0}}=-k t\) where [A] is the concentration of reactant left at time t and [A]o is the initial concentration of the reactant, k is the rate constant.
    The following questions are multiple choice questions. Choose the most appropriate answer :
    (i) The unit of rate constant for a first order reaction is

    (a) s-1 (b) mol L-1 s-1 (c) L mol-1 s-1 (d) L2 mol-2 s-1

    (ii) Half-life period of a first order reaction is 10 min. Starting with initial concentration 12 M, the rate after 20 min is

    (a) 0.693 x 3 M min-1 (b) 0.0693 x 4 M min-1 (c) 0.0693 M min-1 (d) 0.0693 x 3 M min-1

    (iii) For a first order reaction, (A) \(\rightarrow\) products, the concentration of A changes from 0.1 M to 0.025 M in 40 minutes. The rate of reaction when the concentration of A is 0.01 M, is

    (a) 3.47 x 10-4 M/min (b) 3.47 x 10-5 M/min (c) 1.73 x 10-4 M/min  (d) 1.73 x 10-5 M/min

    (iv) The half-life period of a 1st order reaction is 60 minutes. What percentage will be left over after 240 minutes?

    (a) 6.25% (b) 4.25% (c) 5% (d) 6%
  • 4)

    Read the passage given below and answer the following questions :
    Number of molecules which must collide simultaneously to give product is called molecularity. It is equal to sum of coefficients of reactants present in stoichiometric chemical equation. For reaction, \(m_{1} A+m_{2} B \rightarrow \text { Product }\) 
    Molecularity = [m1 + m2 ]
    In complex reaction each step has its own molecularity which is equal to the sum of coefficients of reactants present in a particular step. Molecularity is a theoretical property. Its value is any whole number. Number of concentration terms on which rate of reaction depends is called order of reaction or sum of powers of concentration terms present in the rate equation is called order of reaction.
    If rate equation of reaction is : Rate = \(k \cdot C_{A}^{m_{1}} \cdot C_{B}^{m_{2}}\) 
    Then order of reaction = m1 + m2
    In simple reaction, order and molecularity are same. In complex reaction, order of slowest step is the order of over all reaction. This step is known as rate determining step. Order is an experimental property. Its value may be zero, fractional or negative.
    The following questions are multiple choice questions. Choose the most appropriate answer:
    (i) Higher order (> 3) reactions are rare due to

    (a) shifting of equilibrium towards reactants due to elastic collisions
    (b) loss of active species on collision
    (c) low probability of simultaneous collision of all the reacting species
    (d) increase in entropy and activation energy as more molecules are involved

    ​​​​​​(ii) The molecularity of the reaction:
    \(6 \mathrm{FeSO}_{4}+3 \mathrm{H}_{2} \mathrm{SO}_{4}+\mathrm{KClO}_{3} \rightarrow \mathrm{KCl}+3 \mathrm{Fe}_{2}\left(\mathrm{SO}_{4}\right)_{3}+3 \mathrm{H}_{2} \mathrm{O} \text { is }\)

    (a) 6 (b) 3 (c) 10 (d) 7

    (iii) Which of the following statements is false in the following?

    (a) Order of a reaction may be even zero
    (b) Molecularity of a reaction is always a whole number.
    (c) Molecularity and order always have same values for a reaction.
    (d) Order of a reaction depends upon the mechanism of the reaction.

    (iv) The rate of the reaction \(A+B+C \rightarrow \text { products }\) , is given by \(r=-\frac{d[A]}{d t}=k[A]^{1 / 2}[B]^{1 / 3}[C]^{1 / 4}\) ,The order of the reaction is

    (a) \(\frac{1}{3}\) (b) \(\frac{1}{4}\)   (c) \(\frac{1}{2}\) (d) \(\frac{13}{12}\)
  • 5)

    Read the passage given below and answer the following questions :
    In a reaction, the rates of disappearance of different reactants or rates of formation of different products may not be equal but rate of reaction at any instant of time has the same value expressed in terms of any reactant or product. Further, the rate of reaction may not depend upon the stoichiometric coefficients of the balanced chemical equation. The exact powers of molar concentrations of reactants on which rate depends are found experimentally and expressed in terms of 'order of reaction'. Each reaction has a characteristic rate constant depends upon temperature. The units of the rate constant depend upon the order of reaction.
    The following questions are multiple choice questions. Choose the most appropriate answer :
    (i) The rate constant of a reaction is found to be 3 x 10-3 mol-2 L 2 sec-1.The order of the reaction is

    (a) 0.5 (b) 2 (c) 3 (d) 1

    (ii) In the reaction \(A+3 B \rightarrow 2 C\) ,the rate of formation of C is

    (a) the same as rate of consumption of A (b) the same as the rate of consumption of B
    (c) twice the rate of consumption of A (d) 3/2 times the rate of consumption of B.

    (iii) Rate of a reaction can be expressed by following rate expression, Rate = k[A]2 [B], if concentration of A is increased by 3 times and concentration of B is increased by 2 times, how many times rate of reaction increases?

    (a) 9 times (b) 27 times (c) 18 times (d) 8 times

    (iv) The rate of a certain reaction is given by,rate = k[H+]n . The rate increases 100 times when the pH changes from 3 to 1. The order (n) of the reaction is

    (a) 2 (b) 0 (c) 1 (d) 1.5

Class 12th Chemsitry - Electrochemistry Case Study Questions and Answers 2022 - 2023 - by Study Materials - View & Read

  • 1)

    Read the passage given below and answer the following questions:
    Molar conductivity of ions are given as product of charge on ions to their ionic mobilities and Faraday's constant.
    \(\lambda_{A^{n+}}=n \mu_{A^{n+}} F\) (here \(\mu\)  is the ionic mobility of An+).
    For electrolytes say AXBy, molar conductivity is given by
    \(\lambda_{m\left(A_{x} B_{y}\right)}=x_{n} \mu_{A^{n+}} F+y_{m} \lambda_{A^{m}-F}\) 

    Ions Ionic mobility
    K+ 7.616 x 10- 4
    Ca2+ 12.33 x 10-4
    Br- 8.09 x 10- 4
    \(\mathrm{SO}_{4}^{2-}\) 16.58 x 10- 4

    The following questions are multiple choice questions. Choose the most appropriate answer
    (i) At infinite dilution, the equivalent conductance of CaSO4  is

    (a) 256 x 10-4 (b) 279 (c) 23.7 (d) 2.0 x 10- 8

    (ii) If the degree of dissociation of CaSO4 solution is 10% then equivalent conductance of CaSO4 is

    (a) 3.59 (b) 36.9 (c) 27.9 (d) 30.6

    (iii) What is the unit of equivalent conductivity?

    (a) ohm-1 cm2 eq-1 (b) ohm cm2eq-1
    (c) ohm-1 cm eq-1 (d) ohm cm2 eq-1

    (iv) If the molar conductance value of Ca2+ and Cl- at infinite dilution are 118.88 x 10-4 m2 mho mol-1 and 77.33 x 10-4 m 2 mho mol-1 respectively then the molar conductance of CaCl2 (in m2 mho mol-1) will be

    (a) 120.18 x 10- 4 (b) 135 x 10-4 (c) 273.54 x 10-4 (d) 192.1 x 10-4
  • 2)

    Read the passage given below and answer the following questions:
     Standard electrode potentials are used for various processes:
    (i) It is used to measure relative strengths of various oxidants and reductants.
    (ii) It is used to calculate standard cell potential.
    (iii) It is used to predict possible reactions.
     A set of half-reactions (in acidic medium) along with their standard reduction potential, Eo (in volt) values are given below
    \(\mathrm{I}_{2}+2 e^{-} \rightarrow 2 \mathrm{I}^{-} ; \quad E^{\circ}=0.54 \mathrm{~V}\) 
    \(\mathrm{Cl}_{2}+2 e^{-} \rightarrow 2 \mathrm{Cl}^{-} ; \quad E^{\circ}=1.36 \mathrm{~V}\) 
    \(\mathrm{Mn}^{3+}+e^{-} \rightarrow \mathrm{Mn}^{2+} ; \quad E^{\circ}=1.50 \mathrm{~V}\) 
    \(\mathrm{Fe}^{3+}+e^{-} \longrightarrow \mathrm{Fe}^{2+} ; \quad E^{\circ}=0.77 \mathrm{~V}\) 
    \(\mathrm{O}_{2}+4 \mathrm{H}^{+}+4 e^{-} \longrightarrow 2 \mathrm{H}_{2} \mathrm{O} ; E^{\circ}=1.23 \mathrm{~V}\) 
    The following questions are multiple choice questions. Choose the most appropriate answer:
    (i) Which of the following statements is correct?

    (a) CI- is oxidised by O2 (b) Fe2+ is oxidised by iodine
    (c) I- is oxidised by chlorine. (d) Mn2+ is oxidised by chlorine

    (ii) Mn3+ is not stable in acidic medium, while Fe3+is stable because

    (a) O2 oxidises Mn2+ to Mn3+
    (b) O2 oxidises both Mn2+ to Mn3+ and Fe2+ to Fe3+
    (c) Fe3-oxidises H2O to O2
    (d) Mn3+ oxidises H2O to O2

    (iii) The strongest reducing agent in the aqueous solution is

    (a) I- (b) Cl- (c) Mn2+ (d) Fe2+

    (iv) The emf for the following reaction is
    \(\mathrm{I}_{2}+\mathrm{KCl} \rightleftharpoons 2 \mathrm{KI}+\mathrm{Cl}_{2}\) 

    (a) -0.82 V (b) +0.82 V (c) -0.73 V (d) +0.73 V
  • 3)

    Read the passage given below and answer the following questions :
    All chemical reactions involve interaction of atoms and molecules. A large number of atoms/molecules are present in a few gram of any chemical compound varying with their atomic/molecular masses. To handle such large number conveniently, the mole concept was introduced. All electrochemical cell reactions are also based on mole concept. For example, a 4.0 molar aqueous solution of NaCI is prepared and 500 mL of this solution is electrolysed. This leads to the evolution of chlorine gas at one of the electrode. The amount of products formed can be calculated by using mole concept.
    The following questions are multiple choice questions. Choose the most appropriate answer :
    (i) The total number of moles of chlorine gas evolved is 

    (a) 0.5 (b) 1.0 (c) 1.5 (d) 1.9

    (ii) If cathode is a Hg electrode, then the maximum weight of amalgam formed from this solution is

    (a) 300 g (b) 446 g (c) 396 g (d) 296 g

    (iii) In the electrolysis, the number of moles of electrons involved are

    (a) 2 (b) 1 (c) 3 (d) 4

    (iv) In electrolysis of aqueous NaCl solution when Pt electrode is taken, then which gas is liberated at cathode?

    (a) H2 gas (b) C2 gas (c) O2 gas (d) None of these
  • 4)

    Read the passage given below and answer the following questions:
    The concentration of potassium ions inside a biological cell is at least twenty times higher than the outside. The resulting potential difference across the cell is important in several processes such as transmission of nerve impulses and maintaining the ion balance. A simple model for such a concentration cell involving a metal M is M(s) | M+(aq.; 0.05 molar) || M+(aq; 1 molar) |M(s).
    The following questions are multiple choice questions. Choose the most appropriate answer:
    (i) For the above cell,

    (a) \(E_{\text {cell }}<0 ; \Delta G>0\) (b) \(E_{\text {cell }}>0 ; \Delta G<0\) (c) \(E_{\text {cell }}<0 ; \Delta G^{\circ}>0\) (d) \(E_{\text {cell }}>0 ; \Delta G^{\circ}<0\)

    (ii) The value of equilibrium constant for a feasible cell reaction is

    (a) < 1 (b) = 1 (c) > 1 (d) zero

    (iii) What is the emf ofthe cell when the cell reaction attains equilibrium?

    (a) 1 (b) 0 (c) > 1 (d) < 1

    (iv) The potential of an electrode change with change in

    (a) concentration of ions in solution (b) position of electrodes
    (c) voltage of the cell (d) all of these
  • 5)

    Read the passage given below and answer the following questions:
    The electrochemical cell shown below is concentration cell. M|M2+ (saturated solution of a sparingly soluble salt, MX2 ) || M2+ (0.001 mol dm-3 ) | M The emf of the cell depends on the difference in concentrations of M2+ ions at the two electrodes. The emf of the cell at 298 K is 0.059 V.
    The following questions are multiple choice questions. Choose the most appropriate answer:
    (i) The solubility product (Ksp, mol3 dm-9) of MX2 at 298 K based on the information available for the given concentration cell is (take 2.303 x R x 298/P = 0.059)

    (a) 2 x 10-15 (b) 4 x 10-15 (c) 3 x 10-12 (d) 1 x 1012

    (ii) The value of \(\Delta G\) (in kJ mol-1) for the given cell is (take 1F = 96500 C mol-1)

    (a) 3.7 (b) -3.7 (c) 10.5 (d) -11.4

    (iii) The equilibrium constant for the following reaction is
    \(\mathrm{Fe}^{2+}+\mathrm{Ce}^{4+} \rightleftharpoons \mathrm{Ce}^{3+}+\mathrm{Fe}^{3+}\) 
    (Given, \(E^{0} \mathrm{Ce}^{4+} / \mathrm{Ce}^{3+}=1.44\)  and Eo \(E_{\mathrm{Fe}^{3+} / \mathrm{Fe}^{2+}}=0.68 \mathrm{~V}\))

    (a) 7.6 x 1012 (b) 6.5 x 1010 (c) 5.2 x 109 (d) 3.4 x 1012

    (iv) To calculate the emf of the cell, which of the following options is correct?

    (a) emf = Ecathode - Eanode (b) emf = Eanode - Ecathode
    (c) emf = Eanode + Ecathode (d) None of these

Class 12th Chemistry - Solution Case Study Questions and Answers 2022 - 2023 - by Study Materials - View & Read

  • 1)

    Read the passage given below and answer the following questions:
    The concentration of a solute is very important in studying chemical reactions because it determines how often molecules collide in solution and thus indirectly determine the rate of reactions and the conditions at equilibrium.
    There are several ways to express the amount of solute present in a solution. The concentration of a solution is a measure of the amount of solute that has been dissolved in a given amount of solvent or solution. Concentration can be expressed in terms of molarity, molality, parts per million, mass percentage, volume percentage, etc.
    The following questions are multiple choice questions. Choose the most appropriate answer:
    (i) The molarity (in mol L-1) of the given solution will be

    (a) 1.56 (b) 1.89 (c) 0.263 (d) 1.44

    (ii) Which of the following is correct relationship between mole fraction and molality?

    \(\text { (a) } x_{2}=\frac{m M_{1}}{1+m M_{1}}\) \(\text { (b) } x_{2}=\frac{m M_{1}}{1-m M_{1}}\)
    \(\text { (c) } x_{2}=\frac{1+m M_{1}}{m M_{1}}\) \(\text { (d) } x_{2}=\frac{1-m M_{1}}{m M_{1}}\)

    (iii) Which of the following is temperature dependent?

    (a) Molarity (b) Molality
    (c) Mole fraction (d) Mass percentage

    (iv) Which of the following is true for an aqueous solution of the solute in terms of concentration?

    (a) 1 M = 1 m (b) 1M > 1m
    (c) 1M < 1 m (d) Cannot be predicted
  • 2)

    Read the passage given below and answer the following questions:
    At 298 K, the vapour pressure of pure benzene, C6H6 is 0.256 bar and the vapour pressure of pure toluene
    C6H5CH3 is 0.0925 bar. Two mixtures were prepared as follows:
    (i) 7.8 g of C6H6 + 9.2 g of toluene
    (ii) 3.9 g of C6H6 + 13.8 g of toluene
    The following questions are multiple choice questions. Choose the most appropriate answer:
    (i) The total vapour pressure (bar) of solution 1 is

    (a) 0.128 (b) 0.174 (c) 0.198 (d) 0.258

    (ii) Which of the given solutions have higher vapour pressure?

    (a) I (b) II
    (c) Both have equal vapour pressure (d) Cannot be predicted

    (iii) Mole fraction of benzene in vapour phase in solution 1 is

    (a) 0.128 (b) 0.174 (c) 0.734 (d) 0.266

    (iv) Solution I is an example of a/an

    (a) ideal solution (b) non-ideal solution with positive deviation
    (c) non-ideal solution with negative deviation (d) can't be predicted
  • 3)

    Read the passage given below and answer the following questions:
    An ideal solution may be defined as the solution which obeys Raoult's law exactly over the entire range of concentration. The solutions for which vapour pressure is either higher or lower than that predicted by Raoult's law are called non-ideal solutions.
    Non-ideal solutions can show either positive or negative deviations from Raoult's law depending on whether the A-B interactions in solution are stronger or weaker than A - A and B - B interactions.
    The following questions are multiple choice questions. Choose the most appropriate answer:
    (i) Which of the following solutions is/are ideal solution(s)?
    (i) Bromoethane and iodoethane (ii) Acetone and chloroform
    (iii) Benzene and acetone (iv)n-heptane and n-hexane

    (a) only 1 (b) I and II (c) II and III (d) I and IV

    (ii) Which of the following is not true for positive deviations?

    (a) The A-B interactions in solution are weaker than the A -A and B -B interactions.
    (b) \(P_{A}<P_{A}^{\circ} x_{A} \text { and } P_{B}<P_{B}^{\circ} x_{B}\)
    (c) Carbon tetrachloride and chloroform mixture is an example of positive deviations.
    (d) All of these

    (iii) For water and nitric acid mixture which of the given graph is correct?

    (C) Both of these (d) None of these

    (iv) Water- HCl mixture
    I. shows positive deviations  II. forms minimum boiling azeotrope
    III. shows negative deviations  IV. forms maximum boiling azeotrope

    (a) I and II (b) II and III
    (c) I and IV (d) III and IV
  • 4)

    Read the passage given below and answer the following questions:
    The properties of the solutions which depend only on the number of solute particles but not on the nature of the solute are called colligative properties. Relative lowering in vapour pressure is also an example of colligative properties.
    For an experiment, sugar solution is prepared for which lowering in vapour pressure was found to be 0.061 mm of Hg. (Vapour pressure of water at 20°C is 17.5 mm of Hg.)
    The following questions are multiple choice questions. Choose the most appropriate answer:
    (i) Relative lowering of vapour pressure for the given solution is

    (a) 0.00348 (b) 0.061 (c) 0.122 (d) 1.75

    (ii) The vapour pressure (mm of Hg) of solution will be

    (a) 17.5 (b) 0.61 (c) 17.439 (d) 0.00348

    (iii) Mole fraction of sugar in the solution is

    (a) 0.00348 (b) 0.9965 (c) 0.061 (d) 1.75

    (iv) The vapour pressure (mm of Hg) of water at 293 K when 25 g of glucose is dissolved in 450 g of water is

    (a) 17.2 (b) 17.4 (c) 17.120 (d) 17.02
  • 5)

    Read the passage given below and answer the following questions:
    Few colligative properties are:
    (a) relative lowering of vapour pressure: depends only on molar concentration of solute (mole fraction) and independent of its nature.
    (b) depression in freezing point: it is proportional to the molal concentration of solution.
    (c) elevation of boiling point: it is proportional to the molal concentration of solute.
    (d) osmotic pressure: it is proportional to the molar concentration of solute.
    A solution of glucose is prepared with 0.052 g at glucose in 80.2 g of water. (Kf = 1.86 K kg mol-1 and Kb = 5.2 K kg mol-1)
    The following questions are multiple choice questions. Choose the most appropriate answer:
    (i) Molality of the given solution is

    (a) 0.0052 m (b) 0.0036 m (c) 0.0006 m (d) 1.29 m

    (ii) Boiling point for the solution will be

    (a) 373.05 K (b) 373.15 K (c) 373.02 K (d) 372.98 K

    (iii) The depression in freezing point of solution will be

    (a) 0.0187 K (b) 0.035 K (c) 0.082 K (d) 0.067 K

    (iv) Mole fraction of glucose in the given solution is

    (a) 6.28 x 10-5 (b) 1.23 x 10-4 (c) 0.00625 (b) 0.00028

Class 12th Chemistry - The Solid State Case Study Questions and Answers 2022 - 2023 - by Study Materials - View & Read

  • 1)

    Read the passage given below and answer the following questions:
    The adjective, 'crystalline' when applied to solids, implies an ideal crystal in which the structural units, termed as unit cells, are repeated regularly and indefinitely in three dimensions in space. The unit cell, containing at least one molecule, has a definite orientation and shape defined by the translational vectors, a, band c. The unit cell therefore has a definite volume, V that contains the atoms and molecules necessary for generating the crystal. Every crystal can be classified as a member of one of the seven possible crystal systems or crystal classes that are defined by the relationships between the individual dimensions, a, band c of the unit cell and between the individual angles, \(\alpha, \beta \ and \ \gamma\) of the unit cell. The structure of the given crystal may be assigned to one of the 7 crystal systems, to one of the 14 Bravais lattices, and to one of the 230 space groups. These uniquely define the possible ways of arranging atoms in a three-dimensional solid. Based on these observations, seven crystal systems were identified: triclinic, monoclinic, trigonal or rhombohedral, tetragonal, hexagonal, rhombic or orthorhombic and cubic.
    The following questions are multiple choice questions. Choose the most appropriate answer :
    (i) The crystal system of a compound with unit cell dimensions, a = 0.387 nm, b = 0.387 nm and c = 0.504 nm and \(\alpha, \beta \ =90^o\) and \(\gamma\) = 120° is

    (a) cubic (b) hexagonal (c) orthorhombic (d) rhombohedral.

    (ii) The unit cell with the structure given below represents __________crystal system.

    a) cubic  (b) orthorhombic (c) tetragonal (d) trigonal

    (iii) In a triclinic crystal

    \(\text { (a) } a=b=c, \quad \alpha=\beta=\gamma \neq 90^{\circ}\) \(\text { (b) } a \neq b=c, \quad \alpha=\beta=\gamma=90^{\circ}\)
    \(\text { (c) } a \neq b \neq c, \quad \alpha \neq \beta \neq \gamma \neq 90^{\circ}\) \(\text { (d) } a \neq b \neq c, \quad \alpha=\gamma=90^{\circ}, \beta \neq 90^{\circ}\)

    (iv) The unit cell with dimensions \(\alpha=\beta=\gamma=90^{\circ}, a=b \neq c \) is

    (a) cubic (b) triclinic (c) hexagonal (d) tetragonal
  • 2)

    Read the passage given below and answer the following questions:
    In contrast to the disorders of gases and liquids, there is translational order in crystals. However, disordered or amorphous solids also exist which lack such order, they are really highly viscous liquids. In translational order entire structure or lattice, can be generated by repeated replication of a small regular figure, termed as unit cell. The planes of any crystalline structure can be specified using Miller indices, which is also serve to identify single crystal faces.
    The ordered structure or lattice, of a solid can be determined by X-ray or neutron diffraction studies, in which a beam of X-rays of neutrons is scattered from the sample to produce a diffraction pattern which can be analyzed to reveal the crystal structure of the sample. All crystal lattices can be classified into 14 Bravais lattices belonging to 7 systems. For example, the simple cubic, face-centred cubic and body-centred cubic lattices are the 3 lattices of the cubic system. Cubic and hexagonal close- packed structures have the structure of tightly packed spheres where each sphere touches 12 neighbours, 6 in the same plane and 3 above and 3 below. These two close-packed structures differ in the placement of successive planes or layers. For the hexagonal close packing, a third layer is laid down to reproduce the first layer, so that the structure could be represented by ABABAB .... For cubic close packing, third layer is again displaced, corn spending to ABCABC ...
    The following questions are multiple choice questions. Choose the most a propriate answer:
    (i) In hexagonal close packing, a sphere has coordination number of

    (a) 4 (b) 6 (c) 8 (d) 12

    (ii) Which of the following arrangements correctly represents hexagonal and cubic close packed structure respectively?

    (a) ABCABC .. and ABAB ... (b) ABAB ... and  ABCABC ..
    (c) Both have ABAB ... arrangement. (d) Both have ABCABC .. arrangement

    (iii) The arrangement of the first two layers, one above the other in hep and eep arrangements is

    (a) exactly same in both cases (b) partly same and partly different
    (c) different from each other (d) nothing definite

    (iv) Which of the following statements about amorphous solids is incorrect?

    (a) They melt over a range of temperature.
    (b) There is no orderly arrangement of particles.
    (c) They are anisotropic
    (d) They are rigid and incompressible.
  • 3)

    Read the passage given below and answer the following questions:
    In ideally ionic structures, the coordination numbers of the ions are determined by electrostatic considerations. Cations surround themselves with as many anions as possible and vice versa. This maximizes the attractions between neighbouring ions of opposite charge and hence maximizes the lattice energy of the crystal. This requirement led to the formulation of the radius ratio rule for ionic structures in which the ions and the structure adopted for a particular compound depend on the relative sizes of the ions. Thus, for the stable ionic crystalline structures, there is definite radius ratio limit for a cation to fit perfectly in the lattice of anions, called radius ratio rule. This depends upon the ratio of radii of two types of ions, r+/r-.
    This ratio for coordination numbers 3, 4, 6 and 8 are respectively 0.155 - 0.225, 0.225 - 0.414, 0.414 - 0.732 and 0.732 - 1.000. The coordination number of ionic solids also depends upon temperature and pressure. On applying high pressure, coordination number increases. On the other hand, on applying high temperature, it decreases.
    The following questions are multiple choice questions. Choose the most appropriate answer:
    (i) The ionic radii of K+, Rb+ and Brare 137, 148 and 195 pm. The coordination number of cation in RbBr and KBr structures are respectively

    (a) 8,6 (b) 6,4 (c) 6,8 (d) 4,6

    (ii) If the radius of Na+ ion is 95 pm and that of Clion is 181 pm, the coordination number of Na+ ion is

    (a) 6 (b) 4 (c) 8 (d) 12

    (iii) Which is not the correct statement for ionic solids in which positive and negative ions are held by strong electrostatic attractive forces?

    (a) The radius ratio r+/r- increases as coordination number increases
    (b) As the difference in size of ions increases, coordination number increases.
    (c) When coordination number is eight, r+/r- ratio lies between 0.225 to 0.414
    (d) In ionic solid of the type AX (ZnS, wurtzite), the coordination number of Zn2+ and S2- respectively are 4 and 4

    (iv) If the pressure of CsCl is increased, then its coordination number will

    (a) increase (b) remain the same (c) decrease (d) none of these
  • 4)

    Read the passage given below and answer the following questions:
    In an ideal crystal, there must be regular repeating arrangement of the constituting particles and its entropy must be zero at absolute zero temperature. However, it is impossible to obtain an ideal crystal and it suffers from certain defects called imperfections. In pure crystal, these defects arises either due to disorder or dislocation of the constituting particles from their normal positions or due to the movement of the particles even at absolute zero temperature. Such defects increase with rise in temperature. In addition to this, certain defects arise due to the presence of some impurities. Such defects not only modify the existing properties of the crystalline solids but also impart certain new characteristics to them.
    The following questions are multiple choice questions. Choose the most appropriate answer:
    (i) AgCI is crystallized from molten AgCI containing a little CdCI2. The solid obtained will have

    (a) cationic vacancies equal to number of Cd2+ ions incorporated
    (b) cationic vacancies equal to double the number of Cd2+ ions
    (c) anionic vacancies
    (d) neither cationic nor anionic vacancies.

    (ii)The ionic substances in which the cation and anion are of almost similar size shows

    (a) non-stoichiometric defect (b) Schottky defect
    (c) Frenkel defect (d) all of these.

    (iii) If A13+ions replace Na+ ions at the edge centres of NaCl lattice, then the number of vacancies in 1mole of NaCI will be

    (a) 3.01 x 1023 (b) 6.02 x 1023 (c) 9.03 x 1023 (d) 12.04 x 1023

    (iv) Which of the following gives both Frenkel and Schottky defect?

    (a) AgCl (b) CsCI (c) KCI (d) AgBr
  • 5)

    Read the passage given below and answer the following questions:
    The idealized ionic solid consists of two interpenetrating lattices of oppositely-charged point charges that are held in place by a balance of coulombic force of long range. But real ions occupy space, no such "perfect" ionic solid exists in nature. Chemists usually apply the term "ionic solid" to binary compounds of the metallic elements of groups 1 - 2 with one of the halogen elements or oxygen. The most well known ionic solid is sodium chloride, also known by its geological names as rock-salt or halite. Structurally, each ion in sodium chloride is surrounded and held in tension by six neighbouring ions of opposite charge; this is known as (6, 6) coordination. The resulting crystal lattice is of a type known as simple cubic. There are many other fundamental ionic structures (not all cubic) and these are:
    Zinc blende structure (ZnS) : having ccp arrangement of S2- and Zn2+ in alternate tetrahedral voids; Wurtzite structure (ZnS) : having hcp arrangement of S2- and Zn2+ in alternate tetrahedral voids; Fluorite structure (CaF2) : having ccp arrangement of Ca2+ and F- in all tetrahedral voids; Antifluorite structure (Na2O): having ccp arrangement of O2- and Na+ in all tetrahedral voids. These solids tend to be quite hard and have high melting points.
    The following questions are multiple choice questions. Choose the most appropriate answer:
    (i) In NaCl crystal, each Cl- ion is surrounded by

    (a) 4 Na+ ions (b) 6 Na+ ions (c) 1Na+ ion (d) 2 Na+ ions.

    (ii) In an antifluorite structure, cations occupy

    (a) tetrahedral voids (b) centre of cube (c) octahedral voids (d) corners of cube

    (iii) Antifluorite structure is derived from fluorite structure by

    (a) heating fluorite crystal lattice
    (b) subjecting fluorite structure to high pressure
    (c) interchanging the positions of positive and negative ions in the lattice
    (d) none of these.

    (iv) Ionic solid BaF2 has which kind of structure?

    (a) Fluorite (b) Antifluorite (c) Wurtzite (d) Rock-salt

12th Standard CBSE Chemistry Public Exam Model Question 2020 - by Archana Singh - Deoghar - View & Read

  • 1)

    In an ionic compound A+X-,the radii of A+ and X- ions are 1.0pm and 2.0pm respectively.The volume of the unit cell of the crystal AX will be

  • 2)

    For a weak monobasic acid, if pKa = 4, then at a concentration of 0.01 M of the acib solution, the van't Hoff factor is

  • 3)

    Rust is a mixture of

  • 4)

    At high pressure, the following reaction is of zero order.
    2NH3  \(\xrightarrow [ Platinum \ catalyst ]{ 1130K } \) (g) N2(g) + 3H2(g)
    Which of the following options are correct for this reaction ?

  • 5)

    The correct order of adsorption of the gases studied will be

12th Standard CBSE Chemistry Public Exam Sample Question 2020 - by Archana Singh - Deoghar - View & Read

  • 1)

    The vapour pressure of a dilute solution of a solute is not influenced by

  • 2)

    Conductivity k, is equal to __________________.

  • 3)

    Collision Theory is applicable to

  • 4)

    Adsorption is accompanied by

12th Standard CBSE Chemistry Public Exam Important Question 2019-2020 - by Archana Singh - Deoghar - View & Read

  • 1)

    A metal has cubic close packed (ccp) arrangement,the layer sequence of which is shown below: A face diagonal passes through the centre of atom 4 and the centre(s) of which other atom(s)?

  • 2)

    Increasing the temperature of an aqueous solution will cause

  • 3)

    Given below are the half cell reactions :
    \({ Mn }^{ 2+ }+{ 2e }^{ - }\longrightarrow Mn, \ { \ \ \ \ E }^{ 0 }=-1.18V\)
    \(\\ 2({ Mn }^{ 3+ }+{ e }^{ - }\longrightarrow { Mn }^{ 2+ }), \ \ { E }^{ 0 } \ =+1.51V \ \)

    The E0 for \(3{ Mn }^{ 2+ }\longrightarrow Mn+2{ Mn }^{ 3+ }\) will be

  • 4)

    When ethyl acetate was hydrolysed in presence of 0.1 N HCI, the rate constant was found to be 5.40 \(\times\) 10-5 s-1 . From these values we can say that

  • 5)

    What happens when a lyophilic sol is added to a lyophobic sol?

12th Standard Chemistry Board Exam Sample Question 2020 - by Archana Singh - Deoghar - View & Read

  • 1)

    In a solid 'AB' having NaCl structure, 'A' atoms occupy the corners of the cubic unit cell.If all the face-centred atoms along one of the axes are removed,then the resultant stoichiometry of the solid is

  • 2)

    The sequence of ionic mobility in the aqueous solution is

  • 3)

    At high pressure, the following reaction is of zero order.
    2NH3  \(\xrightarrow [ Platinum \ catalyst ]{ 1130K } \) (g) N2(g) + 3H2(g)
    Which of the following options are correct for this reaction ?

  • 4)

    An emulsion cannot be broken by.........................and......................... .

12th Standard Chemistry Board Exam Model Question 2019-2020 - by Archana Singh - Deoghar - View & Read

  • 1)

    Structure of a mixed oxide is cubic close-packed(c.c.p) The cubic unit cell of mixed oxide is composed of oxide ions.One fourth of the tetrahedral voids are occupied by divalent metal A and the octahedral voids are occupied by a monovalent metal B.The formula of the oxide is:

  • 2)

    Solution A contains 7 g/L MgCI2 and solution B contains 7 g/L of NaCI. At room temperature, the osmotic pressure of

  • 3)

    In the lead-acid battery during charging, the cathode reaction is

  • 4)

    According to Maxwell Boltzmann distribution of energy, ....... .

  • 5)

    Which of the following result the chemist must have observed about his studies with KCI solution ?

12th Standard CBSE Chemistry Public Model Question Paper IV 2019 - 2020 - by Archana Singh - Deoghar - View & Read

  • 1)

    Which of the following defects id is also known as dislocation defect?

  • 2)

    The density (in g mL-1) of a 3.60 M sulphuric acid solution that is 29% H2SO4 (Molar mass = 98 g mol-1) by mass will be 

  • 3)

    If Zn2+ / Zn electrode is diluted 100 times, then the change in emf is

  • 4)

    At high pressure, the following reaction is of zero order.
    2NH3  \(\xrightarrow [ Platinum \ catalyst ]{ 1130K } \) (g) N2(g) + 3H2(g)
    Which of the following options are correct for this reaction ?

  • 5)

    Mond's process of is used for refining of

12th Standard CBSE Chemistry Public Model Question Paper III 2019 - 2020 - by Archana Singh - Deoghar - View & Read

  • 1)

    KCl crystallises in the same type of lattice as does NaCl.Given that rNa+/rCl-=0.55 and rK+/rCl-=0.74.Calculate the side of the unit cell of KCl to that of NaCl

  • 2)

    The osmotic pressure of 0.1 M aqueous solution of NaCl is ................ Osmotic pressure of 0.1 M aqueous solution of glucose

  • 3)

    The metal that cannot be obtained by electrolysis of an aqueous solution of its salt is

  • 4)

    A relation P \(\longrightarrow\) Q is completed 25 % in 25 min 50 % completed in 25 min if [P] is halved, 25% completed in 50 min if [P] is doubled. The order of reaction is 

  • 5)

    Brine solution on electrolysis will give

12th Standard CBSE Chemistry Public Model Question Paper II 2019 - 2020 - by Archana Singh - Deoghar - View & Read

  • 1)

    If the position of Na+ and Cl- are interchanged in NaCl having fcc arrangement of Cl ions then in the unit cell of Nacl - ions then in the unit cell of Nacl

  • 2)

    Two liquids X and Y form an ideal solution. The mixture has a vapour pressure of 400 mm at 300 K when mixed in the molar ratio of 1:1 and a vapour pressure of 350 mm when mixed in the molar ratio of 1:2 at the same temperature. The vapour pressures of the pure liquids X and Y respectively are

  • 3)

    The equivalent conductance of NaCl at concentration C and at infinite dilution are \({ \lambda }_{ c }\) and \({ \lambda }_{ \infty }\) respectively. The correct relationship between \({ \lambda }_{ c }\) and \({ \lambda }_{ \infty }\) is given by (where the constant B is positive)

  • 4)

    A chemical reaction was carried out at 300 K and 280 K the rate constants were found to be K1 and K2 respectively. Then

  • 5)

    Bauxite ore is made up Al2O3 +SiO2+TiO2+Fe2O3.This ore treated with conc.NaOH solution at 500K and 35 bar pressure for few hours and filtered hot.In the filtrate ,the species present are:

12th Standard CBSE Chemistry Public Model Question Paper I 2019 - 2020 - by Archana Singh - Deoghar - View & Read

  • 1)

    KCl crystallises in the same type of lattice as does NaCl.Given that rNa+/rCl-=0.55 and rK+/rCl-=0.74.Calculate the side of the unit cell of KCl to that of NaCl

  • 2)

    Which of them is not equal to zero for an ideal solution ?

  • 3)

    The reaction taking place in the cell
    Pt  | H2 (g) |  HCl (1.0 M) | AgCl | Ag is 1 atm

  • 4)

    The role of a catalyst is to change .............. .

  • 5)

    \(\underset{\text { Impure }}{\mathrm{Ni}}+4 \mathrm{CO} \stackrel{330-350 \mathrm{~K}}{\longrightarrow} \mathrm{Ni}(\mathrm{CO})_{4}\)
    \(\mathrm{Ni}(\mathrm{CO})_{4} \stackrel{450-470 \mathrm{~K}}{\longrightarrow} \underset{\text { Pure }}{\mathrm{Ni}}+4 \mathrm{CO}\)
    The above reactions are of the purification of Ni by

12th CBSE Chemistry - Public Model Question Paper 2019 - 2020 - by Archana Singh - Deoghar - View & Read

  • 1)

    In which pair most efficient packing packing is present?

  • 2)

    A hypothetical reaction, A2 + B2 \(\longrightarrow\) 2 AB, follows the mechanism as given below :

    A2 \(\rightleftharpoons \) A + A

    A + B2 \(\longrightarrow\) AB + B

    A + B \(\longrightarrow\) AB

    The overall order of reaction is 

  • 3)

    In the froth floatation process for benefaction of the ores,the ore particles float because

  • 4)

    Al2 O3 is reduced by electrolysis at low potentials and high currents. If 4.0 x 104 amperes of current is passed through molten Al2 O3 for 6 hours, what mass of aluminium is produced ? (Assume 100% current efficiency. At. mass of Al = 27 g mol-1).

  • 5)

    Which of the following are amphoteric oxides ?
    \({ Mn }_{ 2 }{ O }_{ 7 },{ CrO }_{ 3 },{ Cr }_{ 2 }{ O }_{ 3 },CrO,{ V }_{ 2 }{ O }_{ 5 },{ V }_{ 2 }{ O }_{ 4 }\)

CBSE 12th Chemistry - Chemistry in Everyday Life Model Question Paper - by Archana Singh - Deoghar - View & Read

  • 1)

    Birth control pills contain a mixture of

  • 2)

    Which of the following is not a broad spectrum antibiotic ?

  • 3)

    A narrow spectrum antibiotic is active against ........... .

  • 4)

    Which of the following enhances lathering property of soap ?

  • 5)

    Glycerol is added to soap. It functions ........... .

CBSE 12th Chemistry - Biomolecules Model Question Paper - by Archana Singh - Deoghar - View & Read

  • 1)

    Which of the following is incorrect for glucose?

  • 2)

    The term anomer of glucose refers to 

  • 3)

    Which of the following disaccharide gives a ketose and an aldose only on hydrolysis?

  • 4)

    The amino acids are the product of the digestion of 

  • 5)

    Adenosine is an example of 

CBSE 12th Chemistry - Organic Compounds Containing Nitrogen Model Question Paper - by Archana Singh - Deoghar - View & Read

  • 1)

    Amine that cannot be prepared by Gabriel phthalimide synthesis is

  • 2)

    Choose the incorrect statement

  • 3)

    The amine that reacts with NaNO2+ HCI to give yellow oily liquid is

  • 4)

    When aniline is heated with conc.H2SO4 at 455-475 K, it forms:

  • 5)

    In the following reaction, X is \(X\overset { Bromination }{ \longrightarrow } Y\overset { NaNO_{ 2 }\quad /HCI }{ \longrightarrow } Z\overset { Boiling }{ \underset { { C }_{ 2 }{ H }_{ 5 }OH }{ \longrightarrow } } Tribromobenzene\)

CBSE 12th Chemistry - Aldehydes , Ketones and Carboxylic Acids Model Question Paper - by Archana Singh - Deoghar - View & Read

  • 1)

    In a reaction, RCHO is reduced to RCH3 using amalgamated zinc and concentrated HCl and warming the solution. The reaction is known as

  • 2)

    Which of the following organic compounds answers both iodoform test and Fehling's test?

  • 3)

    The product formed in Aldol condensation is

  • 4)

    One mole of a symmetrical alkene on ozonolysis gives two moles of an aldehyde having a molecular mass of 44 u. The alkene is

  • 5)

    Which of the following is the industrial method of preparation of acetaldeyde ?

CBSE 12th Chemistry - Alcohols , Phenols and Ethers Model Question Paper - by Archana Singh - Deoghar - View & Read

  • 1)

    Which of the following will produce only one product on reduction with LiAIH4?

  • 2)

    Phenol reacts with bromine in CS2 to give

  • 3)

    Among the alkenes which one produces tertiary butyl alcohol on hydration?

  • 4)

    Cyclohexene is best prepared from cyclohexanol by which of the following reagents:

  • 5)

    From amongst the following alcohols, the one that would react fastest with Conc.HCl and anhydrous ZnCl2, is

CBSE 12th Chemistry - Haloalkanes and Haloarenes Model Question Paper - by Archana Singh - Deoghar - View & Read

  • 1)

    A compound is formed by substitution of two chlorine for two hydrogens in propane. The number of possible isomeric compound is

  • 2)

    Which of the following possesses highest melting point?

  • 3)

    Which of the following are arranged in the decreasing order of dipole moment?

  • 4)

    Which of the following is not chiral?

  • 5)

    In the addition of HBr to propene in the absence of peroxides, the first step involves the addition of 

CBSE 12th Chemistry - Coordination Compounds Model Question Paper - by Archana Singh - Deoghar - View & Read

  • 1)

    The ionization isomer of [Cr(H2O)4Cl(NO2)]Cl is 

  • 2)

    The hybridisation of Fe in K4[Fe(CN)6] is 

  • 3)

    Which of the following is an outer orbital complex? 

  • 4)

    Which of the following is not considered as an organometallic compound? 

  • 5)

    2 : 4 Dinitrophenyl hydrazine is an example for

CBSE 12th Chemistry - Polymers Model Question Paper - by Archana Singh - Deoghar - View & Read

  • 1)

    Which of the following is a linear polymer?

  • 2)

    Given the polymers: A=Nylon ; B=Buna-S; C=Polythene. Arrange these in increasing order of their intermolecular forces (Lower to higher)

  • 3)

    Which of the following is not a semisynthetic polymer?

     

  • 4)

    Which of the following polymers are condensation polymers?

  • 5)

    Among cellulose,poly(vinyl chloride),nylon and natural rubber,the polymer in which the intermolecular force of attraction is weakest is

CBSE 12th Chemistry - The d- and f- Block Elements Model Question Paper - by Archana Singh - Deoghar - View & Read

  • 1)

    Which of the following is not a transition element ?

  • 2)

    The maximum oxidation state of osmium is

  • 3)

    Which of the following ion is colourless in aqueous solution ?

  • 4)

    Which metal has highest density ?

  • 5)

    Permanent magnets are generally made of alloys of

CBSE 12th Chemistry - The p-Block Elements Model Question Paper - by Archana Singh - Deoghar - View & Read

  • 1)

    Which of the following on heating does not give nitrogen gas?

  • 2)

    Which one of the following arrangements represents the correct order of electron gain enthalpy (with negative sign) of the given atomic species?

  • 3)

    The formation of O2+[PtF6]is the basis for the formation of xenon fluorides. This is because

  • 4)

    Affinity for hydrogen decreases in the group from fluorine to iodine. Which of the halogen acids should have highest bond dissociation enthalpy?

  • 5)

    Which of the following elements does not show allotropy?

CBSE 12th Chemistry - General Principles and Processes of Isolation of Elements Model Question Paper - by Archana Singh - Deoghar - View & Read

  • 1)

    The salt which is least likely to be found in minerals

  • 2)

    Ore dressing of iron ore is done by

  • 3)

    For which of the following ores,froth floatation process is applicable?

  • 4)

    Which of the following benefaction processes is used for the mineral,Al2O3.2H2O?

  • 5)

    Heating pyrites to remove sulphur is called

CBSE 12th Chemistry - Surface Chemistry Model Question Paper - by Archana Singh - Deoghar - View & Read

  • 1)

    The most adsorbed gas on activated charcoal is

  • 2)

    Which of the following is correct about lyophilic sols?

  • 3)

    The function of gum arabic in the preparation of Indian ink is

  • 4)

    Which of the following are the characteristics of chemisorption ?

    1. High heat of adsorption

    2. Irreversibility

    3. Low activation energy

    Select the correct answer using the code given below :

  • 5)

    Which of the following relation is correct ?
    (i) x/m = constant at high pressure
    (ii) x/m = constant xp1/n (at intermediate pressure)
    (iii) x/m = constant xpn (at lower pressure)

CBSE 12th Chemistry - Chemical Kinetics Model Question Paper - by Archana Singh - Deoghar - View & Read

  • 1)

    Rate constant of a reaction (k) is 175 litre2 mol-2 sec-1. What is the order of reaction?

  • 2)

    75% of the first order reaction was completed in 32 min. 50% of the reaction was completed in

  • 3)

    The half-life period of a radioactive element is 20 days. What will be the remaining mass of 100 g of it after 60 days?

  • 4)

    For a complex reaction ............. .

  • 5)

    Which of the following statements are in accordance with the Arhenius equation ?

CBSE 12th Chemistry - Electrochemistry Model Question Paper - by Archana Singh - Deoghar - View & Read

  • 1)

    The time required to liberate one gram equivalent of an element by passing one ampere current through its solution is

  • 2)

    Which has the highest oxidizing power ?

  • 3)

    For the reduction of silver ions with copper metal, the standard cell potential was found to be + 0.46 V at 25°C. The value of standard Gibbs energy, \({ \Delta G }^{ ° }\) will be (F = 96500 C mol-1)

  • 4)

    Using the data given below find out the strongest reducing agent.
    \({ E }_{ { Cr }_{ 2 }{ O }_{ 7 }^{ 2- }/{ Cr }^{ 3+ } }^{ \circleddash }=1.33V\ ,\ { E }_{ { Cl }_{ 2 }/{ Cl }^{ - } }^{ \circleddash }=1.36V\)
    \({ E }_{ { MnO }_{ 4 }^{ - }/{ Mn }^{ 2+ } }^{ \circleddash }=1.51V\ ,\ { E }_{ { Cr }^{ 3+ }/{ Cr } }^{ \circleddash }=-0.74V\)

  • 5)

    One faraday of electricity is passed thorugh molten Al2 O3 , aqueous solution of CuSO4 and molten NaCl taken in three different electrolytic cells connected in series. The mole ratio of Al, Cu and Na deposited at the respective cathode is

CBSE 12th Chemistry - Solution Model Question Paper - by Archana Singh - Deoghar - View & Read

  • 1)

    200 mL of water is added to 500mL of 0.2 M solution. What is the molarity of the dilluted solution ?

  • 2)

    Which one of the following gases has the lowest value of the Henry's law constant ?

  • 3)

    The system that forms maximum boiling azeotrope is

  • 4)

    If an aqueous solution of glucose is allowed to freeze, then crystals of which will be separated out first ?

  • 5)

    If sodium sulphate is considered to be completely dissociated into cations and anions in aqueous solution, the change in freezing point of water \((\Delta { T }_{ f })\), when 0.01 mol of sodium sulphate is dissolved in 1 kg of water, is (Kf = 1.86 K kg mol-1).

12th Standard CBSE Chemistry - The Solid State Model Question Paper - by Archana Singh - Deoghar - View & Read

  • 1)

    Wax is an example of

  • 2)

    In a compound, atoms of element Y form ccp lattice and those of element X occupy 2/3rd of tetrahedral voids. The formula of the compound will be

  • 3)

    In a solid lattice, the cation has left a lattice site and is located at an interstitial position. The lattice defect is

  • 4)

    In a face-centered cubic lattice, atom A occupies the corner positions and atom B occupies the face centre positions.If one atom of B is missing from one of the face centred points,the formula of the compound is

  • 5)

    In a solid 'AB' having NaCl structure, 'A' atoms occupy the corners of the cubic unit cell.If all the face-centred atoms along one of the axes are removed,then the resultant stoichiometry of the solid is

CBSE 12th Chemistry - Full Syllabus One Mark Question Paper with Answer Key - by Archana Singh - Deoghar - View & Read

  • 1)

    The property of crystalline solid is not

  • 2)

    The crystal with metal deficiency defect is

  • 3)

    Which of the following is not a characteristic of a crystalline solid?

  • 4)

    Which of the following is true about the value of refractive index of Quartz glass?

  • 5)

    KCl crystallises in the same type of lattice as does NaCl.Given that rNa+/rCl-=0.55 and rK+/rCl-=0.74.Calculate the side of the unit cell of KCl to that of NaCl

CBSE 12th Chemistry - Full Syllabus Five Marks Questions - by Archana Singh - Deoghar - View & Read

  • 1)

    Compounds having general molecular formula  \(AFe_{ 2 }O_{ 4 }\) are called ferrites and posses spinel type structures. Some common examples are \(MgFe_{ 2 }O_{ 4 }\)  and \(ZnFe_{ 2 }O_{ 4 }\) They may be thought of being formed by replacing \(Fe^{ 2+ }\) ions present in \(Fe_{ 3 }O_{ 4 }\) by bivalent cations such as \(Mg^{ 2+ },Zn^{ 2+ }\) ions etc. Now answer the following questions :
    (i) What types of materials are ferrites?
    (ii) What are the main uses of ferrites?

  • 2)

    (a)  Differentiate between molarity and molality for a solution. How does a change in temperature influence their values?
    (b) Calculate the freezing point of an aqueous solution containing 10.50 g of MgBr2 = 184 g) (K f for water 1.86 K kg mol-1)

  • 3)

    Using the standard electrode potentials, predict if the reaction between the following is feasible:
    (i) Fe3+(aq) and I-(aq)
    (ii) Ag+(aq) and Cu(s)
    (iii) Fe3+(aq) and Br-(aq) 
    (iv) Ag(s) and Fe3+(aq)
    (v) Br2(aq) and Fe2+(aq)

  • 4)

    Mira's house had a fencing of iron rods. Her father suggested getting it painted. But she and her mother thought it was a waste of time and money as iron rods are strong:
    (i) Whose opinion is acceptable according to you?
    (ii) Give two other methods to prevent corrosion.
    (iii) What is the chemical formula of rust?
    (iv) Mention the values shown by Mira's father.

  • 5)

    (i) For the reaction AB, the rate of reaction becomes twenty-seven times when the concentration of A is increased three times. What is the order of the reaction?
    (ii) The activation energy of a reaction is 75.2 kJ mol-1 in the absence of a catalyst and it lowers to 50.14 kJ mol-1 with a catalyst. How many times will the rate of reaction grow in the presence of a catalyst if the reaction proceeds at 25°C?

CBSE 12th Chemistry - Full Syllabus Three Marks Questions - by Archana Singh - Deoghar - View & Read

  • 1)

    Assign reasons for the following:
    (i) Phosphorus doped silicon is a semiconductor.
    (ii) Schottky defect lowers the density of a solid.
    (iii) Some of the very old glass objects appear slightly milky instead of being transparent.

  • 2)

    Silver forms ccp lattice and X-ray studies of its crystals show that the edge length of its unit cell is 408.6 pm. Calculate the density of silver (Atomic mass = 107.9 u)

  • 3)

    Conc. H2SO4 has a density of 1.9 gmL-1 and is 99 % by weight. calculate the molarity of is 99% by weight. Calculate the molarity of H2SO4 (Mol . Wt pf H2SO4 = 98 g mol-1).

  • 4)

    A 5% solution of cane-sugar (m.wt. = 342) is isotonic with 0.877% solution of urea. Find the molecular weight of urea.

  • 5)

    What is a nickel-cadmium cell? State its one merit and one demerit over lead storage cell. Write the overall reaction that occurs during discharging of this cell.

CBSE 12th Chemistry - Full Syllabus Two Marks Questions - by Archana Singh - Deoghar - View & Read

  • 1)

    Niobium crystallise in body-cetred cubic structure. If density is 8.55 g cm-3, calculate atomic radius of niobium using its atomic mass 93 u.

  • 2)

    Calculate the amount of benzoic acid (C6H5COOH) required  for preparing 250 ml of 0.15 M solution in methanol.

  • 3)

    5 g of a compound was dissolved in 100 g of water at 303 K. The vapour pressure of the solution was found to be 4.16 kilopascal. If the vapour pressure of pure water is 4.24 kPa at this temperature, what is the molecular mass of the compound?

  • 4)

    V1 cc of solution having molarity M1 is diluted to have molarity M2. Derive expression (in terms of M1,M2 and V1) for the volume of water required to be added.

CBSE 12th Chemistry - Chemical Kinetics Problems Questions - by Archana Singh - Deoghar - View & Read

  • 1)

    (a) Express clearly what you understand by 'rate expression' and 'rate constant' of a reaction.
    (b) Nitrogen pentoxide decomposes according to the equation
    2N2O5(g) \(\to\) 4 NO2 (g) + O2 (g)
    This first order reaction was allowed to proceed at 40oC and the data given below were collected:

    [N2O5] (M) Time (min)
    0.400 0.00
    0.289 20.00
    0.209 40.00
    0.151 60.00
    0.109 80.00

    (i) Calculate the rate constant for the reaction.  Include units with you answer.
    (ii) Calculate the initial rate of reaction.
    (iii) After how many minutes will [N2O5] be equal to 0.350 M?

  • 2)

    The half time of first order decomposition of nitramide is 2.1 hour at 15oC. NH2NO2(aq) \(\longrightarrow\) N2O(g) + H2O (I) 
    If 6.2 g of MH2NO2 is allowed to decompose, calculate
    (i) time taken for NH2NO2 to decompose 99% and
    (ii) volume of dry N2O produced at this point, measured at STP.

  • 3)

    The time required for 10% completion of a first order reaction at 298 K is equal to that required for its 25% completion at 318 K. If the pre-exponential factor for the reaction is 3.56 \(\times 10 ^{9} s^{-1}\), calculate its rate constant at 318 K and also the energy of activation.

  • 4)

    For the reaction, N2O5(g) = 2 NO2(g) + 0.5 O2 (g), calculate the mole fraction of N2O5 (g) decomposed at a constant volume and temperature, if the initial presure is 600 mm Hg and the pressure at any time is 960 mm Hg. Assume ideal gas behaviour.

  • 5)

    At constant temperature and volume, X decomposes as 2 X (g) \(\longrightarrow\)  3 Y (g) + 2 Z (g). Px is the partial pressure of X.

    Observation No. Time (in minutes) Px (in mm of Hg)
    1 0 800
    2 100 400
    3 200 200

    (i) What is the order of reaction with respect to X?  
    (ii) Find the time for 75% completion of the reaction.
    (iii) Find the total pressure when pressure of X is 700 mm of Hg.

CBSE 12th Chemistry - Electrochemistry Problems Questions - by Archana Singh - Deoghar - View & Read

  • 1)

    The standard electrode potential (\(E°\))for Daniell cell is +1.1 V. Calculate the \(\Delta G°\) for the reaction.
    Zn(s)+Cu2+(aq)\(\rightarrow\)Zn2+(aq)+Cu(s)

  • 2)

    (a) Calculate \(\Delta G°\)for the reaction Mg(s)+Cu2+\(\rightarrow\)Mg2+(aq)+Cu(s)
    Given: \({ E }_{ cell }^{ ° }\)= +2.7V, 1F = 96500 C mol-1
    (b) Name the type of cell which was used in Apollo space programme for providing electrical power.

  • 3)

    Conductivity of \(2.5\times { 10 }^{ -4 }\)M methanoic acid is \(5.25\times { 10 }^{ -5 }S{ cm }^{ -1 }\). Calculate its molar conductivity and degree of dissociation.
    Given: \({ \lambda }^{ ° }\left( { H }^{ + } \right) \) = 349.5 S cm-2mol-1 and \({ \lambda }^{ ° }\left( { HCOO }^{ - } \right) \) = 50.5 S cm2mol-1

CBSE 12th Chemistry - Solution Problems Questions - by Archana Singh - Deoghar - View & Read

  • 1)

    45 g of ethylene glycol (C2H4O2) is mixed with 600 g of water. Calculate
    (i) the freezing point depression and 
    (ii) the freezing point of the solution.

  • 2)

    Calculate the boiling point of solution when 4 g of MgSO4 (M = 120 g mol-1) was dissolved in 100 g of water, assuming MgSO4 undergoes complete ionization. (Kb for water = 0.52 K kg mol-1)

  • 3)

    A 5 percent solution (by mass) of cane-sugar (M.W. 342) is isotonic with 0.877% solution of substance X. Find the molecular weight of X.

  • 4)

    Calculate the boiling point of a 1M aqueous solution (density 1.04 g mL-1) of potassium chloride (Kb for water = 0.52 K kg mol-1, Atomic masses : K = 39 u, CI = 35.5 u). Assume, potassium chloride is completely dissociated in solution.

  • 5)

    At 10\(°\) C, the average osmotic pressure of blood is 8.8 atm. Find the total concentration of the various constituents in the blood. Assuming that the concentration is the same as the molarity, find the freezing point of the solution (Kf for water = 1.86 K kg mol-1)

CBSE 12th Chemistry - The Solid State Problem Questions - by Archana Singh - Deoghar - View & Read

  • 1)

    With the help of a labelled diagram show that there are four octahedral voids per unit cell in cubic close packed structure.

  • 2)

    Show that in cubic packed structure, eight tatrahedral voids are persent per unit cell.

  • 3)

    Lithium borohydride,LiBH4,crystallize in an orthorhombic system with 4 molecules per unit cell.The unit cell dimensions are: a=6.81\(\overset { 0 }{ A } \) ,b=4.43\(\overset{0}{A}\) and c=7.17\(​​\overset{0}{A}\).Calculate the density of the crystal.Take atomic mass of Li=7,B=11 and H=1 a.m.u.

  • 4)

    If silver iodide crystallizes in a zinc blende structure with I-ions forming the lattice,then calculate fraction of the tetrahedral voids occupied by Ag+ ions

  • 5)

    A compound consisting of the monovalent ions A+,B- crystallizes in the body-centred cubic lattice.(i)What is the of the compound? (ii)If one one of A+ions  from the corner is replaced by a monovalent ion C+,what would be the simplest formula of the resulting compound?

12th CBSE Chemistry - Chemistry in Everyday Life Five Marks Model Question Paper - by Sudha George - Kochi - View & Read

  • 1)

    Due to hectic and busy schedule, Mr. Singh started taking junk food in the lunch break and slowly became habitual of eating food irregularly to excel in his field. One day during meeting he felt severe chest pain and fell down. Mr. Khanna, a close friend of Mr. Singh took him to doctor immediately. The doctor diagnosed that Mr. Singh was suffering from acidity and prescribed some medicines. Mr. Khanna advised him to eat home made food and change his lifestyle by doing yoga, meditation and some physical exercise. Mr. Singh followed his friend's advice and after few days he started feeling bener.
    After reading the above passage, answer the following:
    (i) What are the values (at least two) displayed by Mr. Khanna?
    (ii)What are antacids? Give one example.
    (iiii)Would it be advisable to take antacids for a long period of time? Give reason

  • 2)

    Due to hectic and busy schedule, Mr. Angad made his life full of tensions and anxiety. He started taking sleeping pills to overcome the depression without consulting the doctor. Mr. Deepak, a close friend of Mr. Angad, advised him to stop taking sleeping pills and and suggested to change his lifestyle by doing yoga, meditation and some physical exercise. Mr. Angad followed his friend's advice and after few days he started feeling better. After rearing the above passage, answer the following:
    (i) What are the values (at least two) displayed by Mr. Deepak?
    (ii) Why is it not advisable to take sleeping pills without consulting doctor?
    (iii) What are tranquilizers? Give two examples.

  • 3)

    Ramesh went to a department store to purchase groceries, On one of shelves he noticed sugar-free tablets, He decided to buy them for his grandfather who was a diabetic. There was three types of sugar-free tablets, Ramesh decided to buy sucralose which was good for his grandfather's health.
    (i) Name another sugar-free tablet which Ramesh did not buy,
    (ii) Was it right to purchase such medicines without doctor's prescription?  
    (iii)Wha is quality of Ramesh is reflected above?

  • 4)

    Mr. Roy, the principal of one reputed school organized a seminar in which he invited parents and principals to discuss the serious issue of diabetes and depression in students. They all resolved this issue by strictly banning the junk food in schools and to introduce healthy snacks and drinks like soup, lassi, milk etc. in school canteens. They also decided to make compulsory half an hour physical activities for the students in the morning assembly daily. After six months, Mr. Royconducted the health survey in most of the schools and discovered a tremendous improvement in the health of students.
    After reading the above passage, answer the following:
    (i)What are the values (at least two) displayed by Mr.Roy?
    (ii) As a student, how can you spread awareness about this issue?
    (iii) What are tranquillizers? Give an example.
    (iv)Why is use of aspartame limited to cold foods and drinks?

  • 5)

    On the occasion of world health day, Dr. Satpal organized a 'Health Camp' for poor farmers living in a near village. After check up, he was shocked to see the most of the farmers suffered from cancer due to regular exposure to pesticides and many were diabetic. They distributed free medicines to them. Dr. Satpal immediately reported the matter to the National Human Right Commission (NHRC). On the suggestion of NHRC, the government decided to provide medical care, financial assistance, setting up of super-speciality hospitals for treatment and prevention of the deadly disease in the affected villages all over India.
    (i) Write the values shown by :
    (a) Dr. Satpal
    (b) NHRC
    (ii) What type of analgesics are chiefly used for the relief of pains of terminal cancer?
    (iii) Give an example of artificial sweetener that could have been recommended to diabetic patients.

CBSE 12th Standard Chemistry - Polymers Five Marks Model Question Paper - by Sudha George - Kochi - View & Read

  • 1)

    What is nylon?Write an equation for the chemistry involved contain alternate monomers of each type.What is the weight percent of ethylene in this copolymer.

  • 2)

    Study the given passage carefully and answer the questions that follow:
    Shalini studied a chapter on Polymers in school and came across the following paragraph: The durability, strength, low cost, water and chemicals resistance, welding properties, lesser'. energy, fewer atmosphere emissions and light weight are advantages of plastic bags. Shalini is confused as she has been reading in the
    newspaper about the ban on the usage of plastic substances. She further finds that despite the durability, the use of these materials has presented mankind with serious waste disposal problem as these materials do not disintegrate by themselves. In view of this, certain polymers are being developed which are broken down rapidly by micro-organisms. Shalini feels relaxed that such kinds of biomaterials are being developed.
    (i) Name the class of these useful polymers which do not harm the environment.
    (ii) Give anyone example of these polymers and name its monomers.
    (iii) Comment on the qualities of Shalini.

  • 3)

    After the ban on plastic bags, students of a school decided to make people aware of the harmful effects of plastic bags on the environment and Yamuna River. To make the awareness more impactful, they organised a rally by partnering with other schools and distributed paper bags to vegetable vendors, shopkeepers and departmental stores. All the students pledged not to use polythene bags in the future to save the Yamuna River.
    After reading the above passage, answer the following questions:
    (i) What values are shown by the students?
    (ii) What are bio-degradable polymers ? Give one example.
    (iii) Is polyethene a condensation or an addition polymer?

12th CBSE Chemistry - Biomolecules Five Marks Model Question Paper - by Sudha George - Kochi - View & Read

  • 1)

    How do expimers differ from anomers ?

  • 2)

    Give reason for the following :
    (i) On electrolysis in acidic solution, aminoacids migrate towards cathode while in alkaline solution these migrate towards anode.
    (ii) The mononamino monocarboxylic acids have two pK values.

  • 3)

    What forces are responsible for the stability of \(\alpha\) - helix ? Why is it named as 3.613 helix?

  • 4)

    The Ka and Kb values of \(\alpha\) - amino acids are very low. Explain.

  • 5)

    Write the structure of alanine at pH = 2 and pH = 10.

12th Standard CBSE Chemistry - Organic Compounds Containing Nitrogen ( Amines ) Five Marks Model Question Paper - by Sudha George - Kochi - View & Read

  • 1)

    How will you convert toluene into sym-trinitrobenzene?

  • 2)

    Why is an amide more acidic than amine?

  • 3)

    Why does bromination of aniline, even under very mild conditions, gives 2, 4, 6-tribromoaniline instantaneously?

  • 4)

    (a) Write the structures of the main products when aniline reacts with the following reagents:
    (i) Br2 water
    (ii) HCI
    (iii) (CH3CO)2O / pyridine
    (b) Arrange the following in the increasing order of their boiling point:
    C2H5NH2 , C2H5OH, (CH3)3N
    (c) Give a simple chemical test to distinguish between the following pair of compounds:
    (CH3)2NH and (CH3)3N

  • 5)

    (a) Write the structures of main products when products when benzene diazonium chloride reacts with the following reagents:
    (i) H3PO2+H2O
    (ii) CuCN/KCN
    (iii) H2O
    (b) Arrange the following in the increasing order of their basic character in an aqueous solution:
    C2H5NH2 , (C2H5)2NH, (C2H5)3N
    (c) Give a simple chemical test to distinguish between the following pair of compounds:
    C6H5-NH2 and C6H5-NH-CH3

12th Standard CBSE Chemistry - Aldehydes , Ketones and Carboxylic Acids Five Marks Model Question Paper - by Sudha George - Kochi - View & Read

  • 1)

    (a) Describe the following giving linked chemical equations:
    (i) Cannizzaro reaction
    (ii) Decarboxylation
    (b) Complete the following chemical equations:
          
    (iii) \(C_6H_5CONH_2 \xrightarrow [heat] {H_3O^+}\)

  • 2)

    (a) Identify A, B and C in the following sequence of reactions:

    (b) Predict the structures of the products formed when benzaldehyde is treated with
    (i) conc. NaOH
    (ii) HNO3 / H2SO4 (at 273 - 383 k)

  • 3)

    An organic compound (A) on treatment with ethyl alcohol gives a carboxylic acid (B) and compound (C).  Hydrolysis of (C) under acidified conditions gives (B) and (D).  Oxidation of (D) with KMnO4 also gives (B).  (B) on healting with Ca(OH)2 gives (E) having molecular formula C3H6O.  (E) does not give Tollen's test and does not reduce Fehling's solution but forms a 2, 4-dinitrophenylhydrazone.  Identify (A), (B), (C), (D) and (E).

12th Standard CBSE Chemistry - Alcohols , Phenols and Ethers Five Marks Model Question Paper - by Sudha George - Kochi - View & Read

  • 1)

    PCl5 reacts with ethanol to form chloromethane. However, with phenol, it does not give chlorobenzene but gives triphenylphosphate. Explain.

  • 2)

    Explain why alcohols do not react with NaBr but H2SO4 is added they form alkyl bromides.

  • 3)

    Although both allyl alcohol and 1-propanol are primary alcohols,they can still be distinguished by Lucas reagent. Explain how?

  • 4)

    2,2-Dimethyloxirane can be cleaved by acid (H+).Write its mechanism.

  • 5)

    Miss Usha was asked to synthesize alcohol by acidic hydration of l-butene. She was unaware of the fact that the vessel she used had some coating of a  metal, and in addition to alcohol, compound X was I isolated X forms bisulphate compound as well as 2, 4-dinitrophenylhydrazone. Separation of alcohol could be made by physical as well as by chemical methods.
    (i) How are alcohol and Xformed? 
    (ii) Can alcohol and Xgive iodoform test? 
    (iii) Give different methods of separation? 
    (iv) What value did Miss. Usha lack?

12th Standard CBSE Chemistry - Alcohols , Phenols and Ethers Five Marks Questions - by Archana Singh - Deoghar - View & Read

  • 1)

    PCl5 reacts with ethanol to form chloromethane. However, with phenol, it does not give chlorobenzene but gives triphenylphosphate. Explain.

  • 2)

    Explain why alcohols do not react with NaBr but H2SO4 is added they form alkyl bromides.

  • 3)

    Although both allyl alcohol and 1-propanol are primary alcohols,they can still be distinguished by Lucas reagent. Explain how?

  • 4)

    Gasoline is being used worldwide for running automobile, aeroplane etc. Due to the presence of nitrogen and sulphuric compounds in gasoline, the exhaust gases contain oxides of nitrogen and sulphur which are major pollutants in the environment in contrast, ethanol is a much cleaner fuel because it produces only CO2 and Hp. Now answer the following questions:
    (a) Besides being a cleaner fuel, what are the other advantages of using ethanol as a fuel in automobile?  
    (b) What are the disadvantages of using ethanol as a fuel? What are the value associated with its use?

  • 5)

    Miss Usha was asked to synthesize alcohol by acidic hydration of l-butene. She was unaware of the fact that the vessel she used had some coating of a  metal, and in addition to alcohol, compound X was I isolated X forms bisulphate compound as well as 2, 4-dinitrophenylhydrazone. Separation of alcohol could be made by physical as well as by chemical methods.
    (i) How are alcohol and Xformed? 
    (ii) Can alcohol and Xgive iodoform test? 
    (iii) Give different methods of separation? 
    (iv) What value did Miss. Usha lack?

12th Standard CBSE Chemistry - Organic Compounds Containing Nitrogen Five Marks Questions - by Archana Singh - Deoghar - View & Read

  • 1)

    Illustrate the following reactions giving a chemical equation in each case:
    (i) Gabriel phthalimide synthesis
    (ii) A coupling reaction.
    (iii) Hoffmann's bromamide reaction.

  • 2)

    Give one chemical test each to distinguish between the compounds in the following pairs:
    (i) Methyleamine and dimethylamine
    (ii) Aniline and benzylamine
    (iii) Ethylamine and aniline

  • 3)

    How would you achieve the following conversions?
    (i) Nitrobenzene to aniline
    (ii) An alkyl halide to a quaternary ammonium salt.
    (iii) Aniline to benzonitrile.
    Write the chemical equation with reaction conditions in each case.

  • 4)

    A compound 'X' having molecular formula C3H7NO, reacts with Br2 in presence of KOH to give another compound 'Y'. The compound Y reacts with NHO2 to form ethanol and N2 gas. Identify the compounds X and Y and write the reactions involved.

  • 5)

    An aliphatic compound 'A' with molecular formula C2H3Cl on treatment with AgCN hives two isomeric compounds of unequal amounts with the molecular formula C3H3N. The minor of nthese two products on complete reduction with Hin the presence of Ni gives a compound 'B' with  molecular formula C3H9N. Identify the compounds 'A' , 'B' and write the reactions involved.

CBSE 12th Chemistry - Biomolecules Five Marks Questions - by Archana Singh - Deoghar - View & Read

  • 1)

    How do expimers differ from anomers ?

  • 2)

    Glucose and fructose give the same osazone ? Explain.

  • 3)

    Give reason for the following :
    (i) On electrolysis in acidic solution, aminoacids migrate towards cathode while in alkaline solution these migrate towards anode.
    (ii) The mononamino monocarboxylic acids have two pK values.

  • 4)

    What forces are responsible for the stability of \(\alpha\) - helix ? Why is it named as 3.613 helix?

  • 5)

    The Ka and Kb values of \(\alpha\) - amino acids are very low. Explain.

CBSE 12th Chemistry - Chemistry in Everyday Life Five Marks Questions - by Archana Singh - Deoghar - View & Read

  • 1)

    Due to hectic and busy schedule, Mr. Singh started taking junk food in the lunch break and slowly became habitual of eating food irregularly to excel in his field. One day during meeting he felt severe chest pain and fell down. Mr. Khanna, a close friend of Mr. Singh took him to doctor immediately. The doctor diagnosed that Mr. Singh was suffering from acidity and prescribed some medicines. Mr. Khanna advised him to eat home made food and change his lifestyle by doing yoga, meditation and some physical exercise. Mr. Singh followed his friend's advice and after few days he started feeling bener.
    After reading the above passage, answer the following:
    (i) What are the values (at least two) displayed by Mr. Khanna?
    (ii)What are antacids? Give one example.
    (iiii)Would it be advisable to take antacids for a long period of time? Give reason

  • 2)

    Due to hectic and busy schedule, Mr. Angad made his life full of tensions and anxiety. He started taking sleeping pills to overcome the depression without consulting the doctor. Mr. Deepak, a close friend of Mr. Angad, advised him to stop taking sleeping pills and and suggested to change his lifestyle by doing yoga, meditation and some physical exercise. Mr. Angad followed his friend's advice and after few days he started feeling better. After rearing the above passage, answer the following:
    (i) What are the values (at least two) displayed by Mr. Deepak?
    (ii) Why is it not advisable to take sleeping pills without consulting doctor?
    (iii) What are tranquilizers? Give two examples.

  • 3)

    Ramesh went to a department store to purchase groceries, On one of shelves he noticed sugar-free tablets, He decided to buy them for his grandfather who was a diabetic. There was three types of sugar-free tablets, Ramesh decided to buy sucralose which was good for his grandfather's health.
    (i) Name another sugar-free tablet which Ramesh did not buy,
    (ii) Was it right to purchase such medicines without doctor's prescription?  
    (iii)Wha is quality of Ramesh is reflected above?

  • 4)

    Mr. Roy, the principal of one reputed school organized a seminar in which he invited parents and principals to discuss the serious issue of diabetes and depression in students. They all resolved this issue by strictly banning the junk food in schools and to introduce healthy snacks and drinks like soup, lassi, milk etc. in school canteens. They also decided to make compulsory half an hour physical activities for the students in the morning assembly daily. After six months, Mr. Royconducted the health survey in most of the schools and discovered a tremendous improvement in the health of students.
    After reading the above passage, answer the following:
    (i)What are the values (at least two) displayed by Mr.Roy?
    (ii) As a student, how can you spread awareness about this issue?
    (iii) What are tranquillizers? Give an example.
    (iv)Why is use of aspartame limited to cold foods and drinks?

  • 5)

    On the occasion of world health day, Dr. Satpal organized a 'Health Camp' for poor farmers living in a near village. After check up, he was shocked to see the most of the farmers suffered from cancer due to regular exposure to pesticides and many were diabetic. They distributed free medicines to them. Dr. Satpal immediately reported the matter to the National Human Right Commission (NHRC). On the suggestion of NHRC, the government decided to provide medical care, financial assistance, setting up of super-speciality hospitals for treatment and prevention of the deadly disease in the affected villages all over India.
    (i) Write the values shown by :
    (a) Dr. Satpal
    (b) NHRC
    (ii) What type of analgesics are chiefly used for the relief of pains of terminal cancer?
    (iii) Give an example of artificial sweetener that could have been recommended to diabetic patients.

CBSE 12th Chemistry - Aldehydes , Ketones and Carboxylic Acids Five Marks Questions - by Archana Singh - Deoghar - View & Read

  • 1)

    (a) How will you prepare the following compounds starting with benzene:
    (i) Benzaldehyde
    (ii) Acetophenone
    (b) Give chemical tests to distinguish between the following pairs of compounds
    (i) Ethanal and Propanal
    (ii) Benzaldehyde and Acetophenone
    (iii) Phenol and Benzoic acid

  • 2)

    (a) Explain the mechanism of nucleophilic attack on a carbonyl group of an aldehyde or a ketone.
    (b) How would you bring about the following conversions:
    (i) Propanone to Propene
    (ii) Ethanol to 3-hydroxybutanal
    (iii) Benzaldehyde to Benzophenone

  • 3)

    (a) Write chemical equations to illustrate the following name bearing reactions:
    (i) Cannizzaro's reaction
    (ii) Hell-Volhard-Zelinsky reaction
    (b) Give chemical tests to distinguish between the following pairs of compounds
    (i) Propanal and Propanone
    (ii) Acetophenone and Benzophenone
    (iii) Phenol and Benzoic acid

CBSE 12th Chemistry Polymers Five Marks Questions - by Archana Singh - Deoghar - View & Read

  • 1)

    What is nylon?Write an equation for the chemistry involved contain alternate monomers of each type.What is the weight percent of ethylene in this copolymer.

  • 2)

    Study the given passage carefully and answer the questions that follow:
    Shalini studied a chapter on Polymers in school and came across the following paragraph: The durability, strength, low cost, water and chemicals resistance, welding properties, lesser'. energy, fewer atmosphere emissions and light weight are advantages of plastic bags. Shalini is confused as she has been reading in the
    newspaper about the ban on the usage of plastic substances. She further finds that despite the durability, the use of these materials has presented mankind with serious waste disposal problem as these materials do not disintegrate by themselves. In view of this, certain polymers are being developed which are broken down rapidly by micro-organisms. Shalini feels relaxed that such kinds of biomaterials are being developed.
    (i) Name the class of these useful polymers which do not harm the environment.
    (ii) Give anyone example of these polymers and name its monomers.
    (iii) Comment on the qualities of Shalini.

  • 3)

    After the ban on plastic bags, students of a school decided to make people aware of the harmful effects of plastic bags on the environment and Yamuna River. To make the awareness more impactful, they organised a rally by partnering with other schools and distributed paper bags to vegetable vendors, shopkeepers and departmental stores. All the students pledged not to use polythene bags in the future to save the Yamuna River.
    After reading the above passage, answer the following questions:
    (i) What values are shown by the students?
    (ii) What are bio-degradable polymers ? Give one example.
    (iii) Is polyethene a condensation or an addition polymer?

12th Standard CBSE Chemistry - Haloalkanes and Haloarenes Five Marks Questions - by Archana Singh - Deoghar - View & Read

  • 1)

    Explain why alkyl halides are generally not prepared in the laboratory by free radical halogenation of alkanes.

  • 2)

    Explain why chorination of n-butane in presence of light at 298 K gives a mixture of 72% of 2-chlorobutane and 28 % of 1-chlorobutane.

  • 3)

    Wurtz reaction falils in case of tert-alkyl halides. Explain.

  • 4)

    Explain the following in one or two sentences
    (i) Displacement of cyanic and amide ion is never observed in nucleophilic substitution reactions.
    (ii) RCI is hydrolysed to ROH slowly but the reaction is rapid if a catalytic amount of KI is added to the reaction mixture.

  • 5)

    (R)-2- Bromooctane reacts with NaSH to form (S)-2- octanethiol with inversion of configuration at the stereocentre.How can we obtain (R)-2 octanethiol from (R)-2-bromoctane?

12th Standard CBSE Chemistry - Coordination Compounds Five Marks Questions - by Archana Singh - Deoghar - View & Read

  • 1)

    Using crystal field theory, draw energy level diagram, write electronic configuration of the central metal atom/ion and determine the magnetic moment value in the following :
    (i) \([CoF_6]^{3-}, [Co(H_2O)_6]^{2+},[Co(CN)_6]^{3-}\) 
    (ii) \([FeF_6]^{3-}, \ [Fe(H_2O)_6]^{2+}, [Fe(CN)_6]^{4-}\).

  • 2)

    Using valence bond theory, explain the following in relation to the complexes given below :
    \([Mn(CN)_6]^{3-}, [Co(NH_3)_6]^{3+}, [Cr(H_2O)_6]^{3+}, [FeCl_6]^{4-}\)
    (i) Type of hybridisation    
    (ii) Inner or outer orbital complex.    
    (iii) Magnetic behaviour.    
    (iv) Spin only magnetic moment value.

  • 3)

    CoSO4Cl.5 NHexists in two isomeric forms 'A' and 'B' gives white precipitate with BaClbut does not react with AgNO3.  Answer the following questions.
    (i) Identify 'A' and 'B' and write their structure formulas.    
    (ii) Name the type of isomerism involved    
    (iii) Give the IUPAC name of 'A' and 'B'.

12th Standard CBSE Chemistry - The d- and f- Block Elements Five Marks Questions - by Archana Singh - Deoghar - View & Read

  • 1)

    (a) Given below are the electrode potential values, Eo for the some of the first row of transition elements:

    Element EoM2+/M (V)

    V(23) 

    Cr(24)

    Mn(25)

    Fe(26)

    Co(27)

    Ni(28)

    Cu(29)

    -1.18

    -0.91

    -1.18

    -0.44

    -0.28

    -0.25

    +0.34

    Explain the irregularities in these values on the basis of electronic structures of atoms.
    (b) Complete the following reaction equations:
    (i) Cr2O72-+Sn2++H+\(\longrightarrow \) 
    (ii) MnO4-+Fe2++H+\(\longrightarrow \)

  • 2)

    Calculate the number of unpaired electrons in following gaseous ions: Mn3+, Cr3+, V3+ and Ti3+. Which one of these is the most stable in aqueous solution?

  • 3)

    Give examples and suggest reasons for the following features of the transition metal chemistry:
    (i) The lowest oxide of transition metal is basic, the highest is amphoteric/acidic.
    (ii) A transition metal exhibits highest oxidation state in oxides and fluorides.
    (iii) The highest oxidation state is exhibited in oxoanions of a metal.

  • 4)

    Write down the number of 3d electrons in each of the following ions: Ti2+, V2+, Cr3+, Mn2+, Fe2+, Fe3+, Co2+, Ni2+ and Cu2+. Indicate how would you expect the five 3d orbitals to be occupied for these hydrated ions (octahedral).

  • 5)

    On the basis of Lanthanoid contraction, explain the following:
    (i) Nature of bonding in La2O3 and Lu2O3.
    (ii) Trends in the stability of oxo salts of lanthanoids from LA to Lu.
    (iii) Stability of the comlexes of lanthanoids.
    (iv) Radii of 4d and 5d block elements.
    (v) trends in acidic character of lanthanoid oxides.

CBSE 12th Chemistry - The p-Block Elements Five Marks Questions - by Archana Singh - Deoghar - View & Read

  • 1)

    (a) Draw the structures of the following:
    (i) N2O5
    (ii) XeOF4
    (b) Explain the following observations:
    (i) The electron gain enthalpy of sulphur atom has a greater negative value than that of oxygen atom.
    (ii) Nitrogen does not form pentahalides.
    (iii) In aqueous solutions, HI is a stronger acid than HCl.

  • 2)

    (a) Draw the structures of the following:
    (i) XeF4     (ii) H2S2O7
    (b) Explain the following observations:
    (i) Phosphorus has a greater tendency for catenation than nitrogen.
    (ii) The negative value of electron gain enthalpy is less for fluorine than that for chlorine.
    (iii) Hydrogen fluoride has a much higher boiling point than hydrogen chloride.

  • 3)

    (a) Explain the following:
    (i) NF3 is an exothermic compound whereas NCl3 is not.
    (ii) F2 is most reactive of all the four common halogens.
    (b) Complete the following chemical equations:
    (i) c + H2SO4 (conc.) \(\longrightarrow \)
    (ii) P4 + NaOH + H2\(\longrightarrow \)
    (iii) Cl2 \(\to\) \(\underset{excess}{F_2}\) \(\longrightarrow \)

    \(\)

  • 4)

    On heating, lead (II) nitrate gives a brown gas 'A'. The gas 'A' on cooling changes to colourless solid 'B'. Solid 'B' on heating with NO changes to a blue solid 'C'. Identiy 'A', 'B' and 'C' and also write reactions involved and draw the structures of 'B' and 'C'.

  • 5)

    (a) Assign a reason for each of the following statements:
    (i) All the bonds in \(PCI_{ 5 }\)are not equal in length.
    (b) Write the structural formulae of the following compounds:
    \((i)BrF_{ 3 }\quad (ii)XeF_{ 2 }\)  

12th Standard CBSE Chemistry - General Principles and Processes of Isolation of Elements Five Marks Questions - by Archana Singh - Deoghar - View & Read

  • 1)

    Explain the following
    i. Carbon reduction process is not applied for reducing aluminium oxide to aluminium.
    ii. Aq Sodium chloride cannot be used for the isolation of sodium by electrolytic reduction method.
    iii. Thermite process is quite useful for repairing the broken parts of machines.

  • 2)

    Why is the reduction of a metal oxide easier if the metal formed is in liquid state at the temperature of reduction?

  • 3)

    While washing a miner's overalls, a washer woman noticed that sand and similar dirt fell to the bottom of the wash tub. What was peculiar, the copper bearing compounds that had come to the clothes from the mines were caught in the soaplids and so they came to the top. One of her clients was a chemist, Mrs. Malik. The washer woman told her experience to Mrs. Malik. The latter thought that the idea could be used for separating copper compounds from rocky and earth materials on a large scale. After reading the above passage answer the following questions-
    (i) Name the process of concentration of ore based on the observation of washer woman.
    (ii) What type of ores are concentrated by this process?
    (iii) What values are shown by washer woman and Mrs.Malik?

  • 4)

    While washing a miner's overalls, a washerwoman noticed that sand and similar dirt fell to the bottom of the washtub. What was peculiar, the copper-bearing compounds that had come to the clothes from the mines were caught in the soaplids and so they came to the top. One of her clients was a chemist, Mrs. Malik. The washerwoman told her experience to Mrs. Malik. The latter thought that the idea could be used for separating copper compounds from rocky and earth materials on a large scale. After reading the above passage answer the following questions-
    What type of ores are concentrated by this process?

  • 5)

    While washing a miner's overalls, a washer woman noticed that sand and similar dirt fell to the bottom of the wash tub. What was peculiar, the copper bearing compounds that had come to the clothes from the mines were caught in the soaplids and so they came to the top. One of her clients was a chemist, Mrs. Malik. The washer woman told her experience to Mrs. Malik. The latter thought that the idea could be used for separating copper compounds from rocky and earth materials on a large scale. After reading the above passage answer the following questions-
    What values are shown by washer woman and Mrs.Malik?

12th Standard CBSE Chemistry - Surface Chemistry Five Marks Questions - by Archana Singh - Deoghar - View & Read

  • 1)

    What are the applications of adsorption in chemical analysis?

  • 2)

    A house wife while working in the kitchen gots a cut on the finger. It started bleeding and become panicked. She immediately called her neighbour.She had kept ferric chloride in her house. She immediately applied it on the affected area and the bleeding stopped.
    (i) What is the chemical formula of ferric chloride?
    (ii) Why did bleeding stop on applying it on the affected finger?
    (iii) What is the name of the phenomenon involved?
    (iv) What do we learn from it? What is the value associated with this from the point of view of chemist?

  • 3)

    Radha and Meera are fast friends. They both study in class XII.Radha is a student of science while Meera follows commerce stream. One day, when they were playing, Radha got hurt and started bleeding. Meera took her to house and applied some alum on the cut. The bleeding stopped immediately.
    (i)Why bleeding stopped when some alum is applied on the cut?
    (ii) What are the values associated with Meera's decision to take Radha to her house and apply alum on her cut?

  • 4)

    Reetu is purely vegetarian but her husband Rohit likes non-vegetarian food. He regularly buys nonvegetarian goods (fish, meet, eggs etc.) and store these in fridge. The intermixing of odours creates problem for Reetu. Their daughter Rihana is a student of science. One day Rihana suggested her mother to put some small pieces of charcoal in the fridge as she thinks that this will solve the problem to some extent.
    (i) Do you think by putting some small pieces of charcoal in the fridge will solve the problem of intermixing of odours. Give reason for your answer.
    (ii) What are the values associated with Rihana's suggestion?

  • 5)

    On visit to your native village, you find that lot of smoke, dust and some other gases are coming out of the chimney of a recently set up factory near the village.
    (i) As a chemistry student, what method you will suggest the factory owner to manage dust and smoke?
    (ii) What is the basic principle involved in your suggestion?
    (iii) What values are associated with your advice?

12th Standard CBSE Chemistry - Chemical Kinetics Five Marks Questions - by Archana Singh - Deoghar - View & Read

  • 1)

    (a) Define the following:
    (i) Order of a reaction
    (ii) Elementary step in a reaction
    (b) A first order reaction has a rate constant value of 0.00510 min-1.  If we begin with 0.10 M concentration of the reactant, how much of the reactant will remain after 3.0 hours?

  • 2)

    (a) Explain the following terms:
    (i) Rate of a reaction
    (ii) Activation energy of a reaction
    (b) The decomposition of phosphine (PH3) proceeds according to the following equation:
    4PH3(g) \(\to\)P4 (g) + 6H2 (g)
    It is found that the reaction follows the following rate equation:
    Rate = k [PH3]
    The half-life of PH3 is 37.9 s at 120 oC.
    (i) How much time is required for 3/4th of PH3 to decompose?
    (ii) What fraction of the original sample of PH3 remains behind after 1 minutes?

  • 3)

    (a) Explain the following terms:
    (i) Order of a reaction
    (ii) Molecularity of a reaction
    (b) The rate of a reaction increases four times when the temperature changes from 300 k to  320 K.  Calculate the energy of activation of the reaction, assuming that it does not change with temperature.
    (R = 8.314 J K-1 mol-1)

  • 4)

    What is Arrhenius equation to describe the effect of temperature on rate of a reaction? How can it be used to calculate the activation energy of a reaction?

  • 5)

    Describe briefly the dependence of reaction rate of a chemical reaction on temperature. Explain the effect of temperature on the rate constant of a reaction.

12th Standard CBSE Chemistry - Haloalkanes and Haloarenes Five Mark Model Question Paper - by Sudha George - Kochi - View & Read

  • 1)

    Explain why alkyl halides are generally not prepared in the laboratory by free radical halogenation of alkanes.

  • 2)

    Explain why chorination of n-butane in presence of light at 298 K gives a mixture of 72% of 2-chlorobutane and 28 % of 1-chlorobutane.

  • 3)

    Wurtz reaction falils in case of tert-alkyl halides. Explain.

  • 4)

    Explain the following in one or two sentences
    (i) Displacement of cyanic and amide ion is never observed in nucleophilic substitution reactions.
    (ii) RCI is hydrolysed to ROH slowly but the reaction is rapid if a catalytic amount of KI is added to the reaction mixture.

  • 5)

    (R)-2- Bromooctane reacts with NaSH to form (S)-2- octanethiol with inversion of configuration at the stereocentre.How can we obtain (R)-2 octanethiol from (R)-2-bromoctane?

CBSE 12th Standard Chemistry - Coordination Compounds Five Mark Model Question Paper - by Sudha George - Kochi - View & Read

  • 1)

    Using crystal field theory, draw energy level diagram, write electronic configuration of the central metal atom/ion and determine the magnetic moment value in the following :
    (i) \([CoF_6]^{3-}, [Co(H_2O)_6]^{2+},[Co(CN)_6]^{3-}\) 
    (ii) \([FeF_6]^{3-}, \ [Fe(H_2O)_6]^{2+}, [Fe(CN)_6]^{4-}\).

  • 2)

    Using valence bond theory, explain the following in relation to the complexes given below :
    \([Mn(CN)_6]^{3-}, [Co(NH_3)_6]^{3+}, [Cr(H_2O)_6]^{3+}, [FeCl_6]^{4-}\)
    (i) Type of hybridisation    
    (ii) Inner or outer orbital complex.    
    (iii) Magnetic behaviour.    
    (iv) Spin only magnetic moment value.

  • 3)

    CoSO4Cl.5 NHexists in two isomeric forms 'A' and 'B' gives white precipitate with BaClbut does not react with AgNO3.  Answer the following questions.
    (i) Identify 'A' and 'B' and write their structure formulas.    
    (ii) Name the type of isomerism involved    
    (iii) Give the IUPAC name of 'A' and 'B'.

  • 4)

    What is the relationship between observed colour of the complex and the wavelength of light absorbed by the complex ?

  • 5)

    Why are different colours observed in octahedral and tetrahedral complexes for the same metal and same ligands ?

CBSE 12th Standard Chemistry - The d- and f- Block Elements Five Mark Model Question Paper - by Sudha George - Kochi - View & Read

  • 1)

    (a) Given below the following characteristics of the first series of the transition metals and their trends in the series (Sc to Zn):
    (i) Atomic radii
    (ii) Oxidation status
    (iii) Ionisation enthalpies
    (b) Name an important alloy which contains some of the lanthanoid metals. Mention its two uses.

  • 2)

    Calculate the number of unpaired electrons in following gaseous ions: Mn3+, Cr3+, V3+ and Ti3+. Which one of these is the most stable in aqueous solution?

  • 3)

    Compare the general characteristics of the first series of the transition metals with those of the second and third columns. Give special emphasis on the following points:
    (i) electronic configurations,
    (ii) oxidation states,
    (iii) ionisation enthalpies and 
    (iv) atomic sizes.

  • 4)

    (a) Out of Ag2SO4, CuF2, MgF2 and CuCI, which compound will be coloured and why?
    (b) Explain :
    (i) \({ CrO }_{ 4 }^{ 2- }\) is a strong oxidizing agent while \({ MnO }_{ 4 }^{ 2- }\) is not.
    (ii) Zr and Hf have identical sizes.
    (iii) The lowest oxidation state of manganese is basic while the highest is acidic.
    (iv) Mn (II) shows maximum paramagnetic character amongst the divalent ions of the first transition series.

  • 5)

    Complete the following chemical reaction equations:
    (i) \(CrO_{ 7 }^{ 2\quad - }(aq)+H_{ 2 }S(g)+H^{ + }(aq)\longrightarrow \) 
    (ii) \(CrO_{ 7 }^{ 2\quad - }(aq)+H_{ 2 }S(g)+H^{ + }(aq)\longrightarrow \)
    (b) Explain the following observations:
    (i) Transition metals form compounds which are usually coloured.
    (ii) Transition metals exhibit variable oxidation states.
    (iii) The actinoids exhibit a greater range of oxidation states than the lanthanoids.

12th Standard CBSE Chemistry - The p-Block Elements Five Mark Question Paper - by Sudha George - Kochi - View & Read

  • 1)

    (a) Complete the following chemical equations:
    (i) Cu + HNO3(dilute) \(\rightarrow\)
    (ii) XeF4 + O2F2 \(\rightarrow\)
    (b) Explain the following observations:
    (i) Phosphorus has greater tendency for catenation than nitrogen.
    (ii) Oxygen is a gas but sulphur a solid.
    (iii) The halogens are coloured.  Why?

  • 2)

    (a) Draw the structures of the following molecules:
    (i) (HPO3)3
    (ii) BrF3
    (b) Complete the following chemical equations:
    (i) HgCl2 + PH3 \(\rightarrow\)
    (ii) SO3 + H2SO\(\rightarrow\)
    (iii) XeF4 + H2\(\rightarrow\)

  • 3)

    (a) Draw the structures of the following:
    (i) N2O5
    (ii) XeOF4
    (b) Explain the following observations:
    (i) The electron gain enthalpy of sulphur atom has a greater negative value than that of oxygen atom.
    (ii) Nitrogen does not form pentahalides.
    (iii) In aqueous solutions, HI is a stronger acid than HCl.

  • 4)

    (a) Draw the structures of the following:
    (i) XeF4     (ii) H2S2O7
    (b) Explain the following observations:
    (i) Phosphorus has a greater tendency for catenation than nitrogen.
    (ii) The negative value of electron gain enthalpy is less for fluorine than that for chlorine.
    (iii) Hydrogen fluoride has a much higher boiling point than hydrogen chloride.

  • 5)

    (a) Complete the following chemical equations:
    (i) F2 + H2        \(\to\) .............
    (ii) Ca3P2 + H2O   \(\to\) .............
    (iii) XeF4 + H2O    \(\to\) .............
    (b) Draw structures of the following species:
    (i) H2S2O7 (ii) NO3-

12th Standard CBSE Chemistry - Electrochemistry Five Marks Questions - by Archana Singh - Deoghar - View & Read

  • 1)

    On the basis of the following data, explain why Co(III) is not stable in aqueous solution ?
    \({ Co }^{ 3+ }+{ e }^{ - }\longrightarrow { Co }^{ 2+ },{ E }^{ 0 }=+1.82V\)
    \(2{ H }_{ 2 }O\longrightarrow { O }_{ 2 }+4{ H }^{ + }+{ 4e }^{ - },{ E }^{ 0 }=1.23V\)

  • 2)

    A voltaic cell is set up at \(25°\)C with the half cells Ag+(0.001 M)Ag and Cu2+(0.10M)Cu. What should be its cell potential?

  • 3)

    India is a very big country and is in a phase of modernisation. There is an enormous shortage of electricity and as a result, our industrial growth is hampered. There are major cuts both at household and domestic levels. These days, there is a major emphasis on the use of solar energy.
    (i) How is the solar power used in a country like India?
    (ii) As a student, how will you promote the use of solar power?

  • 4)

    Calculate the \(\triangle G^{ 0 }\)   and emf (E) that can be obtained from the following cell under the standard conditions at 25°C. Zn(s) IZn2+(aq) IISn2+(aq) I Sn(s) [Given \(E^{ 0 }_{ Zn2+/Zn }=-0.76V;\) \(E^{ 0 }_{ sin2+/sn }=-0.14V\) ] and F = 96500 C mol-1

  • 5)

    (i) The conductivity of 0.001 mol L-1 solution of CH3COOH is 3.905 x 10-5 S cm-1. Calculate its molar conductivity and degree of dissociation. Given \(\lambda ^{ 0 }\left( H^{ + } \right) =349.6 \ Scm^{ 2 } \ mol^{ -1 }\) and \(\lambda ^{ 0 }\left( CH_{ 3 }COO^{ - } \right) =40.9 \ scm^{ 2 }mol^{ -1 }\)
    (ii) Define electrochemical cell. What happens if external potential applied becomes greater than E0cell of electrochemical cell?

12th Standard CBSE Chemistry - Solution Five Marks Questions - by Archana Singh - Deoghar - View & Read

  • 1)

    (a) State the following:
    (i) Henry's law about partial pressure of gas in a mixture.
    (ii) Raoult's law in its general form in reference to solutions.
    (b) A solution prepared by dissolving 8.95 mg of a gene fragment in 35.0 ML of water has an osmotic pressure of 0.335 torr at 25oC. Assuming gene fragment is non-electrolyte, determine its molar mass.

  • 2)

    (a) Assuming complete ionization, calculate the expected freezing point of solution prepared by dissolving 6.00 g of Glauber's salt, Na2SO4. 10H2O in 0.1 kg of H2O. (K f for H2O = 1.86 K kg mol-1) [At. mass of Na = 23, S = 32, O= 16, H = 1 u].
    (b) Two liquids X and Y boil at 110 oC and 130 oC respectively. Which of them has higher vapor pressure at 50oC ?

  • 3)

    (a) Define Azeotropes and explain broefly minimum boiling azeotrope by taking suitable example.
    (b) The vapour pressures of pure liquids A and B are 450 mm 700 mm of Hg respectively at 350 K. Calculate the composition of the liquid mixture if total vapour pressure is 600 mm of Hg. Also find the composition of the mixture in the vapour phase.

  • 4)

    The elements A and B form purely covalent compounds having molecular formulae AB2 and AB4. When dissolved in 20g of benzene, 1g of AB2 lowers the freezing point by 2.3K, whereas 1g of AB4 lowers it by 1.3K. the molal depression constant for benzene is 5.1 K kg/mol, calculate the atomic mass of A and atomic mass of B. (A=25.59, B=42.64).

  • 5)

    a) Why constant boiling mixtures behave like a single component when subjected to distillation.
    b) What type of Azeotropic mixtures are formed by the following solution
    i) H2O and HCl ii) H2O and C2H5OH
    c) Give one practical application of depression of freezing point?
    d) A Solid solution is formed between two substances. One whose particles are very large and the other particles are very small. What type of solid solution is this likely to be?
    e) Write the Raoults Law for each component of a binary solution and show that the total vapour pressure of the solution may be expressed as P = P0A + (P0B – P0A) XB.

12th Standard CBSE Chemistry - The Solid State Five Marks Questions - by Archana Singh - Deoghar - View & Read

  • 1)

    What is meant by anisotropy? Which of the two : quartz or glass shows this property?

  • 2)

    Define the following terms:
    (i) ferromagnetism
    (ii) Ferrimagnetism

  • 3)

    Define the following:
    (i) Dislocation
    (ii) F-centre
    (iii) Anti-ferroelectric

  • 4)

    What type of structure is generally adopted by alkali metal halides at ordinary temperature and pressure? What happens to structure when pressure is applied to alkali metal halides?

  • 5)

    What are point defects in crystals and how do they develop?

CBSE Class 12th Chemistry - The p-Block Elements Three Marks Questions - by Archana Singh - Deoghar - View & Read

  • 1)

    Account for the following:
    (i) BiCl3 is less covalent than PCl3
    (ii) O3 acts as a powerful oxidising agent.
    (iii) F2 is a stronger oxidising agent than Cl2

  • 2)

    Give reasons for each of the following:
    (a) Sulphur in vapour state exhibits paramagnetic behaviour.
    (b) Hydrogen fluoride is weaker acid than hydrogen chloride in water.
    (c) NH3 has higher proton affinity than PH3.

  • 3)

    Describe the following about halogen family (group 17 elements):
    (i) Relative oxidising power
    (ii) Relative strength of their hydrides
    (iii) Oxyacids and their related oxidising ability.

CBSE Class 12th Chemistry - The d- and f- Block Elements Three Marks Questions - by Archana Singh - Deoghar - View & Read

  • 1)

    How would you account for the following?
    (I) The Eo value for the Mn3+ /Mn2+ couple is much more positive than that for the Cr3+/Cr2+ couple or Fe 3+/Fe 2+ couple.
    (ii) The highest oxidation state of a metal is exhibited in its oxide or fluoride.
    (iii) The atomic radii ofthe metals ofthe third (Sd) series of transition elements are virtually the same as those of the corresponding members of the second (4d) series.

  • 2)

    Explain the following:
    (i) The transition elements have great tendency for complex formation.
    (ii) There is a gradual decrease in the atomic sizes of transition elements in a series with increasing atomic numbers.
    (iii) Lanthanum and Lutetium do not show colouration in solutions.
    (At. No : La = 57, Lu = 71)

  • 3)

    Give reasons for each of the following:
    (i) Transition metal fluorides are ionic in nature, whereas bromides and chlorides are usually covalent in nature.
    (ii) Size of trivalent lanthanoid cations decreases with increase in the atomic number.
    (iii) Chemistry of all the lanthanoids is quite similar.

  • 4)

    Identify the first row transition metal ions which have outer electronic configurations of 3d4 and 3d6 and describe their oxidation states. 

  • 5)

    Complete the following reactions:
    (a) Cr2O72-+14H++6e-\(\longrightarrow \).......... +7H2O
    (b) CrO42-+..........\(\rightleftharpoons \)........\(\rightleftharpoons \).......+H2O
    (c) MnO4-+2H2O+3e\(\overrightarrow { medium } \)........ +4OH-

CBSE Class 12th Chemistry - Coordination Compounds Three Marks Questions - by Archana Singh - Deoghar - View & Read

  • 1)

    Write the name, the state of hybridization, the shape and the magnetic behaviour of the following complexes:
    [Ni(CN)4]2-, [CoCl4]2-,[Cr(H2O)2(C2O4)2]- (At. No: Co = 27, Ni = 28, Cr = 24)

  • 2)

    Give the formula of each of the following coordination entities:
    (i) Co3+ ion is bound to one Cl-, one NH3 molecule (en) molecules.
    (ii) Ni2+ ion is bound to two water molecules and two oxalate ions.
    Write the name and magnetic behaviour of each of the above coordination entities.
    (At. nos. Co = 27, Ni = 28)

  • 3)

    Write the name, stereochemistry and magnetic behaviour of the following:
    (At. nos. Mn = 25, Co = 27, Ni = 28)
    (i) K4(Mn(CN)6]
    (ii) [Co(NH3)5Cl]Cl2
    (iii) K2[Ni(CN)4]

  • 4)

    (a) Square planar complexes (of MX2Ltype) with coordination number of 4 exhibits geometrical isomerism whereas tetrahedral complexes with similar composition do not. Why?
    (b) Describe the type of hybridization, shape and magnetic property of [Co(NH3)4Cl2]Cl. [Give: At. no.of Co=27]

CBSE Class 12th Chemistry - Haloalkanes and Haloarenes Three Marks Questions - by Archana Singh - Deoghar - View & Read

  • 1)

    Complete the following reactions:
    (i) \(CH_3CH_2OH \xrightarrow{SOCI_2}'A' \xrightarrow{KCN} 'B'\)
    (ii) \(CH_3-CH-CH_3 \xrightarrow{PCl_5} 'A' \xrightarrow{AgCN}'B' \\ \quad \quad \quad \ \ | \\ \quad \quad \quad \ OH\)
    (iii) \(CH_3CH_2\xrightarrow{AgNO_2}\ 'A'\)
    (iv) (CH3)2CHCI + CH \(\equiv \) CNa \(\rightarrow\) 'A'
    (v) \(CH_3CH_2CH_2Cl+CH_3​​​​​​​COOAg \rightarrow \ 'A' +\ 'B'\)
    (vi) \(2(CH_2)_2CHBr + 2Na \xrightarrow{dry \ ether}\)  

  • 2)

    An organic compound 'A' having molecular formula C4H8 on treatment with dil.  H2SO4 gives 'B'. 'B' on treatment with conc.  HCl and anhydrous ZnCl2 gives 'C' and on treatment with sodium ethoxide gives back 'A'. Identify the compounds 'A', 'B' and 'C' and write the equations involved.

  • 3)

    An alkyl chloride (A), on reaction with magnesium in dry ether followed by treatment with ethanol gave 2-methylbutane.  Write all the possible structures of (A).

  • 4)

    Explain why
    (a) vinyl chloride is unreactive in nucleophilic substitution reactions?
    (b) neo-pentyl bromide undergoes nucleophilic substitution reaction very slowly?
    (c) 3-bromocyclohexene is more reactive than 4-bromocyclohexene in hydrolysis with aqueous NaOH?
    (d) tert-butyl chloride reacts with aqueous sodium hydroxide by SN1 mechanism while n-butyl chloride reacts by SN2 mechanism?

CBSE Class 12th Chemistry - Alcohols , Phenols and Ethers Three Marks Questions - by Archana Singh - Deoghar - View & Read

  • 1)

    Draw the structure and name the product formed if the following alcohols are oxidized. Assume that an excess of oxidizing agent is used.
    (i) CH3CH2CH2CH2OH
    (ii) 2-butenol
    (iii) 2-methyl-1-propanol

CBSE Class 12th Chemistry - Aldehydes , Ketones and Carboxylic Acids Three Marks Questions - by Archana Singh - Deoghar - View & Read

  • 1)

    (a) Write the step and conditions involved in the following conversions:
     (i) Acetophenone to 2-phenyl-2-butanol
    (ii) Propene to acetone
    (b) Describe simple chemical tests to distinguish between the following pairs of compounds:
     Diethyl ether and Propanol

  • 2)

    How will you carry out the following conversions:
    (i) Acetone to chloroform?
    (ii) Acetylene to Acetic acid?

  • 3)

    Why is carboxyl group in benzoic acid meta directing?  Support your answer with two examples.

  • 4)

    An aliphatic compound 'A' with a molecular formula of C3H6O reacts with phenylhydrazine to give compound 'B'.  Reaction of 'A' with I2 in alkaline medium on warming, gives a yellow precipitate 'C'.  Identify the compounds A, B and C.

  • 5)

    Write chemical reactions to affect the following transformations:
    (a) Butan-1-ol to butanoic acid
    (b) Benzyl alcohol to phenylethanoic acid
    (c) 3-Nitrobromobenzene to 3-nitrobenzoic acid
    (d) 4-Methylacetophenone to benzene-1, 4-di-carboxylic acid
    (e) Cyclohexene to hexane-1,6-dioic acid
    (f) Butanal to butanoic acid.

CBSE Class 12th Chemistry - Organic Compounds Containing Nitrogen Three Marks Questions - by Archana Singh - Deoghar - View & Read

  • 1)

    An aliphatic compound 'A' with molecular formula C2H3Cl on treatment with AgCN hives two isomeric compounds of unequal amounts with the molecular formula C3H3N. The minor of nthese two products on complete reduction with Hin the presence of Ni gives a compound 'B' with  molecular formula C3H9N. Identify the compounds 'A' , 'B' and write the reactions involved.

  • 2)

    Iodomethane reacts with KCN to form a major product 'A' . Compund 'A' on reduction in presence of LiAIH4 forms a higher amine 'B'. Compound B on treatment with CuCl2 frms a blue colour complex 'C'. Identify the compounds 'A', 'B'  and 'C' .

  • 3)

    An organic compound 'A' having molecular formula C2H5O2N reacts with HNO2 and gives'B' C2H4O3N2. On reduction, 'A' gives a compound 'C' on treatment with HNO gives 'D' which gives positive iodoform test. Identify 'A' .

CBSE Class 12th Chemistry - Biomolecules Three Marks Questions - by Archana Singh - Deoghar - View & Read

  • 1)

    How can reducing and non-reducing sugars be distinguished? Mention the structural feature characterizing reducing sugars

  • 2)

    Explain the following terms
    (i) Mutarotation
    (ii) Avitaminosis

  • 3)

    Answer the following questions briefly:
    (i) What are reducing sugars?
    (ii) What is meant by denaturation of a protein?
    (iii) How is oxygen replenished in our atmosphere?

  • 4)

    Name the main diseases caused due to lack of vitamin and its source in each of the following: A,B6 and E6 and E.

  • 5)

    Define the following terms:
    (i) Co-enzymes
    (ii) Mutation in biomolecules
    (iii) Nucleotides

CBSE Class 12th Chemistry - Polymers Three Marks Questions - by Archana Singh - Deoghar - View & Read

  • 1)

    Explain the following terms giving a suitable example for each :
    (i) Elastomers
    (ii) Condensation polymers
    (iii) Addition polymers

  • 2)

    Write chemical equations for the synthesis of 
    (a) terylene
    (b) neoprene
    (c) teflon

  • 3)

    Giving one example each of 
    (i) addition polymers,
    (ii) condensation polymers,
    (iii) copolymers.

  • 4)

    Which of the following polymers soften on heating and harden on cooling? What are the polymers with this property collectively called? What are the structural similarities between such polymers Bakelite, urea-formaldehyde resin, polythene, polyvinyl, polystyrene.

CBSE Class 12th Chemistry - Chemistry in Everyday Life Three Marks Questions - by Archana Singh - Deoghar - View & Read

  • 1)

    (a) Differentiate between disinfectant and an antiseptic. Give one example of each.
    (b) What is tincture of iodine and what is it used for? 

  • 2)

    What are biodegradable and non-biodegradable detergents? Give one example of each.

  • 3)

    Mention the action of the following on the human body in bringing relief from a disease.
    (i) Brompheniramine
    (ii) Aspirin
    (iii) Equalin

  • 4)

    Explain the following types of substances with one suitable example, for each case:
    (i) Cationic detergents
    (ii) Food preservatives.
    (iii) Analgesics.

  • 5)

    What are detergents? How are they classified? Why are detergents usually preferred to soaps for washing clothes? Give an example of detergents.

CBSE Class 12 Chemistry - General Principles and Processes of Isolation of Elements Three Marks Questions - by Archana Singh - Deoghar - View & Read

  • 1)

    How can  you obtain pure alumina form a bauxite ore associated with silica? Write the reactions involved in this process.

  • 2)

    Write the reactions involved in the following processes:
    (i) Leaching of bauxite ore to prepare pure alumina.
    (ii) Refining of zirconium by van Arkel method.
    (iii) Recovery of gold after gold ore has been leached with NaCN solution. 

  • 3)

    State the role of 
    (i) depressant in froth floatation process.
    (ii) silica in the metallurgy of copper.
    (iii) graphite rod in the electrolytic reduction of alumina.

  • 4)

    Describe the role of the following:
    (i) NaCN in the extraction of silver from a silver ore.
    (ii) Iodine in the refining of titanium
    (iii) Cryolite in the metallurgy of aluminium

  • 5)

    Write down the reactions taking place in different zones in the blast furnace during the extraction of iron.

CBSE 12th Standard Chemistry - General Principles and Processes of Isolation of Elements Five Mark Question Paper - by Sudha George - Kochi - View & Read

  • 1)

    State the role of 
    (i) depressant in froth floatation process.
    (ii) silica in the metallurgy of copper.
    (iii) graphite rod in the electrolytic reduction of alumina.

  • 2)

    The reaction:
    \({ Cr }_{ 2 }{ O }_{ 3 }+2Al\rightarrow { Al }_{ 2 }{ O }_{ 3 }+2Cr;\ \triangle { G }^{ ° }=-421\quad kJ\) 
    is thermodynamically feasible as is apparent from Gibbs energy value. Why does it not take place at room temperature?

  • 3)

    Explain the role of
    (i) cryolite in the electrolytic reduction of alumina.
    (ii) carbon monoxide in the purification of nickel.

  • 4)

    (i) Name the method of refining of metals such as germanium.
    (ii) In the extraction of Al, impure \({ Al }_{ 2 }{ O }_{ 3 }\) is dissolved in conc. NaOH to form sodium aluminate and leaving impurities behind. What is the name of this process?
    (iii) What is the role of coke in the extraction of iron from its oxides?

12th Standard CBSE Chemistry - Surface Chemistry Five Mark Question Paper - by Sudha George - Kochi - View & Read

  • 1)

    Explain the following:
    (a) Same substance can act both as colloids and crystalloids.
    (b) Artifical rain is caused by spraying salt over clouds.
    (c) When a beam of light is passed through a colloidal sol, the path of the beam gets illuminated.

  • 2)

    What is demulsification? Name two demulsifiers.

  • 3)

    Explain the following terms:
    (i) Electrophoresis
    (ii) Coagulation
    (iii) Dialysis                        
     (v) Tyndal effect

  • 4)

    Write one difference in each of the following:
    (i) Lyophobic sol and lyophilic sol.
    (ii) Solution and colloid.
    (iii) Homogeneous catalysis and heterogeneous catalysis

  • 5)

    (i) In reference to Freundlich's adsorption isotherm, write the expression for adsorption of gases on solids in the form of an equation.
    (ii) Write an important characteristic of lyophilic sols.
    (iii) Based on the type of particles of dispersed phase, give one example each of associated colloid and multimolecular colloid.

12th Standard CBSE Chemistry - Chemical Kinetics Five Mark Question Paper - by Sudha George - Kochi - View & Read

  • 1)

    (a) what is meant by rate of a reaction.
    (b) In a pseudo first order hydrolysis of ester in water, the following results are obtained:

    t in seconds 0 30 60 90
    [Ester] M 0.55 0.31 0.17 0.085

    (i) Calculate the average rate of reaction between the time interval 30 to 60 seconds.
    (ii) Calculate the pseudo first order rate constant for the hydrolysis of ester.

  • 2)

    (a) Define the following:
    (i) Order of a reaction
    (ii) Elementary step in a reaction
    (b) A first order reaction has a rate constant value of 0.00510 min-1.  If we begin with 0.10 M concentration of the reactant, how much of the reactant will remain after 3.0 hours?

  • 3)

    (a) Explain the following terms:
    (i) Order of a reaction
    (ii) Molecularity of a reaction
    (b) The rate of a reaction increases four times when the temperature changes from 300 k to  320 K.  Calculate the energy of activation of the reaction, assuming that it does not change with temperature.
    (R = 8.314 J K-1 mol-1)

  • 4)

    The half-life for decay of radioactive 14C is 5730 years.  An archaeological artefact containing wood had only 80% of 14 C activity as found in a living tree.  Calculate the age of the artefact

  • 5)

    Describe briefly the dependence of reaction rate of a chemical reaction on temperature. Explain the effect of temperature on the rate constant of a reaction.

12th CBSE Chemistry - Electrochemistry Five Mark Question Paper - by Sudha George - Kochi - View & Read

  • 1)

    (a) Define molar conductivity of a solution and explain how molar conductivity changes with change in concentration of solution for a weak and a strong electrolyte.
    (b) The resistance of a conductivity cell containing 0.001 M KCl solution at 298 K is 1500 \(\Omega \). What is the cell constant if the conductivity of 0.001 M KCl solution at 298 K is \(0.146\times { 10 }^{ -3 }S{ cm }^{ -1 }\)?

  • 2)

    Three electrolytic cells A, B, C containing solutions of ZnSO4, AgNO3, and CuSO4, respectively are connected in series. A steady current of 1.5 amperes was passed through them until 1.45 g of silver deposited at the cathode of cell B. How long did the current flow? What mass of copper and zinc were deposited?

  • 3)

    Tarnished silver contains Ag2 S. Can this tarnish be removed by placing tarnished silver ware in an aluminium pan containing an inert electrolytic solution such as NaCl ? The standard electrode potential for the half reactions are : For \({ Ag }_{ 2 }S(s)+{ 2e }^{ - }\longrightarrow 2Ag(s)+{ S }^{ 2- },\) it is - 0.71 V and for \({ Al }^{ 3+ }+{ 3e }^{ - }\longrightarrow Al(s)\), it is - 1.66 V

  • 4)

    Three iron sheets have been coated separately with three metals (A, B and C) whose standard electrode potentials are given below :

    Metal A B C Iron
    E0value - 0.46 - 0.66 V - 0.20 V - 0.44 V

    Identify in which case rusting will take place faster when coating is damaged.

  • 5)

    A voltaic cell is set up at \(25°\)C with the half cells Ag+(0.001 M)Ag and Cu2+(0.10M)Cu. What should be its cell potential?

12th Standard CBSE Chemistry - Solution Five Mark Question Paper - by Sudha George - Kochi - View & Read

  • 1)

    Alternative definition of Henry's law

  • 2)

    Konowaloff's rule.

  • 3)

    How many grams of sucrose (M.wt. = 342) should be dissolved in 100g water in order to produce a solution with 105.0°C difference between the freezing point and the boiling point ?(Kf = 1.86°C/m, Kb = 0.51°C/m)

  • 4)

    (a)  Differentiate between molarity and molality for a solution. How does a change in temperature influence their values?
    (b) Calculate the freezing point of an aqueous solution containing 10.50 g of MgBr2 = 184 g) (K f for water 1.86 K kg mol-1)

12th CBSE Chemistry - The Solid State Five Mark Question Paper - by Sudha George - Kochi - View & Read

  • 1)

    Predict the structure of MgO crystal and coordination number of its cation in which cation and anion radii are equal to 65 pm and 140 pm respectively.

  • 2)

    Calculate the approximate numbert of unit cells present in 1 g of ideal NaCl crystals.

  • 3)

    What are ferromagnets and ferro-electrics? Give on example of each case.

  • 4)

    What are point defects in crystals and how do they develop?

CBSE 12th Chemistry - Surface Chemistry Three Marks Questions - by Archana Singh - Deoghar - View & Read

  • 1)

    Write three features of chemisorptions which are not found in physisorptions. Illustrate your answer with suitable examples.

  • 2)

    (a) How can a colloidal solution and true solution of the same colour be distinguished from each other?
    (b) List four applications of adsorption.

  • 3)

    (a) Which will be adsorbed more readily on the surface of charcoal and why? NH3 or CO2 
    (b) In what way are multimolecular colloids different from macromolecular colloids? Give one example of each,
    (c) State the principle of electrodialysis.

     

  • 4)

    (a) In which of the following does adsorption take place and why?
    (i) Silica gel placed in the atmosphere saturated with water.
    (ii) Anhydrous CaCl2 placed in the atmosphere saturated with water.
    (b) How does BF3 act as a catalyst in industrial process?
    (c) Give an example of shape-selective catalysis.

  • 5)

    Define the following:
    (i) Cationic detergents
    (ii) Broad spectrum antibiotics
    (iii) Tranquilisers

CBSE 12th Chemistry - Chemical Kinetics Three Marks Questions - by Archana Singh - Deoghar - View & Read

  • 1)

    A first order reaction takes 100 minutes for completion of 60% of the reaction.  Find the time when 90% of the reaction will be completed.

  • 2)

    In general it is observed that the rate of chemical reaction doubles with every 10o rise in temperature.  If the generalization holds good for the reaction in the temperature range 295 K to 305 K, what would be the value of activation energy for the this reaction?
    (R = 8.314 J mol-1  K-1)

  • 3)

    The reaction,
    N2(g) + O2(g) \(\rightleftharpoons \)2NO(g)
    Contributes to air pollution whenever a fuel is burnt in air at a high temperature.  At 1500 K, equilibrium constant K for it is 1.0 \(\times\) 10-5.  Suppose in a case [N2] = 0.80 mol L-1and [O2] = 0.20 mol L-1 before any reaction occurs. Calculate the equilibrium concentration of the reactants and the product after the mixture has been heated to 1500 K.

  • 4)

    The half-life for the decomposition of nitramide is 2.1 hour at 15 oC
    NH2NO2(aq) \(\to\) N2O (g) + H2O(l)
    If 6.2 g of NH2NO2 is allowed to decompose, calcualte
    (i) time taken for NH2NO2 to decompose 99%
    (ii) volume of N2O (dry) produced at STP.

  • 5)

    The half-life for the reaction:
    N2O5 \(\to\) 2NO2 + \(1\over2\)O2 is 2.4 hours at 30 oC.
    (a) Starting with 100 grams of N2Os, how many gram will remain after 9.6 hours?
    (b) What time would be required to reduce 5 \(\times\) 1010 molecules of N2O5 to 108 molecules?

12th Standard CBSE Chemistry - Electrochemistry Three Marks Questions - by Archana Singh - Deoghar - View & Read

  • 1)

    An aqueous solution of copper sulphate, CuSO4 was electrolysed between platinum electrodes using a current of  0.1287 ampere for 50 minutes. (Atomic mass of Cu=63.5 g mol-1].
    (a) Write the cathodic reaction.
    (b) Calculate:
    (i) Electric charge passed during electrolysis.
    (ii) Mass of copper deposited at the cathode
    [Given : 1F = 96,500 C mol-1]

  • 2)

    For the cell
    \(Zn(s)\left| { Zn }^{ 2+ }(2M) \right| \left| { Cu }^{ 2+ }(0.5M) \right| Cu(s)\)
    (a) Write equation for each half-reaction.
    (b) Calculate the cell potential at 25 oC.
    [Given: \({ E }_{ { Zn }^{ 2+ }/Zn }^{ o }=-0.76V;{ E }_{ { Cu }^{ 2+ }/Cu }^{ o }=+0.34V\)]

  • 3)

    The Eo values at 298 K corresponding to the following two reduction electrode processes are:
    (i) Cu+/Cu = +0.52 V
    (ii) Cu2+/Cu+ = +0.16 V
    Formulate the galvanic cell for their combination. What will be the cell potential? Calculate the \({ \triangle }_{ r }{ G }^{ o }\) for the cell reaction.(1F = 96500 C mol-1).

  • 4)

    Calculate the standard cell potentials of galvanic cell in which the following reactions take place:
    (i) \(2Cr(s)+{ 3Cd }^{ 2+ }(aq)\rightarrow { 2Cr }^{ 3+ }(aq)+3Cd(s)\)
    (ii) \({ Fe }^{ 2+ }(aq)+{ Ag }^{ + }(aq)\rightarrow { Fe }^{ 3+ }(aq)+Ag(s)\)
    Calculate the \({ \triangle }_{ r }{ G }^{ o }\) and equilibrium constant for the reactions.

  • 5)

    A solution of CuSO4 is electrolysed for 10 minutes with a current of 1-5 amperes. What is the mass of copper deposited at the cathode? (Molar mass of Cu = 63.5 g mol-1)

12th CBSE Chemistry - Chemistry in Everyday Life Two Marks Questions - by Archana Singh - Deoghar - View & Read

  • 1)

    A solution of urea in water has a boiling point of 373.128 K. Calculate the freezing point of the same solution. [Given: For water, K f = 1.86 Km-1, Kb = 0.52 Km-1]

  • 2)

    0.1 mole of acetic acid was dissolved in 1 kg of benzene. Depression in freezing point of benzene was determined to be 0.256 K. what conclusion can you draw about the state of the solute in solution? [Given: Kf for benzene = 5.12 K/m]

  • 3)

    A 0.1539 molal aqueous solution of cane sugar (mol. mass = 342 g mol -1) has a freezing point of 271 K while the freezing point of pure water is 273.15 K. what will be the freezing point of an aqueous solution containing 5 g of glucose (mol. mass = 180 g mol-1) per 100 g of solution?

  • 4)

    The vapour pressure of water is 12.3 kPa at 300 K. Calculate vapour pressure of 1 molal solution of a non-volatile solute in it.

  • 5)

    What role does the molecular interaction play in solution of alcohol and water?

12th Standard CBSE Chemistry Unit 1 The Solid State Three Marks Questions - by Archana Singh - Deoghar - View & Read

  • 1)

    Explain the following properties given suitable examples:
    (i) Ferromagnetism;
    (ii) Paramagnetism;
    (iii) Ferrimagnetism.

  • 2)

    Silver crystallizes in face-centred cubic unit cell. Each side of this unit cell has a length of 400 pm. Calculate the radius of the silver atom. (Assume the atoms just touch each other on the diagonal across the face of the unit cell. That is , each face atom is touching the four corner atoms.)

  • 3)

    The well-known mineral fluorite is chemically calcium fluoride. It is known that in one unit cell of this mineral, there are 4 Ca2+ ions and 8 Fions and that Ca2+ ions are arranged in a fcc lattice. The F- ions fill all tetrahedral holes in the face-centred cubic lattice of Ca2+ ions. The edge of the unitcell is 5.46\(\times \)10-8 cm in length. The density of the solid is 3.18 g cm-3. Use this information to calculate Avogadro's number. (Molar mass of CaF= 78.08 G mol-1)

  • 4)

    Calculate the efficiency of packing in case of a metal crystal for
    (a) simple cubic,
    (b) body-centred cubic, and
    (c) face-centred cubic (with the assumption that atoms are touching each other).

  • 5)

    What makes a glass different from a solid such as quartz? Under what conditions could quartz be converted into glass?

12th Standard CBSE Chemistry - Chemistry in Everyday Life Two Marks Questions - by Archana Singh - Deoghar - View & Read

  • 1)

    Name the medicines used for the treatment of the following diseases:
    (i) Tuberculosis
    (ii) Typhoid

  • 2)

    Why do we need to classify drugs in different ways?

12th Standard CBSE Chemistry - Polymers Two Marks Questions - by Archana Singh - Deoghar - View & Read

  • 1)

    What is the role of benzoyl peroxide in addition polymerisation of alkenes ?

  • 2)

    Name the sub-groups into which polymers are classified on the basis of magnitude of intermolecular forces.

  • 3)

    How are polymers classified on the basis of structure ?

  • 4)

    Classify the following as addition and condensation polymers : Terylene, Bakelite, Polyvinyl chloride, polythene.

  • 5)

    What are polymers?Why are they called macromolecules?

View all

CBSE Education Study Materials

CBSEStudy Material - Sample Question Papers with Solutions for Class 12 Session 2020 - 2021

Latest Sample Question Papers & Study Material for class 12 session 2020 - 2021 for Subjects Biology, Physics, Maths, Accountancy, Introductory Micro and Macroeconomics, Business Studies, Economics, Computer Science, Geography, English, History, Indian Society, Physical Education, Sociology, Tamil, Bio Technology, Engineering Graphics, Entrepreneurship, Hindi Core, Hindi Elective, Home Science, Legal Studies, Political Science, Psychology in PDF form to free download [ available question papers ] for practice. Download QB365 Free Mobile app & get practice question papers.

More than 1000+ CBSE Syllabus Sample Question Papers & Study Material are based on actual Board question papers which help students to get an idea about the type of questions that will be asked in Class 12 Final Board Public examinations. All the Sample Papers are adhere to CBSE guidelines and its marking scheme , Question Papers & Study Material are prepared and posted by our faculty experts , teachers , tuition teachers from various schools in Tamilnadu.

Hello Students, if you like our sample question papers & study materials , please share these with your friends and classmates.

Related Tags